1. Trang chủ
  2. » Mẫu Slide

Các phương pháp chứng minh bất đẳng thức - Nguyễn Tất Thu - TOANMATH.com

174 6 0

Đang tải... (xem toàn văn)

Tài liệu hạn chế xem trước, để xem đầy đủ mời bạn chọn Tải xuống

THÔNG TIN TÀI LIỆU

Nội dung

Đến đây thì bất đẳng thức 9 đã được chứng minh hay bước 2 được giải quyết hoàn toàn.. Bước 3 được giải quyết hoàn toàn.[r]

(1)Mục lục Các bất đẳng thức cổ điển Bất đẳng thức AM - GM I Bất đẳng thức AM - GM II Một số ví dụ áp dụng III Bài tập Bất đẳng thức Cauchy - Schwarz I Bất đẳng thức Cauchy-Schwarz dạng đa thức II Bất đẳng thức Cauchy-Schwarz dạng phân thức III Các ví dụ minh họa IV Bài tập Một số bất đẳng thức khác I Bất đẳng thức Schur Bất đẳng thức Schur Các trường hợp đặc biệt Bất đẳng thức Schur mở rộng Các ví dụ II Bất đẳng thức Holder Bất đẳng thức Holder Trường hợp đặc biệt Ví dụ minh họa III Bất đẳng thức Chebyshev Bất đẳng thức Chebyshev Ví dụ minh họa IV Bài tập Phương pháp quy nạp I Lý thuyết II Ví dụ minh họa Phương pháp phân tích bình phương SOS I Lý thuyết Một số tiêu chuẩn đánh giá Một số biểu diễn sở II Các ví dụ III Bài tập Phương pháp dồn biến I Lý thuyết II Ví dụ minh họa III Bài tập Các phương pháp chứng minh bất đẳng thức Phương pháp p, q, r I Lý thuyết Bất đẳng thức Schur 3 13 19 19 19 19 27 31 31 31 31 31 31 34 34 34 34 35 35 36 36 38 38 38 42 42 42 42 43 46 48 48 48 51 đại 53 54 54 54 (2) MỤC LỤC 2 Một số biểu diễn đa thức đối xứng ba biến qua p, q, r Một số đánh giá p, q, r II Một số ví dụ III Bài tập Phương pháp sử dụng tiếp tuyến và cát tuyến I Lý thuyết Hàm lồi - Dấu hiệu hàm lồi Bất đẳng thức tiếp tuyến - Bất đẳng thức cát tuyến II Các ví dụ minh họa III Bài tập Một số chuyên đề Ứng dụng điều kiện có nghiệm phương trình bậc ba đẳng thức I Lý thuyết Mở đầu Một số kết II Ví dụ minh họa III Bài tập Bài toán tìm số tốt bất đẳng thức I Lý thuyết II Ví dụ minh họa III Bài tập Các 54 55 55 56 58 58 58 58 59 66 68 chứng minh bất 68 68 68 68 70 74 75 75 75 82 bất đẳng thức cổ điển 86 Bất đẳng thức AM-GM 86 Bất đẳng thức Cauchy-Schwarz 109 Một số bất đẳng thức khác 124 Một số phương pháp chứng minh bất đẳng thức Phương pháp quy nạp Phương pháp phân tích bình phương SOS Phương pháp dồn biến Phương pháp p, q, r Phương pháp tiếp tuyến và cát tuyến 129 129 130 135 148 150 Một số chuyên đề 156 Ứng dụng kiện có nghiệm phương trình bậc ba 156 Bài toán tìm số tốt 159 (3) Chương Các bất đẳng thức cổ điển §1 Bất đẳng thức AM - GM Bất đẳng thức AM − GM là bất đẳng thức cổ điển sử dụng nhiều các bài toán chứng a1 + a2 + · · · + an minh bất đẳng thức Ta biết trung bình cộng nsố thực a1 ,a2 , · · · ,an là số n √ √ và trung bình nhân n số đó là n a1 a2 · · · an (với điều kiện là n a1 a2 · · · an tồn tại) Bất đẳng thức AM − GM cho chúng ta đánh giá trung bình cộng các số thực không âm và trung bình nhân chúng Cụ thể sau: I Bất đẳng thức AM - GM Định lí Cho n số thực không âm a1 , a2 , · · · , an ta có √ a1 + a2 + · · · + an ≥ n a1 · a2 · · · an n Đẳng thức xảy a1 = a2 = · · · = an Chứng minh Có nhiều cách đề chứng minh bất đẳng thức AM − GM , đây ta chứng minh bất đẳng thức AM − GM phương pháp quy nạp Trước hết ta chứng minh bất đẳng thức AM − GM cho trường hợp n = Tức là, cần chứng minh a1 + a2 √ ≥ a1 · a2 Bất đẳng thức này tương đương với √ √ √ a1 + a2 ≥ a1 a2 ⇔ ( a1 − a2 ) ≥ Bất đẳng thức cuối hiển nhiên đúng Đẳng thức xảy a1 = a2 Tiếp theo ta chứng minh cho trường hợp n = Tức là cần chứng minh √ a1 + a2 + a3 + a4 ≥ a1 · a2 · a3 · a4 Áp dụng trường hợp n = ta có và a1 + a2 √ ≥ a1 · a2 a3 + a4 √ ≥ a3 · a4 Do đó a1 + a2 + a3 + a4 = a1 + a2 a3 + a4 √ √ + a1 a2 + a3 a4 √ 2 ≥ ≥ a1 a2 a3 a4 2 (4) BẤT ĐẲNG THỨC AM - GM Nên trường hợp n = chứng minh Tiếp đến ta chứng minh trường hợp n = 3, tức là chứng minh √ a1 + a2 + a3 ≥ a1 · a2 · a3 Đặt a4 = a1 + a2 + a3 Áp dụng cho trường hợp n = ta có √ a1 + a2 + a3 + a4 ≥ a1 · a2 · a3 · a4 , hay a1 + a2 + a3 r a1 + a2 + a3 ≥ a1 · a2 · a3 · a1 + a2 + a3 + Suy √ a1 + a2 + a3 ≥ a1 · a2 · a3 (đpcm) Để chứng minh cho trường hợp tổng quát ta chứng minh theo hai bước sau: Bước 1: Ta chứng minh bất đẳng thức đúng với n = 2m +) Với m = 1, ta có n = 2nên bất đẳng thức đúng với m = +) Giả sử bất đẳng thức đúng với n = 2m−1 , ta chứng minh bất đẳng thức đúng với n = 2m Tức là √ a1 + a2 + · · · + a2m−1 + · · · + an ≥ n a1 a2 · · · an (1) n Đặt a2m−1 +1 + a2m−1 +2 + · · · + a2m a1 + a2 + · · · + a2m−1 ,y= x= m−1 2m−1 Theo giả thiết quy nạp ta có √ √ m−1 x≥ a1 a2 · · · a2m−1 ,y ≥ 2m−1 a2m−1 +1 · · · an Áp dụng cho trường hợp n = ta có: x+y √ ≥ xy hay √ a1 + a2 + · · · + a2m−1 + a2m−1 +1 + · · · + an m ≥ a1 a2 · · · an m Hay (1) chứng minh Bước 2: Ta chứng minh bất đẳng thức đúng với n ≥ thì đúng với n − Gải sử √ a1 + a2 + · · · + an ≥ n a1 a2 · · · an n Ta chứng minh a1 + a2 + · · · + an−1 √ ≥ n−1 a1 · a2 · · · an−1 n−1 a1 + a2 + · · · + an−1 Thật vậy: Đặt an = ÁP dụng bất đẳng thức AM-GM cho n số ta có n−1 √ a1 + a2 + · · · + an ≥ n a1 a2 · · · an , n hay a1 + a2 + · · · + a1 + a2 + · · · + an−1 r a1 + a2 + · · · + an−1 n−1 ≥ n a1 a2 · · · an−1 · n n−1 (5) BẤT ĐẲNG THỨC AM - GM Suy a1 + a2 + · · · + an−1 ≥ n−1 √ n−1 a1 · a2 · · · an−1 (đpcm) Từ hai bước trên ta có bất đẳng thức AM − GM chứng minh Hệ Cho các số thực dương a1 ,a2 , · · · ,an Ta có 1 n2 + + ··· + ≥ a1 a2 an a1 + a2 + · · · + an Đẳng thức xảy a1 = a2 = · · · = an II Một số ví dụ áp dụng Ví dụ 1.1 Cho a,b,c > thỏa a2 + b2 + c2 = Chứng minh a5 + b5 + c5 ≥ Áp dụng bất đẳng thức AM-GM ta có a5 + a5 + + + ≥ 3a2 hay 2a5 + ≥ 3a2 Tương tự 2b5 + ≥ 3b2 và 2c5 + ≥ 3c2 Cộng ba bất đẳng thức trên ta có đpcm Nhận xét Ta có bài toán tổng quát sau: Cho a,b,c > thỏa mãn a + b + c = (hoặc abc = 1) và m,n ∈ N,m ≥ n Khi đó am + bm + cm ≥ an + bn + cn (1) Bất đẳng thức (1) còn đúng m,n là các số hữu tỉ dương Và ta có thể tổng quát biến thành k biến Ví dụ 1.2 Cho a,b,c > thỏa a + 4b + 9c = 6.Chứng minh a3 + b3 + c3 ≥ Xét x, y, z là các số thực dương Áp dụng bất đẳng thức AM-GM ta có a3 + 2x3 = a3 + x3 + x3 ≥ 3x2 a, đẳng thức xảy a = x Tương tự ta có: b3 + 2y ≥ 3y b, c3 + 2z ≥ 3y c Đẳng thức xảy b = y, c = z Cộng các bất đẳng thức trên theo vế ta a3 + b3 + c3 ≥ 3(x2 a + y b + z c) − 2(x3 + y + z ) (6) BẤT ĐẲNG THỨC AM - GM Ta chọn x, y, z cho    x=     x + 4y + 9z = a + 4b + 9c =   2 ⇒ y=   x = y = z = t2     z = Do đó a3 + b3 + c3 ≥ 3t2 (a + 4b + 9c) − 2(x3 + y + z ) = Ví dụ 1.3 Cho a, b, c > thỏa ab + bc + ca = Chứng minh a3 + b3 + c3 ≥ Áp dụng bất đẳng thức AM-GM ta có a3 + b3 + ≥ 3ab b3 + c3 + ≥ 3bc c3 + a3 + ≥ 3ca Cộng ba bất đẳng thức trên ta có đpcm Ví dụ 1.4 Cho các số thực dương a, b, c có tổng bình phương Chứng minh ab bc ca + + ≥ c a b Gọi P là vế trái bất đẳng thức cần chứng minh, ta có  2 ab bc ca P = + + c a b 2 2 ab cb c2 a2 = + + + 2(a2 + b2 + c2 ) c a b      2 2 c2 b2 c2 a2 a2 b2 c2 a2 ab cb = + + + + + +6 c2 a a2 b c2 b ≥ b2 + c2 + a2 + = Suy P ≥ Đẳng thức xảy a = b = c = Ví dụ 1.5 Cho a, b, c > và a + b + c = abc Chứng minh : a b c + + ≥ b c a Ta có bất đẳng thức cần chứng minh tương đương với:   a b c abc + + ≥ a + b + c b c a (7) BẤT ĐẲNG THỨC AM - GM Hay a2 c b2 a c2 b + + ≥ a + b + c b2 c a (1) Áp dụng bất đẳng thức Cô si cho ba số ta : r 2 a2 c b2 a a c b a + + c ≥ .c = 3a b2 c2 b2 c2 Tương tự : b2 a c2 b c2 b a2 c + + a ≥ 3b ; + + b ≥ 3c c2 a2 a2 b Cộng ba bất đẳng thức trên ta có bất đẳng thức (1) Bài toán chứng minh Đẳng thức xảy ⇔ a = b = c = √ Ví dụ 1.6 Cho a, b, c > Chứng minh : a5 b5 c5 + + ≥ a3 + b3 + c3 b2 c2 a2 Áp dụng bất đẳng thức Cô si : r a5 a5 + ab ≥ ab = 2a3 b2 b2 Tương tự : b5 c + bc ≥ 2b ; + ca2 ≥ 2c3 c2 a2 Công bất đẳng thức trên lại với ta :  a5 b5 c5 3 3 3 2 + + ≥ a + b + c + a + b + c − ab − bc − ca b2 c2 a2 Nên ta cần chứng minh : a3 + b3 + c3 − ab2 − bc2 − ca2 ≥ ⇔ a3 + b3 + c3 ≥ ab2 + bc2 + ca2 Áp dụng bất đẳng thức Cô si : √ a3 + b3 + b3 ≥ a3 b3 b3 = 3ab2 ⇒ a3 + 2b3 ≥ 3ab2 Tương tự : b3 + 2c3 ≥ 3bc2 ; c3 + 2a3 ≥ 3ca2 Công bất đẳng thức trên lại với ta có (1) Vậy bài toán chứng minh Ví dụ 1.7 Cho các số thực dương a,b,c Chứng minh a4 b4 c4 a+b+c + + ≥ b2 (c + a) c2 (a + b) a2 (b + c) (1) (8) BẤT ĐẲNG THỨC AM - GM Áp dụng bất đẳng thức AM-GM ta có b b c+a a4 + + + ≥ 2a b (c + a) 2 hay a4 c+a +b+ ≥ 2a b (c + a) Tương tự, ta có b4 a+b c4 b+c + c + ≥ 2b và + a + ≥ 2c c2 (a + b) a2 (b + c) Cộng ba bất đẳng thức trên theo vế ta có đpcm Ví dụ 1.8 (BĐT Nesbit cho số) Cho a, b, c > Chứng minh a b c + + ≥ b+c c+a a+b Bất đẳng thức cần chứng minh tương đương với       a b c +1 + +1 + +1 ≥ b+c c+a a+b Hay  (a + b + c) Ta có 1 + + a+b b+c c+a  ≥ 1 9 + + ≥ = a+b b+c c+a a+b+b+c+c+a (a + b + c) Nên (1) đúng Ví dụ 1.9 Cho các số thực dương a, b, c thỏa a + b + c = Chứng minh 1 1 + + + ≥ 30 a2 + b2 + c2 ab bc ca Ta có: (a + b + c)2 = ab + bc + ca ≤ 3 1 + + ≥ ab bc ca ab + bc + ca 1 + + ≥ = 2 a +b +c ab + bc + ca ab + bc + ca (a + b + c)2 Do đó + a2 + b2 + c2 ab + bc + ca 1 7 = + + + ≥ + = 30 2 a +b +c ab + bc + ca ab + bc + ca ab + bc + ca VT ≥ Ta có điều phải chứng minh (1) (9) BẤT ĐẲNG THỨC AM - GM Ví dụ 1.10 Cho các số thực dương x,y,z thỏa mãn : xy + yz + zx = 3.Chứng minh rằng: + ≥ xyz (x + y)(y + z)(z + x) Ta có: p xyz (x + y) (y + z) (z + x) ≤ x (y + z) + y (z + x) + z (x + y) = Suy xyz ≥ (x + y) (y + z) (z + x) Do đó VT ≥ xyz xyz 1 + ≥ + + ≥1+ = xyz 2xyz 2xyz 2 Bài toán chứng minh Ví dụ 1.11 (IMO 2012) Cho n ≥ và các số thực dương a2 , a3 , , an thỏa mãn a2 a3 · · · an = Chứng minh (1 + a2 )2 (1 + a3 )3 · · · (1 + an )n > nn Áp dụng bất đẳng thức AM-GM ta có k (1 + ak ) =  1 + + ··· + + ak k−1 k−1 k−1 k ≥ k k ak (k − 1)k−1 Suy (1 + a2 )2 (1 + a3 )3 · · · (1 + an )n ≥ 22 33 44 nn · · · · · a1 a2 · · · an = n n 11 22 33 (n − 1)n Ta thấy không có đẳng thức xảy Vậy bài toán chứng minh Ví dụ 1.12 Cho các số thực dương a, b, c có tích Chứng minh 1+ ≥ a+b+c ab + bc + ca Bất đẳng thức cần chứng minh tương đương với ab + bc + ca + 3(ab + bc + ca) ≥ a+b+c Áp dụng bất đẳng thức AM-GM ta có s ab + bc + ca + 3(ab + bc + ca) ≥2 a+b+c 3(ab + bc + ca)2 a+b+c (1) (10) BẤT ĐẲNG THỨC AM - GM Mặt khác (ab + bc + ca)2 ≥ 3(ab · bc + bc · ca + ca · ab) = 3abc(a + b + c) = 3(a + b + c) Suy ab + bc + ca + 3(ab + bc + ca) ≥ a+b+c Vậy bài toán chứng minh Ví dụ 1.13 (Moldova TST 2014) Cho các số thực dương a,b,c thỏa mãn abc = Chứng minh a3 + b + c3 + ab bc ca + + ≥ a2 + b2 b2 + c2 c2 + a2 Bất đẳng thức cần chứng minh tương đương với  a3 + b3 + c3 + 2ab 2bc 2ca + + ≥9 a2 + b2 b2 + c2 c2 + a2 (1) Ta có x3 + y ≥ x2 y + y x với x,y > nên a3 + b + c3 ≥ c (a2 + b2 ) b (c2 + a2 ) a (b2 + c2 ) + + 2 Suy  V T (1) ≥      c (a2 + b2 ) 2ab b (c + a2 ) 2bc a (b2 + c2 ) 2ca + + + + + +3abc ≥ a + b2 b + c2 c + a2 Bài toán chứng minh Ví dụ 1.14 Chứng minh số thực dương a,b,c ta luôn có: ab bc ca a+b+c + + ≤ a + 3b + 2c b + 3c + 2a c + 3a + 2b Ta có : ab ab ab = ≤ a + 3b + 2c (a + c) + (b + c) + 2b  1 + + a + c b + c 2b  Tương tự : bc bc ≤ b + 3c + 2a  1 + + a + b a + c 2c  ac ac , ≤ c + 3a + 2b  1 + + b + c a + b 2a  Cộng vế theo vế ta ab bc ca + + ≤ a + 3b + 2c b + 3c + 2a c + 3a + 2b  bc + ac bc + ab ab + ac + + a+b a+c b+c  + (a + b + c) 18 Hay ab bc ca 1 a+b+c + + ≤ (a + b + c) + (a + b + c) = a + 3b + 2c b + 3c + 2a c + 3a + 2b 18 10 (11) BẤT ĐẲNG THỨC AM - GM Ví dụ 1.15 Cho các số thực dương a,b,c thỏa a + b + c = Chứng minh √ ab bc ca +√ +√ ≤ c2 + a2 + b2 + Ta có (ab + bc + ca) ≤ (a + b + c)2 = ⇒ ab + bc + ca ≤ Suy 1 1 √ ≤√ =p ≤ c2 + c2 + ab + bc + ca (a + c)(b + c) Do đó: ab √ ≤ c2 +  ab ab + a+c b+c  1 + a+c b+c   Tương tự: bc √ ≤ a2 +  bc bc + a+b a+b  ca và √ ≤ b2 +  ca ca + b+a b+c  Cộng ba bất đẳng thức trên theo vế ta có: √ ab bc ca +√ +√ ≤ (a + b + c) = 2 c2 + a2 + b2 + Ví dụ 1.16 (IMO 2005) Cho các số thực dương x,y,z thỏa xyz ≥ Chứng minh x5 − x2 y5 − y2 z5 − z2 + + ≥ x5 + y + z y + z + x2 z + x2 + y Bất đẳng thức cần chứng minh tương đương với 1− ⇔ x − x2 y5 − y2 z5 − z2 + − + − ≤3 x5 + y + z y + z + x2 z + x2 + y 1 + + ≤ x5 + y + z y + z + x2 z + x + y x2 + y + z Ta có x5 + y + z ≥ Do đó (1) x4 2x4 + (y + z ) (x2 + y + z ) ≥ + y2 + z2 ≥ yz y2 + z2 y2 + z2 y2 + z2 ≤ x5 + y + z 2 (x2 + y + z )2 Chứng minh tương tự z + x2 x2 + y ≤ và ≤ y + z + x2 (x2 + y + z )2 z + x2 + y 2 x2 + y + z Suy 1 + + ≤ x5 + y + z y + z + x2 z + x2 + y x2 + y + z Hay (1) đúng 11 (12) BẤT ĐẲNG THỨC AM - GM Ví dụ 1.17 (IMO Shortlist 2009) Cho các số thực dương a,b,c thỏa ab+bc+ca ≤ 3abc Chứng minh s s s  2 2 √ √ √ √ a +b b +c c2 + a2 + + +3≤ a+b+ b+c+ c+a a+b b+c c+a Ta có s p 2(a + b) = (a + b)2 = a+b s  r  s 2ab a2 + b2 a + b2 2ab + ≥ + a+b a+b a+b a+b Suy r VP ≥ 2ab + a+b r 2bc + b+c r 2ca + c+a s 2 a +b + a+b s 2 b +c + b+c s c2 + a2 c+a Mặt khác áp dụng bất đẳng thức   1 (x + y + z)2 ≥ 27 + + x2 y z ta suy v √ u x + y + z ≥ 3u t 1 1 + + x2 y z Do đó r 2ab + a+b r 2bc + b+c r v √ u 2ca ≥ 3u !2 u r c+a u a + b t + 2ab r 3abc =3 = ab + bc + ca r b+c 2bc !2 Từ đó, ta có đpcm Ví dụ 1.18 Cho các số thực dương a,b,c Chứng minh a3 b3 c3 a+b+c + + ≥ a2 + b2 b2 + c2 c2 + a2 Ta có a3 a (a2 + b2 ) − ab2 ab2 b = =a− ≥a− 2 2 a +b a +b a +b Tương tự b3 c c3 a ≥ b − và ≥ c − b2 + c2 c2 + a2 Cộng các bất đẳng thức trên theo vế ta có đpcm 12 r + c+a 2ca 2 (13) BẤT ĐẲNG THỨC AM - GM Ví dụ 1.19 Cho các số thực a,b,c thỏa abc < và a + b + c = Chứng minh rằng:   1 12abc − + + (1 − ab − bc − ca) + ≥ 16 a b c ab + bc + ca Gọi P là vế trái bất đẳng thức Đặt m = − (ab + bc + ca) ,n = −abc Do a + b + c = ⇒ 2(ab + bc + ca) = − (a2 + b2 + c2 ) < ⇒ m,n > Khi đó: m(1 + m) 12n + P = + n m Áp dụng bất đẳng thức Cô sita có: m3 + 8n2 + 8n ≥ 12mn và m2 + 4n2 ≥ 4mn Suy m3 + m2 + 12n2 + 8n ≥ 16mn Do đó: P = m(1 + m) 12n + + ≥ 16 n m Đẳng thức xảy và m = 2,n = 1, tức là a,b,c là ba nghiệm phương trình √ −1 ± x − 2x + = ⇔ (x − 1)(x + x − 1) = ⇔ x = 1,x = III Bài tập Bài 1.1 Cho các số thực dương a,b,c Chứng minh  √ 3 a) (1 + a) (1 + b) (1 + c) ≥ + abc b)      a b  c a+b+c 1+ 1+ ≥2 1+ √ 1+ b c a abc Bài 1.2 Cho các số thực dương a1 , a2 , · · · , an Chứng minh (1 + a1 )(1 + a2 ) · · · (1 + an ) ≥ (1 + √ n n a1 · a2 · · · an ) Bài 1.3 Cho các số thực a, b, c thỏa mãn a + b + c = Chứng minh (1 + a) (1 + b) (1 + c) ≥ 64abc Bài 1.4 Cho 2n số thực dương a1 ,a2 , ,an ,b1 ,b2 , ,bn Chứng minh p p √ n (a1 + b1 ) (a2 + b2 ) · · · (an + bn ) ≥ n a1 a2 · · · an + n b1 b2 · · · bn 13 (14) BẤT ĐẲNG THỨC AM - GM Bài 1.5 (BĐT AM-GM suy rộng) Cho ≥ (i = 1,n) và các số hữu tỉ dương αi thỏa mãn n P αi = Chứng minh rằng: i=1 n X αi ≥ aα1 · aα2 · · · aαnn i=1 Bài 1.6 Cho n số thực dương a1 , a2 , · · · , an và số nguyên dương k Chứng minh ak1 + ak2 + · · · + akn ≥ n  a1 + a2 + · · · + an n k Bài 1.7 Cho a, b, c > Chứng minh 1 1 1 + + ≥ + + a + 3b b + 3c c + 3a a + 2b + c b + 2c + a c + 2a + b Bài 1.8 Cho các số thực a,b,c > thỏa ab + bc + ca ≤ 3abc Chứng minh 1 + + ≤ √    √ √ 4 √ √ √ 4 16 4 ( c + a) 4 a+ b b+ c Bài 1.9 Cho a,b,c > Chứng minh   a b c b c a + + ≥1+ + + b + 2c c + 2a a + 2b b + 2a c + 2b a + 2c Bài 1.10 Cho x, y, z là các số thực dương thoả mãn x2 + y + z = 12.Tìm giá trị nhỏ biểu thức 1 P =√ +p +√ 1+x + z3 + y3 Bài 1.11 Cho số thực dương a,b,c thoả mãn a + b + c = Chứng minh : b c a + + ≥ 2 1+b 1+c 1+a Bài 1.12 Cho các số thực dương a,b,c thỏa a + b + c = Chứng minh rằng: a2 b2 c2 + + ≥ 2 a + 2b b + 2c c + 2a Bài 1.13 Chứng minh xy + yz + zx = thì 3x2 + 3y + z ≥ 10 Bài 1.14 Cho a,b,c > Chứng minh bất đẳng thức b3 c3 a+b+c a3 + + ≥ (a + 2b) (b + 2c) (b + 2c) (c + 2a) (c + 2a) (a + 2b) 14 (15) BẤT ĐẲNG THỨC AM - GM Bài 1.15 Cho các số thực dương a,b,c > thỏa abc = Chứng minh a4 b4 c4 + + ≥ b2 (c + 2) c2 (a + 2) a2 (b + 2) Bài 1.16 Chứng minh a, b, c > thì : ! r r r r r r a+b b+c c+a c a b + + ≥2 + + c a b a+b b+c a+c Bài 1.17 Cho các số thực a,b,cthỏa a2 + b2 + c2 = Chứng minh a4 b4 c4 + + ≥ b+2 c+2 a+2 Bài 1.18 Cho các số thực dương a,b,c thỏa a2 + b2 + c2 = Chứng minh     4 +1 +1 + ≥ (a + b + c)2 a2 + b2 b2 + c2 c2 + a2 Bài 1.19 Cho các số thực dương a,b,c thỏa: √ Chứng minh rằng: a2 + b2 + √ b2 + c2 + √ − abc c2 + a2 = √ a2 b2 c2 + + ≥ b+c c+a a+b Bài 1.20 Chứng minh a,b,c > và thỏa mãn a.b.c = thì a2 1 1 + + ≤ 2 + 2b + b + 2c + c + 2a + Bài 1.21 (Baltic Way 2014) Cho các số thực dương a,b,c thỏa 1 + + = Chứng minh a b c √ a3 +b +√ 1 +√ ≤√ +c c +a b3 Bài 1.22 (USA 2011) Với a, b, c là các số thực dương thỏa a2 + b2 + c2 + (a + b + c)2 ≤ 4, chứng minh ab + bc + ca + ≥ + + (a + b) (b + c) (c + a)2 Bài 1.23 Cho a, b, c > Chứng minh s 2 s 2 s 2 2a 2b 2c 3 + + ≥ b+c c+a a+b 15 (16) BẤT ĐẲNG THỨC AM - GM Bài 1.24 Cho các số thực dương a, b, c thỏa abc = Chứng minh r r r 3 3 3 a + b b + c c + a + + + ≤ (a + b + c) 2 Bài 1.25 Cho các số thực a,b,c thỏa a + b + c = Chứng minh 13a2 b2 c2 − 2abc − ≤ 2 (a + b + c ) Bài 1.26 Cho các số thực không âm a,b,c Chứng minh rằng: q √ (a + b + c)3 ≥ (a − b) (b − c) (c − a) Bài 1.27 Cho các số thực a,b,c phân biệt thỏa a + b + c = và ab + bc + ca > Tìm giá trị nhỏ 2 + + +√ P = |a − b| |b − c| |c − a| ab + bc + ca Bài 1.28 (JBMO 2014) Cho các số thực dương a, b, c thỏa mãn abc = Chứng minh  2  2  2 1 a+ + b+ + c+ ≥ 3(a + b + c + 1) b c a Bài 1.29 Cho các số thực dương a, b thỏa mãn ab ≥ Chứng minh    2 b + 2a + ≥ 16 a + 2b + a+1 b+1 Bài 1.30 (IMO Shortlist 2009) Cho các số thực dương a,b,c thỏa a + b + c = 1 + + a b c Chứng minh 1 + + ≤ 16 (2a + b + c) (2b + c + a) (2c + a + b) Bài 1.31 Cho a, b, c là các số thực dương thỏa mãn: √ √ √ 3 a3 + b3 + b3 + c3 + c3 + a3 + abc = Chứng minh giá trị nhỏ biểu thức: P = √ a3 b3 c3 + + b2 + c2 c2 + a2 a2 + b2 32m, đó m là nghiệm phương trình t3 + 54t − 162 = Bài 1.32 (Đề thi chọn đội tuyển, vòng 1, Hà Tĩnh, năm học 2017-20178) Cho các số thực không âm a, b, c thoả mãn điều kiện a2 + b2 + c2 ≤ Chứng minh (a + b + c)(a + b + c − abc) ≥ 2(a2 b + b2 c + c2 a) 16 (17) BẤT ĐẲNG THỨC AM - GM Bài 1.33 (Đề thi chọn đội tuyển, vòng 2, Nam Định, năm học 2017-2018) Xét các số thực a,b,c ∈ [0; 1] Tìm giá trị lớn biểu thức P = a b c + + + (1 − a) (1 − b) (1 − c) b+c+1 c+a+1 a+b+1 Bài 1.34 (Đề thi chọn đội tuyển, vòng 2, Quảng Ngãi, năm học 2017-2018) Cho a, b, c là các số thực dương thỏa mãn 3bc + 4ac + 5ab ≤ 6abc Tìm giá trị lớn biểu thức P = 3a + 2b + c (a + b)(b + c)(c + a) Bài 1.35 (ĐỀ THI HSG TỈNH TÂY NINH,VÒNG 1, 2017-2018) Cho ba số thực dương x, y, z thỏa mãn xyz = Chứng minh rằng: √ 1 √ p p ≤ + + 4 z + 2x3 + x3 + 2y + y + 2z + Bài 1.36 (Đề thi chọn đội tuyển, Lâm Đồng, √ năm học 2017-2018) Cho x,y,z là các số thực dương thỏa mãn điều kiện x3 + y + z = + 1 1 Tìm giá trị nhỏ biểu thức P = + + x y z Bài 1.37 (Đề thi chọn đội tuyển, vòng 1, Hà Tĩnh, năm học 2016-2017) Cho các số thực a,b,c dương và thỏa a5 + b5 + c5 = Chứng minh rằng: a6 b6 + b6 c6 + c6 a6 ≤ Bài 1.38 Tìm số nguyên dương k nhỏ cho bất đẳng thức xk y k z k (x3 + y + z ) ≤ đúng với số thực dương x, y, z thỏa mãn điều kiện x + y + z = 1 Bài 1.39 (VN TST 2010) Cho các số thực dương a, b, c thỏa mãn 16 (a + b + c) ≥ + + a b c Chứng minh  1 3 +  3 +  3 ≤ p p p a + b + (a + c) b + c + (b + a) c + a + (c + b) Bài 1.40 (IMO 2001) Cho các số thực dương a, b, c Chứng minh a2 b2 c2 √ +√ +√ ≥ a2 + 8bc b2 + 8ca c2 + 8ab Bài 1.41 (Turkey TST 2017) Cho các số thực dương a, b, c thỏa mãn a + b + c = Chứng minh a3 b + b3 c + c3 a + ≥ 4(ab + bc + ca) Bài 1.42 (IMO Shortlits A5-2008) Cho các số thực dương a,b,c,d thỏa mãn abcd = và a+b+c+d≥ Chứng minh a + b + c + d ≤ a b c d + + + b c d a b c d a + + + a b c d 17 (18) BẤT ĐẲNG THỨC AM - GM Bài 1.43 Cho các số thực không âm a, b, c thỏa mãn a + b + c = Chứng minh    a3 + b3 b3 + c3 c3 + a3 ≤ Bài 1.44 (Hàn Quốc MO 2016) Cho các số thực x, y, z thỏa mãn x2 + y + z = Tìm GTLN biểu thức P = (x2 − yz)(y − zx)(z − xy) Bài 1.45 Cho các số thực dương x, y, z thỏa mãn x + y + z = Chứng minh x2 y y2z2 z x2 + + + 3xyz ≤ 1−z 1−x 1−y Bài 1.46 Cho các số thực dương a,b,c thỏa mãn:   a4 + b4 + c4 − 25 a2 + b2 + c2 + 48 = Tìm giá trị nhỏ biểu thức: a2 b2 c2 F = + + b + 2c c + 2a a + 2b Bài 1.47 (TST Quảng Nam 2014-2015) Cho các số thực dương a, b, c Chứng minh √ p √ √ √ √ √  2 2 2 5a + 4bc + 5b + 4ca + 5c + 4ab ≥ (a + b + c ) + ab + bc + ca Bài 1.48 Cho các số thực dương a, b, c thỏa mãn a + b + c = Chứng minh a2 b b2 c c2 a + + ≤ 1+a+b 1+b+c 1+c+a Bài 1.49 (P122, Tạp chí Pi, tháng 12 năm 2017) Chứng minh rằng, với số thực dương a, b, c ta luôn có bất đẳng thức: s s s a2 + bc b2 + ca c2 + ab + + ≥ a(b + c) b(c + a) c(a + b) Hỏi đẳng thức xảy nào? Bài 1.50 Cho 2018 số dương a1 ,a2 , ,a2018 thỏa: a1 + a2 + · · · + a2018 = Chứng minh rằng: a1 + a2 + · · · + a2018 ≥ 2018 18 1 + +···+ a1 a2 a2018 (19) BẤT ĐẲNG THỨC CAUCHY - SCHWARZ §2 Bất đẳng thức Cauchy - Schwarz I Bất đẳng thức Cauchy-Schwarz dạng đa thức Định lí Cho 2n số thực a1 ,a2 , · · · ,an ,b1 ,b2 , · · · ,bn Khi đó, ta có   a21 + a22 + · · · + a2n b21 + b22 + · · · + b2n ≥ (a1 b1 + a2 b2 + · · · + an bn )2 Đẳng thức xảy = kbi với i = 1,2, · · · ,n Chứng minh Nếu = ∀i = 1,n thì bất đẳng thức hiển nhiên đúng n P Nếu a2i > 0, ta xét tam thức i=1 f (x) = n X ! a2i x −2 n X i=1 Ta có f (x) = ! bi x+ i=1 n X n X b2i i=1 (ai x − bi )2 ≥ ∀x ∈ R i=1 Do đó ∆0 = n X !2 bi n X − i=1 ! a2i i=1 n X ! b2i ≤0 i=1 Hay bất đẳng thức chứng minh Đẳng thức xảy x − bi = ⇔ = k.bi II Bất đẳng thức Cauchy-Schwarz dạng phân thức Định lí Cho các n số thực a1 ,a2 , · · · , an và n số thực dương b1 ,b2 , · · · ,bn Khi đó, ta có a2n (a1 + a2 + · · · + an )2 a21 a22 + + ··· + ≥ b1 b2 bn b1 + b2 + · · · + bn Đẳng thức xảy và a1 a2 an = = ··· = b1 b2 bn Chứng minh Áp dụng bất đẳng thức Cauchy-Schwarz dạng đa thức ta có !2 ! n !2 ! n n p n X X X X a2 i = bi √ ≤ bi b bi i=1 i=1 i=1 i=1 i Hay a21 a22 a2n (a1 + a2 + · · · + an )2 + + ··· + ≥ (đpcm) b1 b2 bn b1 + b2 + · · · + bn III Các ví dụ minh họa Ví dụ 2.1 Cho a, b, c > thỏa mãn a + b + c = Chứng minh r r r √ 1 a2 + + b2 + + c2 + ≥ 82 b c a 19 (20) BẤT ĐẲNG THỨC CAUCHY - SCHWARZ Áp dụng bất đẳng thức Cauchy – Schwarz ta có     2 1 a a + +9 ≥ + b b hay r Tương tự, ta có r b2 + ≥ √ c 82 a2 + ≥√ b 82   b + c  a + b r và c2  + ≥√ a 82  c + a  Công ba bất đẳng thức theo vế ta có r r r    a + b + c 1 1 1 +3 + + a2 + + b2 + + c2 + ≥ √ b c a a b c 82 Lại có 1 + + ≥ = nên ta suy a b c a+b+c r r r   √ 1 2 a + 2+ b + 2+ c + ≥√ + 27 = 82 b c a 82 Đẳng thức xảy a = b = c = Ví dụ 2.2 Cho các số thực dương a,b,c thỏa abc = Chứng minh p    + a2 + b2 + c2 ≥ (a + b) (b + c) (c + a) Áp dụng bất đẳng thức Bunhiacopsky cho hai số (1; a) và (b; 1) ta có     + a2 + b2 = + a2 b2 + ≥ (a + b)2 Tương tự + b2     + c2 ≥ (b + c)2 , + c2 + a2 ≥ (a + c)2 Nhận các bất đẳng thức trên theo vế ta    + a2 + b2 + c2 ≥ (a + b) (b + c) (c + a) Mặt khác √ √ √ (a + b) (b + c) (c + a) ≥ ab.2 bc.2 ca = 8abc = Suy q (a + b) (b + c) (c + a) [(a + b) (b + c) (c + a)]2 √ p p ≥ (a + b) (b + c) (c + a) 82 = (a + b) (b + c) (c + a) (đpcm) (a + b) (b + c) (c + a) = p 3 20 (21) BẤT ĐẲNG THỨC CAUCHY - SCHWARZ Ví dụ 2.3 Cho a,b,c > thỏa a2 1 + + ≥ 2 + b + b + c + c + a2 + Chứng minh ab + bc + ca ≤ Áp dụng bất đẳng thức Cauchy-Schwarz cho hai số (a; b; 1) và (1; 1; c) ta có   a2 + b2 + 1 + + c2 ≥ (a + b + c)2 Suy + c2 ≤ a2 + b2 + (a + b + c)2 Tương tự: + a2 + b2 ≤ , ≤ b2 + c2 + (a + b + c)2 c2 + a2 + (a + b + c)2 Suy + a2 + b2 + c2 + + ≥ 1, (a + b + c)2 (a + b + c)2 (a + b + c)2 Do đó ta có + a2 + b2 + c2 ≥ (a + b + c)2 ⇒ ab + bc + ca ≤ (đpcm) Ví dụ 2.4 Cho các số thực dương a,b,c thỏa mãn a + b + c = 1 + + Chứng minh a b c √ √ √ √ a2 + + b2 + + c2 + ≤ (a + b + c) Áp dụng bất đẳng thức Cauchy – Schwarz ta có r r r √ √ √ √ √ √ 2 a + + b + + c + = a a + + b b + + c c + a b c s   1 ≤ (a + b + c) a + + b + + c + a b c √ = (a + b + c) Đẳng thức xảy a = b = c = Ví dụ 2.5 Cho các số thực a, b, c, x, y, z Chứng minh ax + by + cz + p (a2 + b2 + c2 )(x2 + y + z ) ≥ (a + b + c)(x + y + z) Ta có (a + b + c)(x + y + z) − (ax + by + cz) 2y + 2z − x 2z + 2x − y 2x + 2y − z =a· +b· +c· 3 v u    2  2 ! u 2y + 2z − x 2z + 2x − y 2x + 2y − z ≤ t(a2 + b2 + c2 ) + + 3 21 (22) BẤT ĐẲNG THỨC CAUCHY - SCHWARZ Hơn  2y + 2z − x 2  + 2z + 2x − y 2  + 2x + 2y − z 2 = x2 + y + z Nên ta có đpcm Ví dụ 2.6 Cho các số thực a,b,c thỏa a2 + b2 + c2 = Chứng minh (a + b + c) − abc ≤ 10 Không tính tổng quát, ta giả sử |a| ≤ |b| ≤ |c|   a2 + b2 ≤ a2 + b2 + c2 = 18 ⇒ a2 + b2 ≤ Áp dụng bất đẳng thức Cauchy-Schwarz cho hai số q   (a + b + c) − abc = (a + b) + c (2 − ab) ≤ + (2 − ab)2 (a + b)2 + c2 p = (8 − 4ab + a2 b2 ) (a2 + b2 + c2 + 2ab) p = (8 − 4ab + a2 b2 ) (9 + 2ab) Do đó, ta cần chứng minh p (8 − 4ab + a2 b2 ) (9 + 2ab) ≤ 10 ⇔ 2a3 b3 + a2 b2 − 20ab − 28 ≤ ⇔ (ab + 2)2 (2ab − 7) ≤ (*) Vì 2ab ≤ a2 + b2 ≤ ⇒ 2ab − < 0, đó (*) đúng Ví dụ 2.7 (VQB Cẩn) Cho các số thực dương a,b,c thỏa mãn a + b + c = và a2 + b2 + c2 = 14 Chứng minh 4a + b 31 2≤ ≤ c Ta có 4a + b ≥ ⇔ −4a − b + 2c ≤ ⇔ 3a + 6b + 9c ≤ (a + b + c) = 42 c (1) Áp dụng bất đẳng thức Cauchy – Schwarz ta có p 3a + 6b + 9c ≤ (32 + 62 + 92 ) (a2 + b2 + c2 ) = 42 Suy (1) đúng Đẳng thức xảy a = 1,b = 2,c = Tương tự 4a + b 31 ≤ ⇔ 8a + 2b − 31c ≤ ⇔ 57a + 51b + 18c ≤ 49 (a + b + c) = 294 c Áp dụng bất đẳng thức Cauchy – Schwarz ta có p 57a + 51b + 18c ≤ (572 + 512 + 182 ) (a2 + b2 + c2 ) = 294 Hay (2) chứng minh Đẳng thức xảy a = 22 19 17 ,b = ,c = 7 (2) (23) BẤT ĐẲNG THỨC CAUCHY - SCHWARZ Ví dụ 2.8 Cho các số thực dương a, b, c Chứng minh r r r 2a 2b 2c + + ≤ a+b b+c c+a Áp dụng bất đẳng thức Cauchy – Schwarz ta có s !2 r r √ √ √ a b c V T2 = a+c + b+a + c+b (a + b)(a + c) (b + c)(b + a) (c + a)(c + b)   a b c ≤ (a + b + c) + + (a + b) (a + c) (b + a) (b + c) (c + a) (c + b) (a + b + c) [ab + bc + ca] = (a + b) (b + c) (c + a) Do đó, ta cần chứng minh (a + b + c) (ab + bc + ca) (a + b + c) (ab + bc + ca) ≤ ⇔ ≤ (a + b) (b + c) (c + a) (a + b) (b + c) (c + a) Đây là kết quen thuộc Ví dụ 2.9 Cho các số thực dương a,b,c thỏa a2 1 + + = +2 b +2 c +2 Chứng minh rằng: ab + bc + ca ≤ Từ giả thiết suy ra: 1= a2 b2 c2 (a + b + c)2 + + ≥ a2 + b + c2 + a2 + b + c2 + Do đó: a2 + b2 + c2 + ≥ (a + b + c)2 ⇔ ab + bc + ca ≤ (đpcm) Ví dụ 2.10 Cho các số thực a,b,c > thỏa mãn a + b + c = Chứng minh b2 c2 a2 + + ≥ a + 2b2 b + 2c2 c + 2a2 Gọi P là vế trái bất đẳng thức cần chứng minh Áp dụng bất đẳng thức Cauchy – Schwarz ta có a4 b4 c4 P = + + a + 2a2 b2 b3 + 2b2 c2 c3 + 2c2 a2 (a2 + b2 + c2 ) ≥ a + b3 + c3 + (a2 b2 + b2 c2 + c2 a2 ) Với a + b + c = ta có  2 a2 + b2 + c2 ≥ a3 + b3 + c3  2 a3 + b3 + c3 (a + b + c) ≥ a2 + b2 + c2  a2 + b2 + c2 ≥ (a + b + c)2 a4 + b4 + c4  23 (24) BẤT ĐẲNG THỨC CAUCHY - SCHWARZ Nhân ba bất đẳng thức trên theo vế ta   a4 + b4 + c3 ≥ (a + b + c) a3 + b3 + c3 Hay a4 + b4 + c4 ≥ a3 + b3 + c3 Do đó  2 a2 + b2 + c2 = a4 + b4 + c4 + +2 a2 b2 + b2 c2 + c2 a2  ≥ a3 + b3 + c3 + a2 b2 + b2 c2 + c2 a2 Vậy P ≥ Đẳng thức xảy a = b = c = Ví dụ 2.11 Cho các số thực dương a,b,c Chứng minh rằng:  2  2  2 a b c + + ≥ a+b b+c c+a Vì b c a = nên tồn các số thực dương x,y,z cho a b c yz c zx a xy b = 2, = 2, = a x b y c z Bất đẳng thức cần chứng minh trở thành y4 z4 x4 + + ≥ 2 (x2 + yz) (y + zx) (z + xy) Áp dụng bất đẳng thức Cauchy – Schwarz ta có x4 y4 z4 (x2 + y + z ) + + ≥ (x2 + yz)2 (y + zx)2 (z + xy)2 (x2 + yz)2 + (y + zx)2 + (z + xy)2 Ta chứng minh (x2 + y + z ) ≥ 2 (x2 + yz) + (y + zx) + (z + xy) Biến đổi và rút gọn ta thu bất đẳng thức  x4 + y + z + x2 y + y z + z x2 ≥ 6xyz (x + y + z) (∗) Ta có x4 + y + z ≥ x2 y + y z + z x2 ≥ xyz (x + y + z) Nên suy (∗) đúng Vậy bài toán chứng minh Ví dụ 2.12 (P61, Tạp chí Pi, tháng năm 2017) Cho a, b, c là độ dài ba cạnh tam giác Chứng minh a b c 2(ab + bc + ca) + + + ≤ 2 b+c c+a a+b a +b +c Hỏi dấu xảy và nào? Ta biết với a, b, c là ba số thực tùy ý, luôn có ab + bc + ca ≤ a2 + b2 + c2 24 (25) BẤT ĐẲNG THỨC CAUCHY - SCHWARZ Do đó 2(ab + bc + ca) ab + bc + ca ≤ + 2 a +b +c a + b2 + c2 (1) b c ab + bc + ca a + + + ≤ 2 b+c c+a a+b a +b +c (2) Tiếp theo ta chứng minh Thật vậy, ta có  (2) ⇔ −      b c ab + bc + ca a + 1− + 1− ≥ + b+c c+a a+b a + b2 + c2 (a + b + c)2 b+c−a c+a−b a+b−c + + ≥ ⇔ b+c c+a a+b 2(a2 + b2 + c2 ) (b + c − a)2 (c + a − b)2 (a + b − c)2 (a + b + c)2 ⇔ + + ≥ (b + c − a)(b + c) (c + a − b)(c + a) (a + b − c)(a + b) 2(a2 + b2 + c2 ) (3) Do đó a, b, c là độ dài cạnh tam giác nên b + c > a, a + c > b và a + b > c Do đó, tất các phân thức nằm vế trái (3) có mẫu thức dương Vì thế, ký hiệu VT là biểu thức nằm vế trái (3), theo hệ bất đẳng thức Cauchy - Schwarz, ta có VT ≥ (a + b + c)2 2(a2 + b2 + c2 ) Vì (b + c − a) + (c + a − b) + (a + b − c) = a + b + c (b + c − a)(b + c) + (c + a − b)(c + a) + (a + b − c)(a + b) = 2(a2 + b2 + c2 ) nên (3) chứng minh và vì (2) chứng minh Từ (1) và (2), hiển nhiên, suy bất đẳng thức cần chứng minh theo yêu cầu bài toán Từ điều kiện cần và đủ để dấu xảy các bất đẳng thức trên, dễ dàng suy dấu bất đẳng thức đề bài xảy và a, b, c là độ dài ba cạnh tam giác Ví dụ 2.13 Cho a,b,c > thỏa a + b + c = Chứng minh rằng: b c a √ +√ +√ ≤ 4a + 3bc 4b + 3ca 4c + 3ab Áp dụng bất đẳng thức Cauchy – Schwarz ta có:  2   a b c a b c √ +√ +√ ≤ (a + b + c) + + 4a + 3bc 4b + 3ca 4c + 3ab 4a + 3bc 4b + 3ca 4c + 3ab   a b c =2 + + 4a + 3bc 4b + 3ca 4c + 3ab Ta chứng minh: a b c + + ≤ 4a + 3bc 4b + 3ca 4c + 3ab bc ca ab ⇔ + + ≥ 4a + 3bc 4b + 3ca 4c + 3ab Ta có V T (1) ≥ (ab + bc + ca)2 bc(4a + bc) + ca(4b + ca) + ab(4c + ab) 25 (1) (26) BẤT ĐẲNG THỨC CAUCHY - SCHWARZ Do bc(4a + bc) + ca(4b + ca) + ab(4c + ab) = 3(ab + bc + ca)2 Nên ta có: V T (1) ≥ (đpcm) Đẳng thức xảy và a = b = c = 3 Ví dụ 2.14 Cho các số thực x,y,z > Chứng minh x+y p x2 + y + zx + zy +p y+z y + z + xy + xz z+x +p z + x2 + yz + xy ≤ Áp dụng bất đẳng thức Cauchy – Schwarz ta có # " 2 (y + z) (z + x) (x + y) + + V T2 ≤ x + y + zx + yz y + z + xy + xz z + x2 + zy + yx Mặt khác (x + y)2 (x + y)2 x2 y2 x y = ≤ + = + 2 x + y + zx + yz x(x + z) + y(y + z) x (x + z) y (y + z) x+z y+z Tương tự (y + z)2 y z (z + x)2 z x ≤ + và ≤ + 2 y + z + xy + xz y+x z+x z + x + zy + yx z+y x+y Suy V T ≤ ⇔ V T ≤ 3, từ đây ta có đpcm Ví dụ 2.15 Cho a, b, c là các số thực không âm và không có hai số nào đồng thời Chứng minh b2 c2 a2 + + ≤ (2a + b)(2a + c) (2b + c)(2b + a) (2c + a)(2c + b) Áp dụng bất đẳng thức Cauchy-Schwarz, ta có 9a2 (2a + a)2 = (2a + b)(2a + c) 2a(a + b + c) + (2a2 + bc) 4a2 a2 ≤ + 2a(a + b + c) 2a + bc 2a a2 = + a + b + c 2a + bc Do đó 9V T ≤ + a2 b2 c2 + + 2a2 + bc 2b2 + ca 2c2 + ab Nên ta chứng minh a2 b2 c2 + + ≤1 2a2 + bc 2b2 + ca 2c2 + ab bc ca ab + + ≥ (1) ⇔ 2a + bc 2b + ca 2c + ab Áp dụng bất đẳng thức Cauchy-Schwarz ta có V T (1) ≥ (ab + bc + ca)2 (ab + bc + ca)2 = = 2abc(a + b + c) + a2 b2 + b2 c2 + c2 a2 (ab + bc + ca)2 26 (27) BẤT ĐẲNG THỨC CAUCHY - SCHWARZ IV Bài tập Bài 2.1 (Bất đẳng thức Mincopski) Cho các 2n số thực a1 ,a2 , · · · ,an ,b1 ,b2 , · · · ,bn Chứng minh q q q 2 2 2 a1 + a2 + · · · + an + b1 + b2 + · · · + bn ≥ (a1 + b1 )2 + (a2 + b2 )2 + · · · + (an + bn )2 Bài 2.2 Cho các số thực dương a,b,c Chứng minh    a2 + b2 + c2 + ≥ (a + b) (b + c) (c + a) Bài 2.3 Cho các số thực dương a,b,c thỏa mãn a + b + c = Chứng minh    1 2 + + a +b +c +3≥9 a2 + b2 + c2 + Bài 2.4 Cho a,b,c > và a + b + c = Chứng minh √ √ √ a a2 + 8bc + b b2 + 8ca + c c2 + 8ab ≤ Bài 2.5 Cho các số thực dương a, b, c thỏa a2 + b2 + c2 = Chứng minh rằng: a3 b3 c3 + + ≥ b + 2c c + 2a a + 2b Bài 2.6 Cho a, b, c là các số thực dương thỏa: a2 + b2 + c2 ≥ Chứng minh rằng: √ a3 b3 c3 +√ +√ ≥ b2 + c2 + c2 + a2 + a2 + b2 + Bài 2.7 Cho các số thực dương a,b,c có tổng Chứng minh rằng: 4a2 1 1 + + ≤ 2 2 2 +b +c a + 4b + c a + b + 4c Bài 2.8 Cho các số thực không âm a, b, c thỏa mãn a + b + c = Chứng minh 1 + + ≥ + 2b + 2c + 2a2 Bài 2.9 Cho các số thực không âm a, b, c thỏa mãn ab + bc + ca = Chứng minh a2 1 + + ≥ +1 b +1 c +1 Bài 2.10 Cho các số thực dương a, b, c thỏa mãn a2 + b2 + c2 = Chứng minh 1 + + ≤ − ab − bc − ca 27 (28) BẤT ĐẲNG THỨC CAUCHY - SCHWARZ Bài 2.11 Cho ba số thực dương x, y, z thỏa mãn x + y + z = Chứng minh rằng: x3 4y + 4z + 162 4x + + + ≥ 2 2 + xy + 3xyz y + yz + 3xyz z + zx + 3xyz x + y + z + 27 Bài 2.12 Cho a, b, c > thỏa mãn a + b + c = Chứng minh a2 b2 c2 + + ≥ 2 2 (b + 2c) (a + b) (c + 2a) (b + c) (a + 2b) (c + a) Bài 2.13 Cho a,b,c ∈ (1; 2) Chứng minh √ √ √ b a c b a c √ √ + √ √ + √ √ ≥ 4b c − c a 4c a − a b 4a b − b c Bài 2.14 Cho a,b,c là các số thực dương thỏa mãn a2 b2 + b2 c2 + c2 a2 ≥ a2 b2 c2 Chứng minh √ b2 c2 c2 a2 a2 b + + ≥ 2 2 c (a + b ) a (b + c ) b (c + a ) Bài 2.15 (IMO Shortlist-2007) Cho các số thực không âm a1 , a2 , , a100 thỏa mãn điều kiện a21 + a22 + · · · + a2100 = Chứng minh √ 2 2 S = a1 a2 + a2 a3 + · · · + a100 a1 ≤ Bài 2.16 (China TST 2005) Cho các số thực không âm a,b,c thỏa ab + bc + ca = Chứng minh 1 + + ≤ a − bc + b − ca + c − ab + Bài 2.17 Cho các số thực dương a, b, c Chứng minh r r r   ab bc ca b2 c2 a2 + + ≥ a + + b + + c + ≥ 2(a + b + c) c a b c a b Bài 2.18 Cho x,y,z > −1 Chứng minh + x2 + y2 + z2 + + ≥ + y + z + z + x2 + x + y Bài 2.19 (P77, Tạp chí Pi, tháng năm 2017) Cho a, b, c là các số thực dương Chứng minh a b c p + + ≥ 4(b3 + c3 ) c + a a + b Hỏi đẳng thức xảy và nào? Bài 2.20 Cho ba số thực không âm a, b, c thỏa mãn điều kiện (b + c)(c + 2a)(c + 4a) > Chứng minh a b 2c + + ≥ b + c c + 4a c + 2a Hỏi đẳng thức xảy và nào? 28 (29) BẤT ĐẲNG THỨC CAUCHY - SCHWARZ Bài 2.21 Cho a,b,c > thỏa mãn a2 + b2 + c2 = Chứng minh 1 + + ≥ 2−a 2−b 2−c Bài 2.22 Cho bốn số thực a,b,c,d thỏa mãn a2 + b2 + c2 + d2 = Chứng minh 1 1 16 + + + ≤ − ab − bc − cd − da Bài 2.23 Cho x,y,z > thỏa mãn xyz = Chứng minh 1 + + ≥ 2 1+x+x 1+y+y + z + z2 Bài 2.24 Cho x,y,z > thỏa mãn xyz = Chứng minh y2 z2 x2 + + ≥ x2 + 2x + y + 2y + z + 2z + Bài 2.25 (IMO 2008) Cho các số thực x,y,z 6= và xyz = Chứng minh  2  2  2 x y z + + ≥ x−1 y−1 z−1 Bài 2.26 Cho a, b, clà các số thực dương Chứng minh s s r bc ac ab + + ≥ a(3b + a) b(3c + b) c(3a + c) Bài 2.27 Cho các số thực a, b, c tất không cùng dấu Chứng minh (a2 + ab + b2 )(b2 + bc + c2 )(c2 + ca + a2 ) ≥ 3(ab + bc + ca)3 Bài 2.28 Cho a, b, c ≥ và không có hai số nào đồng thời Chứng minh a2 b2 c2 + + ≥ 2b2 − bc + 2c2 2c2 − ca + 2a2 2a2 − ab + 2b2 Bài 2.29 Cho a, b, c là các số thực thỏa mãn điều kiện 3a2 + 2b2 + c2 = Tìm giá trị lớn và giá trị nhỏ biểu thức P = 2(a + b + c) − abc Bài 2.30 (Iran MO 2016 day 3) Cho các số thực dương a, b, c thỏa mãn abc = Chứng minh a+b b+c c+a + + ≥ 2 (a + b + 1) (b + c + 1) (c + a + 1) a+b+c 29 (30) BẤT ĐẲNG THỨC CAUCHY - SCHWARZ Bài 2.31 Cho các số thực dương x, y, z thỏa mãn xyz ≥ Chứng minh (x4 + y)(y + z)(z + x) ≥ (x + y )(y + z )(z + x2 ) Bài 2.32 (Serbia TST 2016) Cho các số thực dương a, b, c minh a b c √ +√ +√ ≤ 3c + a 3a + b 3b + c thỏa mãn a + b + c = Chứng Bài 2.33 (Hải Dương TST 2016) Cho a, b, c ∈ [−1,1] thỏa mãn: + 2abc ≥ a2 + b2 + c2 Chứng minh : + 2a3 b3 c3 ≥ a6 + b6 + c6 Bài 2.34 Cho các số thực dương a, b, c thỏa mãn a2 + b2 + c2 = Chứng minh   b+c c+a a+b 1 + + + 2+ ≥ a b c a b c Bài 2.35 Cho n (n ≥ 1) x1 , x2 , , xn thỏa mãn x1 + x2 + · · · + xn = Chứng minh (n − 2)x21 + 2x1 (n − 2)x22 + 2x2 (n − 2)x2n + 2xn + + · · · + ≥ (n − 1)x21 + (n − 1)x22 + (n − 1)x2n + Bài 2.36 Cho các số thực dương a,b,c thỏa mãn a2 + b2 + c2 = Chứng minh 1 (a + b + c)2 + + ≥ 2a2 + bc 2b2 + ac 2c2 + ab ab + bc + ac Bài 2.37 Cho a, b, c là các số thực dương và n ∈ N, n ≥ Tìm giá trị nhỏ biểu thức s s s 2 a + bc b + ac c2 + ab P = n + n + n a(b + c) b(a + c) c(a + b) Bài 2.38 Cho các số dương a, b, c thỏa mãn abc = Chứng minh rằng: b3 + c3 + a3 + + + ≥ a3 (b + c) b3 (c + a) c3 (a + b) Bài 2.39 Cho số nguyên dương n ≥ và 2n số thực dương a1 ; a2 ; ; an ; b1 ; b2 ; ; bn thỏa mãn: n n X X ak = 1; b2k = k=1 Chứng minh rằng: n X k=1 ak (bk + ak+1 ) < (với an+1 = a1 ) k=1 30 (31) MỘT SỐ BẤT ĐẲNG THỨC KHÁC §3 Một số bất đẳng thức khác I Bất đẳng thức Schur Bất đẳng thức Schur Định lí Cho các số thực không âm x,y,z và số thực dương r Khi đó, ta có bất đẳng thức sau xr (x − y)(x − z) + y r (y − x)(y − z) + z r (z − x)(z − y) ≥ Đẳng thức xảy a = b = c c = 0,a = b và các hoán vị Chứng minh Vì bất đẳng thức cần chứng minh là đối xứng ba biến nên ta giả sử x ≥ y ≥ z, đó z r (z − x)(z − y) ≥ và xr (x − y)(x − z) + y r (y − x)(y − z) ≥ (x − y) (xr (y − z) − y r (y − z)) = (x − y)(y − z)(xr − y r ) ≥ Từ hai bất đẳng thức trên ta suy đpcm Các trường hợp đặc biệt • Xét r = ta có các dạng sau a) x3 + y + z + 3xyz ≥ xy(x + y) + yz(y + z) + zx(z + x) b) 4(a3 + b3 + c3 ) + 15abc ≥ (a + b + c)3 c) xyz ≥ (x + y − z)(y + z − x)(z + x − y) 9xyz d) x2 + y + z + ≥ 2(xy + yz + zx) x+y+z e) (x + y + z)3 + 9xyz ≥ 4(x + y + z)(xy + yz + zx) • r = ta có các dạng sau a) x4 + y + z + xyz(x + y + z) ≥ xy(x2 + y ) + yz(y + z ) + zx(z + x2 ) b) 6xyz(x + y + z) ≥ [2(xy + yz + zx) − (x2 + y + z )] (x2 + y + z + xy + yz + zx) Bất đẳng thức Schur mở rộng Định lí Cho các số thực dương a, b, c, x, y, z cho các (a, b, c) và (x, y, z) là các đơn điệu Khi đó, ta có bất đẳng thức a(x − y)(x − z) + b(y − z)(y − x) + c(z − x)(z − y) ≥ Chứng minh Việc chứng minh bất đẳng thức này tương tự chứng minh bất đẳng thức Schur trên Các ví dụ Ví dụ 3.1 (Đồng Nai TST 2016) Cho các số thực dương a, b, c thỏa mãn abc = Chứng minh rằng: r √ r r a b c 3 + + ≥√ b+c c+a a+b a3 + b3 + c3 + 31 (32) MỘT SỐ BẤT ĐẲNG THỨC KHÁC Với x, y, z > ta có: (x + y + z)  1 + 2+ 2 x y z  ≥9 Suy : p r x2 y z · 3 √ 3 x+y+z ≥ s 2 x2 y z = 27 (∗) 1 + + x y z r r r a b c Áp dụng (*) với x = ,y= ,z= ta có b+c c+a a+b r √ r r 3 a b c + + ≥r b+c c+a a+b b+c c+a a+b + + a b c √ 3 =p ab (a + b) + bc (b + c) + ca (c + a) Mặt khác, theo bất đẳng thức Schur ta có a3 + b3 + c3 + 3abc ≥ ab (a + b) + bc (b + c) + ca (c + a) r √ r r 3 a b c Nên ta có : + + ≥√ b+c c+a a+b a3 + b3 + c3 + Đẳng thức có a = b = c = Ví dụ 3.2 (Nghệ an TST 2014, ngày 2) Cho các số thực x, y, z > thỏa xyz = Chứng minh r r r 5(x + y + z) + z+x y + z x + y + + ≤ 2z 2x 2y Đặt x = a3 , y = b3 , z = c3 bất đẳng thức cần chứng minh trở thành r r r 3 3 3 5(a3 + b3 + c3 ) + a + b b + c c + a + + ≤ 2c3 2a3 2b3 Theo bất đẳng thức Schur ta có a3 + b3 + c3 + = a3 + b3 + c3 + 3abc ≥ ab(a + b) + bc(b + c) + ca(c + a) Do đó 5(a3 + b3 + c3 ) + = 2(a3 + b3 + c3 ) + 3(a3 + b3 + c3 + 3) ≥ 2(a3 + bb + c3 ) + 3ab(a + b) + 3bc(b + c) + 3ca(c + a) = (a + b)3 + (b + c)3 + (c + a)3 (1) Ta chứng minh (a + b)3 ≥ r 32 a3 + b3 2c3 (2) (33) MỘT SỐ BẤT ĐẲNG THỨC KHÁC Thật (1) tương đương với (a + b)3 ≥ 8abc c r a3 + b3 a3 + b3 ⇔ (a + b)9 ≥ 83 a3 b3 2 (3) Ta có · 82 · a3 b3 (a3 + b3 ) = ab · ab · ab(a2 − ab + b2 )(a + b)  4 ab + ab + ab + a2 − ab + b2 ≤4·8 (a + b) = (a + b)9 Do đó (2) đúng Tương tự ta có (b + c)3 ≥ r b3 + c3 (c + a)3 ≥ , 2a3 r c3 + a3 b3 (4) Công các bất đẳng thức (2), (4) và từ (1) ta có đpcm Ví dụ 3.3 (VMO 2014) Cho a, b, c ≥ Chứng minh 3(a2 + b2 + c2 ) ≥ P ≥ (a + b + c)2 , √ √ √  với P = (a + b + c) ab + bc + ca + (a − b)2 + (b − c)2 + (c − a)2 Ta có 3(a2 + b2 + c2 ) ≥ P ⇔ a + b + c ≥ √ √ ab + bc + √ ca Bất đẳng√thức này√là kết √ quen thuộc Đặt x = a, y = b, z = c Khi đó, bất đẳng thức X X X X P ≥ (a + b + c)2 ⇔ x4 + xyz x+ xy(x2 + y ) ≥ x2 y (1) Sử dụng bất đẳng thức Schur (với trường hợp r = 2) ta có X X X x4 + xyz x≥ xy(x2 + y ) đó V T (1) ≥ X xy(x2 + y ) ≥ X xy.2xy = X x2 y Hay (1) chứng minh Ví dụ 3.4 Cho a,b,c > Chứng minh a2 + bc b2 + ca c2 + ab 1 + + ≥ + + 2 a (b + c) b (c + a) c (a + b) a b c Ta có a2 + bc a2 + bc − a(b + c) (a − b)(a − c) − = = 2 a (b + c) a a (b + c) a2 (b + c) Do đó, bất đẳng thức cần chứng minh tương đương với x(a − b)(a − c) + y(b − c)(b − a) + z(c − a)(c − b) ≥ (1) 1 ,y= ,z= + c) b (c + a) c (a + b) 1 ab(b − a) + c(b2 − a2 ) Giả sử a > b > c, ta có − = > hay x < y a (b + c) b (c + a) a2 b2 (b + c)(c + a) Do đó, (x,y,z) là đơn điệu giảm Do đó, theo bất đẳng thức Schur suy rộng, ta có (1) đúng Với x = a2 (b 33 (34) MỘT SỐ BẤT ĐẲNG THỨC KHÁC II Bất đẳng thức Holder Bất đẳng thức Holder Định lí Cho mn số thực dương aij với i = 1,m và j = 1,n Khi đó ta có bất đẳng thức sau ! !m m n n Y m Y X X am ≥ j ij i=1 j=1 j=1 i=1 Trường hợp đặc biệt • m = ta có bất đẳng thức Cauchy-Schwarz • m = ta có (a31 + a32 + · · · + a3n )(b31 + b32 + · · · + b3n )(c31 + c32 + · · · + c3n ) ≥ (a1 b1 c1 + · · · + an bn cn )3 Ví dụ minh họa Ví dụ 3.5 Cho các số thực dương a, b, c Chứng minh (a3 + 2)(b3 + 2)(c3 + 2) ≥ (a + b + c)3 Áp dụng bất đẳng thức Holder ta có (a3 + 2)(b3 + 2)(c3 + 2) = (a3 + + 1)(1 + b3 + 1)(1 + + c3 ) ≥ (a + b + c)3 Ví dụ 3.6 Cho các số thực dương a, b, c Chứng minh √ b+c c+a a+b √ +√ +√ ≥ a + b + c a + 2c b + 2a c + 2b Áp dụng bất đẳng thức Holder ta có 2 X   b+c c+a a+b √ +√ (a + b)(a + 2c) ≥ 8(a + b + c)3 +√ a + 2c b + 2a c + 2b Mặt khác X Do đó (a + b)(a + 2c) = (a + b + c)2 + 3(ab + bc + ca) ≤ 2(a + b + c)2 √ a+b b+c c+a √ +√ +√ ≥ a + b + c a + 2c b + 2a c + 2b Ví dụ 3.7 Cho các số thực dương a, b, c Chứng minh a b c √ +√ +√ ≥ a2 + 8bc b2 + 8ca c2 + 8ab 34 (35) MỘT SỐ BẤT ĐẲNG THỨC KHÁC Gọi P là vế trái bất đẳng thức áp dụng bất đẳng thức Holder ta có X  2 P a(a + 8bc) ≥ (a + b + c)3 Mà X a(a2 + 8bc) = a3 + b3 + c3 + 24abc ≤ (a + b + c)3 , nên ta có P ≥ III Bất đẳng thức Chebyshev Bất đẳng thức Chebyshev Định lí a) Với hai dãy n số thực a1 ≥ a2 ≥ · · · ≥ an và b1 ≥ b2 ≥ · · · ≥ bn cùng tăng cùng giảm, tức là ( ( a1 ≥ a2 ≥ · · · ≥ an a1 ≤ a2 ≤ · · · ≤ an b1 ≥ b2 ≥ · · · ≥ bn b1 ≤ b2 ≤ · · · ≤ b n ta luôn có a1 + a2 + · · · + an b1 + b2 + · · · + bn a1 b1 + a2 b2 + · · · + an bn ≥ ··· n n n b) Với hai dãy n số thực a1 ≥ a2 ≥ · · · ≥ an và b1 ≥ b2 ≥ · · · ≥ bn có dãy tăng và dãy giảm, tức là ( ( a1 ≥ a2 ≥ · · · ≥ an a1 ≤ a2 ≤ · · · ≤ an b1 ≤ b2 ≤ · · · ≤ bn b ≥ b2 ≥ · · · ≥ bn ta luôn có a1 + a2 + · · · + an b1 + b2 + · · · + bn a1 b1 + a2 b2 + · · · + an bn ≤ ··· n n n Chứng minh Ta chứng minh cho trường hợp ( a1 ≥ a2 ≥ · · · ≥ an b1 ≥ b2 ≥ · · · ≥ b n a1 + a2 + · · · + an , đó tồn số k cho ak ≤ a ≤ ak+1 , với k đó ta chọn số b n cho bk ≤ b ≤ bk+1 Khi đó ta có Đặt a = (a − )(b − bi ) ≥ ⇔ ab − abi − + bi Cho i chạy từ đến n và cộng n bất đẳng thức đó ta nab − a n X bi − b i=1 Mà b n P n X + i=1 n X bi ≥ i=1 = nab nên ta có i=1 n X i=1 bi ≥ a n X i=1 bi = n Ta có đpcm 35 n X i=1 ! n X i=1 ! bi (36) MỘT SỐ BẤT ĐẲNG THỨC KHÁC Ví dụ minh họa Ví dụ 3.8 Cho các số thực dương a, b, c thỏa a2 + b2 + c2 ≥ Chứng minh a3 b3 c3 + + ≥ b+c c+a a+b Gải sử a ≥ b ≥ c, đó a2 ≥ b2 ≥ c2 và a b c ≥ ≥ , nên áp dụng bất đẳng thức b+c c+a a+b Trebyshev ta có b c a + b2 + c2 b+c c +a a+b  a 1 b c 2 ≥ (a + b + c ) + + ≥ b+c c+a a+b V T = a2 Ví dụ 3.9 Cho các số thực không âm x, y, z thỏa mãn x + y + z = Chứng minh 27 1 ≤ + + 2 1+x 1+y z +1 10 Bất đẳng thức cần chứng minh tương đương với       1 9 − − − + + ≥0 10 + x2 10 + y 10 + z 3y + 3z + 3x + + (3y − 1) + (3z − 1) ≥ ⇔ (3x − 1) + x2 + y2 + z2 (1) Với ≥ a ≥ b ≥ ta có (3a + 1)(b2 + 1) − (3b + 1)(a2 + 1) = (a − b)(3ab − a − b + 3) ≥ ⇒ Giả sử x ≥ y ≥ z, ta có 3x − ≥ 3y − ≥ 3z − và 3a + 3b + ≥ 1+a + b2 3y + 3z + 3x + ≥ ≥ Do đó áp dụng 2 1+x 1+z + z2 bất đẳng thức Chebyshev ta có  V T (1) ≥ (3x − + 3y − + 3z − 1) 3x + 3y + 3z + + + + x2 + z2 + z2  = Vậy bài toán chứng minh IV Bài tập Bài 3.1 Cho các số thực dương a, b, c Chứng minh r 2b 2c 3abc 2a + + + ≥ b+c c+a a+b a + b + c3 Bài 3.2 Cho các số thực dương a, b, c thỏa mãn a + b + c = Chứng minh   ab bc ca + + + ≥ 6(ab + bc + ca) a+b b+c c+a 36 (37) MỘT SỐ BẤT ĐẲNG THỨC KHÁC Bài 3.3 Cho các số thực dương a, b, c Chứng minh s √ X a2 + bc abc ≥ b2 + c2 a+b+c cyc Bài 3.4 (APMO 2004) Cho các số thực dương a,b,c Chứng minh (a2 + 2)(b2 + 2)(c2 + 2) ≥ 9(ab + bc + ca) Bài 3.5 Cho các số thực không âm a, b, c Chứng minh s s s √ (a + b)3 (b + c)3 (c + a)3 + + ≥ 2 ab(4a + 4b + c) bc(4b + 4c + a) ca(4c + 4a + b) Bài 3.6 (Hello IMO 2007- Trần Nam Dũng) Chứng minh với a, b, c ≥ 0,ta có: 2(a2 + b2 + c2 ) + abc + ≥ 5(a + b + c) Bài 3.7 Cho các số thực dương a,b,c thỏa mãn abc = Chứng minh 1 + + + ≥ 2(a + b + c) a2 b2 c2 Bài 3.8 Cho các số thực dương a, b, c thỏa mãn abc = Chứng minh a2 + b2 + c2 + ≥ 2(ab + bc + ca) Bài 3.9 Cho các số thực dương a, b, c thỏa a + b + c = Chứng minh 4a3 + 9b3 + 36c3 ≥ Bài 3.10 Cho các số thực dương a, b, c có a + b + c = Chứng minh √ a b c +√ +√ ≥ 3 c + 2a a + 2b b + 2c a b c + + = Chứng minh x y z √ 3 √ √ 3 x2 + y + z ≥ a2 + b2 + c2 Bài 3.11 Cho các số thực dương x, y, z và a, b, c thỏa mãn Bài 3.12 Cho a, b, c, x, y, z là các số thực dương thỏa ax + by + cz = Chứng minh  √ √ √ 1−n n−1 xn + y n + z n ≥ n−1 an + bn + n−1 cn Bài 3.13 Cho số thực dương a,b,c Chứng minh bất đẳng thức:   √ ab + bc + ca √ 1 (a + b + c) + + +4 2 ≥ + a b c a + b2 + c2 37 (38) PHƯƠNG PHÁP QUY NẠP §4 Phương pháp quy nạp I Lý thuyết Quy nạp toán học là phương pháp mạnh để chứng minh các phát biểu phụ thuộc vào số tự nhiên Cho (P (n))n≥0 là dãy các mệnh đề Phương pháp quy nạp toán học sử dụng để chứng minh P (n) đúng với n ≥ n0 với n0 là số tự nhiên Phương pháp quy nạp toán học (dạng yếu): Giả sử • P (n0 ) đúng • Với k ≥ n0 và P (k) đúng thì P (k + 1) đúng Khi đó P (n) đúng với n ≥ n0 Phương pháp quy nạp toán học (bước nhảy s): Cho s là số nguyên dương Giả sử • P (n0 ) , P (n0 + 1) , ,P (n0 + s − 1) đúng • Với k ≥ n0 , P (k) đúng kéo theo P (k + s) đúng Khi đó P (n) đúng với n ≥ n0 Phương pháp quy nạp toán học (Dạng mạnh): Giả sử • P (n0 ) đúng • Với k ≥ n0 , P (m) đúng với m mà n0 ≤ m ≤ k kéo theo P (k + 1) đúng Khi đó P (n) đúng với n ≥ n0 II Ví dụ minh họa Ví dụ 4.1 Chứng minh √ √ 1 n ≤ + √ + √ + ··· + √ ≤ n n (1.1) với số nguyên dương n Ta thấy (1.1) đúng với n = Giả sử (1.1) đúng với n = k ≥ 1, tức là √ √ 1 k ≤ + √ + √ + · · · + √ ≤ k, k (1.2) ta chứng minh (1.1) đúng với n = k + 1, tức là √ √ 1 k + ≤ + √ + √ + ··· + √ ≤ k + k+1 Thật vậy, dựa vào (1.2) ta có √ Mặt khác √ 1 1 k+√ ≤ + √ + √ + ··· + √ ≤2 k+√ k+1 k+1 k+1 √ √ 1 k+√ − k+1= √ −√ > 0, √ k+1 k+1 k+ k+1 38 (1.3) (39) PHƯƠNG PHÁP QUY NẠP và √ √ 1 =√ > k+1−2 k− √ −√ √ k+1 k+1 k+ k+1 Từ đó ta có đpcm Ví dụ 4.2 (VMO 2011) Chứng minh với ∀n ≥ 1,∀x > ta có bất đẳng thức: xn (xn+1 + 1) ≤ xn +  x+1 2n+1 (1.4) Đẳng thức xảy nào? • Với n = ta cần chứng minh: x(x2 + 1) ≤ x+1  x+1 3 ⇔ 8x(x2 + 1) ≤ (x + 1)4 hay là: x4 − 4x3 + 6x2 − 4x + ≥ ⇔ (x − 1)4 ≥ (đúng) Suy (1.4) đúng với n = Đẳng thức xảy x = • Giả sử (1.4) đúng với n = k ≥ 1, tức là: xk (xk+1 + 1) ≤ xk +  x+1 2k+1 (1.5) Ta cần chứng minh: xk+1 (xk+2 + 1) ≤ xk+1 +  x+1 2k+3 (1.6) Thật vậy, ta có:  x+1 2k+3  = x+1 2  x+1 2k+1  ≥ x+1 2 xk (xk+1 + 1) xk + Nên để chứng minh (1.6) ta cần chứng minh  x+1 2 xk (xk+1 + 1) xk+1 (xk+2 + 1) ≥ , xk + xk+1 + hay  x+1 2 (xk+1 + 1)2 ≥ x(xk+2 + 1)(xk + 1) Khai triển (1.7), biến đổi và rút gọn ta thu được: x2k+2 (x − 1)2 − 2xk+1 (x − 1)2 + (x − 1)2 ≥ ⇔ (x − 1)2 (xk+1 − 1)2 ≥ 0, bất đẳng thức cuối hiển nhiên đúng Đẳng thức có x = Vậy bài toán chứng minh 39 (1.7) (40) PHƯƠNG PHÁP QUY NẠP Ví dụ 4.3 Cho n ≥ số thực không âm a1 ≥ a2 ≥ · · · ≥ an có tổng Chứng minh rằng: !2 n n X X n − (i − 1)a2i ≤ (1.8) 2n i=1 i=1 (a1 + a2 )2 đúng a2 ≤ a1 , nên (1.8) đúng với n = • Với n = ta có: a22 ≤ • Giả sử (1.8) đúng với n = k, tức là: k X (i − 1)a2i i=1 k−1 ≤ 2k k X !2 (1.9) i=1 Ta chứng minh (1.8) đúng với n = k + 1, tức là: k+1 X i=1 ⇔ k X (i − 1)a2i + ka2k+1 i=1 Đặt x = k+1 X k (i − 1)a2i ≤ 2(k + 1) !2 i=1 k ≤ 2(k + 1) k X !2 + ak+1 (1.10) i=1 k 1P xi ,x ≥ xk+1 Sử dụng (1.9), ta cần chứng minh: k i=1 k−1 2k k X i=1 !2 + ka2k+1 k ≤ 2(k + 1) k X !2 + ak+1 i=1 k k(k − 1) x + kx2k+1 ≤ (kx + xk+1 )2 2(k + 1) ⇔ (k − 1)x2 + 2(k + 1)xk+1 ≤ (kx + xk+1 )2 ⇔ x2 + 2kx · xk+1 − (2k + 1)x2k+1 ≥ ⇔ ⇔ (x − xk+1 ) [x + (2k + 1)xk+1 ] ≥ (đúng) Vậy (1.8) luôn đúng Ví dụ 4.4 Cho các số nguyên dương phân biệt a1 , a2 , · · · , an Chứng minh a31 + a32 + · · · + a3n ≥ (a1 + a2 + · · · + an )2 (1.11) đúng với n ≥ Không tính tổng quát, ta giả sử a1 < a2 < · · · < an Với n = ta có a31 ≥ a21 nên (1.11) đúng với n = Giả sử (1.11) đúng với n = k và a1 < a2 < · · · < ak < ak+1 là các số nguyên dương Khi đó ak+1 ≥ ak + 1, nên (ak+1 − 1) ak+1 ak (ak + 1) ≥ = + + · · · + ak 2 Vì + + · · · + ak là tổng ak số nguyên dương đầu tiên, nên + + · · · + ak ≥ a1 + a2 + · · · + ak 40 (41) PHƯƠNG PHÁP QUY NẠP Suy (ak+1 − 1) ak+1 ≥ a1 + a2 + · · · + ak Hay a3k+1 ≥ 2ak+1 (a1 + a2 + · · · + ak ) + a2k+1 Do đó a31 + a32 + · · · + a3k + a3k+1 ≥ (a1 + a2 + · · · + ak )2 + 2ak+1 (a1 + a2 + · · · + ak ) + a2k+1 = (a1 + a2 + · · · + ak+1 )2 Suy (1.11) chứng minh Ví dụ 4.5 Cho 2n số thực dương a1 , a2 , · · · , an và b1 , b2 , · · · , bn Chứng minh n X (b1 + b2 + · · · + bk )bk k=1 a1 + a2 + · · · + ak <2 n X b2 i i=1 (1.12) Ta thấy bất đẳng thức (1.12) đúng với n = Giả sử bất đẳng thức (1.12) đúng với n ≥, tức là n X (b1 + b2 + · · · + bk )bk k=1 a1 + a2 + · · · + ak <2 n X b2 i i=1 , (1.13) (1.14) ta chứng minh (1.12) đúng với n + 1, tức là n+1 X (b1 + b2 + · · · + bk )bk k=1 a1 + a2 + · · · + ak <2 n+1 X b i i=1 Áp dụng cho trường hợp 2n số (a1 + a2 , a3 , a4 , an+1 , b1 + b2 , b3 , b4 , , bn+1 ) ta có n+1 X (b1 + b2 + · · · + bk )bk k=1 a1 + a2 + · · · + ak <2 n+1 X b i i=1 n n X (b1 + b2 )2 (b1 + b2 )2 X (b1 + · · · + bk ) bk b2k + ≤ +2 a1 + a2 a a a + · · · + ak + a2 k k=3 k=3 Bất đẳng thức tương đương với b21 (b1 + b2 ) b2 (b1 + b2 )2 b21 b2 + + ≤ +2 a1 a1 + a2 a1 + a2 a1 a2 2 2 b + 4b1 b2 + 3b2 b b ⇔ ≤ +4 a1 + a2 a1 a2 a2 a1 2 ⇔ b1 + 4b1 b2 + 3b2 ≤ b1 + b1 + b22 + 4b22 a1 a2 a2 a1 2 ⇔ 4b1 b2 ≤ b1 + b2 + b2 a1 a2 Bất đẳng thức cuối đúng, cách áp dụng bất đẳng thức AM-GM r a2 a1 a2 a1 b1 + b ≥ b · b = 4b1 b2 a1 a2 a1 a2 41 (42) PHƯƠNG PHÁP PHÂN TÍCH BÌNH PHƯƠNG SOS §5 Phương pháp phân tích bình phương SOS I Lý thuyết Một số tiêu chuẩn đánh giá Xét biểu thức: S = f (a, b, c) = Sa (b − c)2 + Sb (c − a)2 + Sc (a − b)2 đó Sa , Sb , Sc là các biểu thức chứa a, b, c là các số thực không âm Tính chất Nếu Sa , Sb , Sc ≥ thì S ≥ Tính chất Nếu a ≥ b ≥ c và Sb , Sb + Sc ≥ 0, Sb + Sa ≥ thì S ≥ Thật vậy: Vì a ≥ b ≥ c nên (c − a)2 = [(a − b) + (b − c)]2 ≥ (a − b)2 + (b − c)2 Suy   S ≥ Sa (b − c)2 + Sb (a − b)2 + (b − c)2 + Sc (a − b)2 = (Sa + Sb ) (b − c)2 + (Sb + Sc ) (a − b)2 ≥ Tính chất Nếu a ≥ b ≥ c và Sb ≤ 0, Sa + 2Sb ≥ 0, Sc + 2Sb ≥ thì S ≥ Thật vậy, ta có (a − c)2 = [(a − b) + (b − c)]2 ≤ (a − b)2 + (b − c)2 Suy S ≥ (Sa + 2Sb ) (b − c)2 + (Sc + 2Sb ) (a − b)2 ≥ Tính chất Nếu a ≥ b ≥ c và Sb ≥ 0, Sc ≥ 0, a2 Sb + b2 Sa ≥ thì S ≥ a−c b ≥ Suy Ta có a ≥ b ≥ c ⇒ b−c a "  2 #   c−a b2 2 2 Sa (b − c) + Sb (c − a) = (b − c) Sa + Sb ≥ (b − c) Sa + Sb ≥ b−c a Một số biểu diễn sở • a b + −2= (a − b)2 b a ab • a2 + b2 + c2 − ab − bc − ca = • p (a2 + b2 )  1 (a − b)2 + (b − c)2 + (c − a)2 (a − b)2 p − (a + b) = a + b + (a2 + b2 ) • a3 + b3 + c3 − 3abc =   (a + b + c) (a − b)2 + (b − c)2 + (c − a)2 • a3 + b3 − ab (a + b) = (a + b) (a − b)2 42 (43) PHƯƠNG PHÁP PHÂN TÍCH BÌNH PHƯƠNG SOS • a3 + b3 + c3 − a2 b − b2 c − c2 a =  1 (2a + b) (a − b)2 + (2b + c) (b − c)2 + (2c + a) (c − a)2 • (a + b) (b + c) (c + a) − 8abc = a (b − c)2 + b (c − a)2 + c (a − b)2 • a2 b + b2 c + c2 a − ab2 − bc2 − ca2 = −  1 (a − b)3 + (b − c)3 + (c − a)3 a b c (a − b)2 (b − c)2 (c − a)2 • + + − = + + b+c c+a a+b 2 (a + c) (b + c) (b + a) (c + a) (c + b) (a + c) II Các ví dụ Ví dụ 5.1 (IMO 2005) Cho x, y, z > thỏa mãn xyz ≥ Chứng minh x5 − x2 y5 − y2 z5 − z2 + + ≥ x5 + y + z y + z + x2 z + x2 + y Ta có (1) x5 − x2 xyz x4 − x2 yz x5 − x2 ≥ = x5 + y + z x5 + (y + z ) xyz x4 + yz (y + z ) 2x4 − x2 (y + z ) 2a2 − a (b + c) ≥ = , 2x4 + (y + z )2 2a2 + (b + c)2 với a = x2 , b = y , c = z Suy X 2a2 − a (b + c) X a (a − b) + a (a − c) = V T (1) ≥ 2a2 + (b + c)2 2a2 + (b + c)2 a, b, c a, b, c   X b a − = (a − b) 2a2 + (b + c)2 2b2 + (c + a)2 X c (2a + 2b + c) + a2 − ab + b2  = (a − b)2 ≥ (đúng) + (b + c)2 (2b2 + (c + a)2 ) 2a a, b, c Ví dụ 5.2 (VMO 2015) Cho a, b, c ≥ Chứng minh √ √ √  (a + b + c) ab + bc + ca + (a − b)2 + (b − c)2 + (c − a)2 ≥ (a + b + c)2 Bất đẳng thức cần chứng minh tương đương với  √ √ √  (a − b)2 + (b − c)2 + (c − a)2 ≥ (a + b + c) a + b + c − ab − bc − ca Đặt x = √ a,y = √ √ b,z = c ta cần chứng minh X 2   x2 − y ≥ x2 + y + z x2 + y + z − xy − yz − zx Mà x2 + y + z − xy − yz − zx = 1 (x − y)2 + (y − z)2 + (z − x)2 2 43 (2) (44) PHƯƠNG PHÁP PHÂN TÍCH BÌNH PHƯƠNG SOS Nên (2) trở thành Sx (y − z)2 + Sy (z − x)2 + Sz (x − y)2 ≥ 0, (3) với Sx = y + z + 4yz − x2 ,Sy = z + x2 + 4zx − y , Sz = x2 + y + 4xy − z Gải sử x ≥ y ≥ z , đó Sy ≥ 0,Sz ≥ và Sx + Sy = 2z + 4yz + 4zx ≥ Lại có (x − z)2 ≥ (y − z)2 nên V T (3) ≥ (Sx + Sy ) (y − z)2 ≥ nên (3) đúng Ví dụ 5.3 Cho a, b, c > Chứng minh rằng: a2 + b2 + c2 8abc + ≥ ab + bc + ca (a + b)(b + c)(c + a) Ta có  1 (a − b)2 + (b − c)2 + (c − a)2 (a + b) (b + c) (c + a) − 8abc = a (b − c)2 + b (c − a)2 + c (a − b)2 a2 + b2 + c2 − (ab + bc + ca) = Bất đẳng thức cần chứng minh tương đương với 2a (b − c)2 + 2b (c − a)2 + 2c (a − b)2 (a − b)2 + (b − c)2 + (c − a)2 ≥ ab + bc + ca (a + b) (b + c) (c + a)   X 2c ⇔ (a − b) − ≥0 ab + bc + ca (a + b) (b + c) (c + a) X ⇔ (a − b)2 Sc ≥ Với 2a − ab + bc + ca (a + b) (b + c) (c + a) 2b Sb = − ab + bc + ca (a + b) (b + c) (c + a) 2c Sc = − ab + bc + ca (a + b) (b + c) (c + a) Sa = Giả sử a ≥ b ≥ c ta có Sb , Sc ≥ và Sa + Sb = 2 − ≥ ab + bc + ca (a + c) (b + c) Suy (4) đúng Ví dụ 5.4 Cho a, b, c > Chứng minh rằng: 3(a3 + b3 + c3 ) a2 b2 c2 + + ≥ b c a a2 + b2 + c2 Ta giả sử b nằm a và c Ta có a2 b2 c2 (a − b)2 (b − c)2 (c − a)2 + + − (a + b + c) = + + b c a b c a 3 3(a + b + c ) (a + b) (a − b) + (b + c) (b − c)2 + (c + a) (c − a)2 − (a + b + c) = a2 + b2 + c2 a2 + b2 + c2 44 (4) (45) PHƯƠNG PHÁP PHÂN TÍCH BÌNH PHƯƠNG SOS Nên bất đẳng thức cần chứng minh tương đương với Sa (b − c)2 + Sb (c − a)2 + Sc (a − b)2 ≥ Với Sa = (5) b2 + c2 − ac a2 + c2 − ab a2 + b2 − bc ; Sb = ; Sc = c a b • Nếu a ≥ b ≥ c ⇒ Sa ≥ 0,Sc ≥ và Sa + 2Sb = Ta có b2 + c2 − ca a2 + b2 − bc +2 > c a c2 + a2 − ab b2 + c2 − ca +2 b a 2 b + c − ca c +a−b+2 ≥ b a c2 b2 + c2 − ca ≥ +a−b+2 , a a Sc + 2Sb = hay Sc + 2Sb ≥ a2 + 2b2 + 3c2 − ab − 2ca a Mặt khác    a2 + 3c2 ≥ 2ca   a2 + 2b2 > ab Do đó, ta có Sc + 2Sb > ⇒ a2 + 2b2 + 3c2 ≥ 2ca + ab • Nếu a ≤ b ≤ c ⇒ Sb ≥ thì ta có b2 + c2 − ca a2 + b2 − bc + >0 a c b2 + c2 − ca c2 + a2 − ab + > Sb + Sc = a b Sb + Sa = Ví dụ 5.5 (Iran 1996) Cho a, b, c là số dương Chứng minh   1 + + ≥ (ab + bc + ca) (a + b)2 (b + c)2 (c + a)2 Đặt x = a + b, y = b + c, z = c + a ⇒ a = : x+z−y x+y−z z+y−x ,b = ,c = 2 Bất đẳng thức cần chứng minh trở thành    1 2 ≥9 2xy + 2yz + 2zx − x − y − z + + x2 y z       1 1 2 2 2 ⇔ xy + yz + zx − x − y − z + + + x +y +z + + −9≥0 x2 y z x2 y z        1 x y2 y z2 2 2 ⇔ − (x − y) + (y − z) + (z − x) + + + + −2 + + −2 + x2 y z y x2 z2 y2   z x2 + + −2 ≥0 x2 z 2 2 ⇔ Sz (x − y)2 + Sx (y − z)2 + Sy (z − x)2 ≥ 0; Sx = − , Sy = − , Sz = − yz x zx y xy z 45 (46) PHƯƠNG PHÁP PHÂN TÍCH BÌNH PHƯƠNG SOS Giả sử x ≥ y ≥ z suy Sx ≥ 0,Sx ≥ Sy ≥ Sz Mặt khác y + z ≥ x ⇒ zx ≤ z (y + z) ≤ 2y ⇒ Sy ≥ y2 (x − z)2 ≥ (x − y)2 ⇔ (y − z) (y + z − x) ≥ (đúng) z Suy Sz (x − y)2 + Sx (y − z)2 + Sy (z − x)2 ≥ Sy (z − x)2 + Sz (x − y)2 y2 ≥ (x − y)2 Sy + Sz (x − y)2 z Mà     2 12 12 2y 2z 2 2 y Sy + z Sz = y − +z − = + −2≥0 zx y xy z zx xy ⇔ y + z ≥ xyz Do y + z ≥ x,y + z − yz ≥ yz suy y Sy + z Sz ≥ III Bài tập Bài 5.1 (Bất đẳng thức Schur) Cho a, b, c là các không âm Chứng minh : a3 + b3 + c3 + 3abc ≥ ab (a + b) + bc (b + c) + ca (c + a) Bài 5.2 Cho a, b, c ≥ và không có hai số nào đồng thời Chứng minh ab − bc + ca bc − ca + ab ca − ab + bc + + ≥ 2 2 2 b +c c +a a +b Bài 5.3 Cho a, b, c là các không âm Chứng minh : p p p a3 + b3 + c3 + 3abc ≥ ab (a2 + b2 ) + bc (b2 + c2 ) + ca (c2 + a2 ) Bài 5.4 Cho a, b, c là số không âm.Chứng minh : 2a2 + bc 2b2 + ca 2c2 + ab + + ≥ 2 2 b +c c +a a +b Bài 5.5 Cho a, b, c là các số dương Chứng minh   a3 + b3 + c3 + ab2 + bc2 + ca2 ≥ a2 b + b2 c + c2 a + 12abc Bài 5.6 (VN TST 2006) Chứng minh với số thực x,y,z ∈ [1; 2] , ta luôn có bất đẳng thức sau :     1 x y z (x + y + z) + + ≥6 + + x y z y+z z+x x+y Hỏi đẳng thức xảy và nào ? Bài 5.7 Bài 5.8 Cho các số thực a,b,c ∈ [0; 1] thỏa mãn a + b + c = 2 Chứng minh (a2 + b2 + c2 ) ≤ a3 + b3 + c3 ≤ (a2 + b2 + c2 ) − 3abc 3 46 (47) PHƯƠNG PHÁP PHÂN TÍCH BÌNH PHƯƠNG SOS Bài 5.9 Cho a, b, c > Chứng minh rằng: a2 + b2 + c2 a b c + ≥ + + ab + bc + ca b+c c+a a+b Bài 5.10 Cho a, b, c > Chứng minh rằng: p a2 + b b2 + c2 c2 + a2 3(a2 + b2 + c2 ) ≤ + + a+b b+c c+a Bài 5.11 Cho a, b, c > Chứng minh rằng: a2 + b2 b2 + c2 c2 + a2 3(a2 + b2 + c2 ) ≥ + + a+b+c a+b b+c c+a Bài 5.12 Cho a, b, c > Chứng minh rằng:   a b2 c2 3p + + +a+b+c≥ 3(a + b2 + c2 ) b+c c+a a+b Bài 5.13 Cho a, b, c > Chứng minh rằng: a b c + + ≥ b+c c+a a+b  ab + bc + ca 4− a + b2 + c2 Bài 5.14 Cho a, b, c > Chứng minh rằng: 2(a3 + b3 + c3 ) 9(a + b + c)2 + ≥ 33 abc a + b2 + c2 47  (48) PHƯƠNG PHÁP DỒN BIẾN §6 Phương pháp dồn biến I Lý thuyết Mục đích phương pháp này là làm giảm biến bất đẳng thức cần chứng minh II Ví dụ minh họa Ví dụ 6.1 Cho a, b, c ≥ −3 và a + b + c = Chứng minh 1 1 1 + + + + ≥ a2 b2 c2 a b c   P 1 Giả sử a = min{a, b, c} và đặt f (a, b, c) = − a2 a cyc Ta có −3 ≤ a ≤ và     b+c b+c 1 f (a, b, c) − f a, , = 2+ 2− + − − 2 b c (b + c)2 b c b+c (b − c)2 (b2 + 4bc + c2 − bc(b + c)) = b2 c2 (b + c)2 (b − c)2 ((3 − a)2 + bc(a − 1)) = b2 c2 (b + c)2   (3 − a)2 (a − 1) 2 (b − c) (3 − a) + ≥ 2 b c (b + c) (b − c) (3 − a)2 (a + 3) = ≥ 4b2 c2 (b + c)2 Tiếp theo ta chứng minh f (a,t,t) ≥ với t = 3−a a+b = Thật 2 2 + 2− − a t a t t2 + 2a2 − at2 − 2ta2 = a2 t2 (a + 3)(a − 1)2 ≥0 = 4a2 t2 f (a,t,t) = Vậy bài toán chứng minh Ví dụ 6.2 Cho các số thực dương a, b, c có tích Chứng minh 1 + + + ≥ a b c a+b+c 48 (49) PHƯƠNG PHÁP DỒN BIẾN Gải sử a = max{a, b, c}, suy bc ≤ ≤ a √ 1 Đặt f (a, b, c) = + + + và x = bc Ta có a b c a+b+c 1 6 + − + − b c x a + b + c a + 2x √ √ √ 2 √ 2 b− c b− c = − bc (a + b + c)(a + 2x)    √ √ b− c − = bc (a + b + c)(a + 2x) f (a, b, c) − f (a,x,x) = Ta có  (a + b + c)(a + 2x) − 6bc ≥ + 2x x2   + 2x − 6x2 ≥ · − 6x2 > x2 Suy  f (a, b, c) ≥ f (a,x,x) = f  ,x,x x2   Ta chứng minh f ,x,x ≥ x2 Tuy nhiên (1) tương đương với (1) (x − 1)2 (2x4 + 4x3 − 4x2 − x + 2) ≥ (2) Bất đẳng thức (2) đúng < x ≤ Ví dụ 6.3 Cho các số thực dương a, b, c thỏa mãn a2 + b2 + c2 = Chứng minh a3 (b + c) + b3 (c + a) + c3 (a + b) ≤ r Giả sử a = min{a, b, c} Đặt f (a, b, c) = a3 (b + c) + b3 (c + a) + c3 (a + b) và t = b2 + c2 ≥ a Xét P = f (a, b, c) − f (a,t,t) = a3 (b + c − 2t) + a(b3 + c3 − 2t3 ) + t2 (2bc − 2t2 ) Ta có (b + c)2 ⇒ t ≥ (b + c) ⇒ b + c − 2t ≤ 0, 2 3 b + c − 2t = (b + c)(b + c2 − bc) − 2t3 ≤ 2t(b2 + c2 − bc) − 2t3 = t(b − c)2 2bc − 2t2 = −(b − c)2 b2 + c2 ≥ Suy P ≤ + ta(b − c)2 − t2 (b − c)2 = t(b − c)2 (a − t) ≤ ⇒ f (a, b, c) ≤ f (a,t,t) Mặt khác, ta có a2 + 2t2 = nên f (a,t,t) ≤ ⇔ a3 t + t3 a ≤ − t4 ⇔ at(a2 + t2 ) ≤ − t4 ⇔ a2 t2 (3 − t2 )2 ≤ (3 − t4 )2 ⇔ (t2 − 1)2 (t4 − 3t2 + 3) ≥ (bđt này luôn đúng) 49 (50) PHƯƠNG PHÁP DỒN BIẾN Ví dụ 6.4 Cho các số thực không âm a, b, c Chứng minh (a2 + b2 + c2 )2 ≥ 4(a + b + c)(a − b)(b − c)(c − a) Kí hiệu (1) là bất đẳng thức cần chứng minh Không tính tổng quát, ta giả sử a = {a, b, c} +) b > c thì (1) luôn đúng +) Xét c > b Khi đó 2 và c − a c V T (1) > b2 + c2 Ta chứng minh : (a + b + c) (b − a) b (b + c) (2) Thật (2) ⇔ ab − a2 + b2 − ab + bc − ca b2 + bc ⇔ a (c + a) > (luôn đúng) Từ đó, suy  V P (1) bc (b + c) (c − b) = bc c2 − b2 Để chứng minh (1), ta chứng minh: b2 + c2 2 > 4bc b2 − c2  (3) c > Khi đó (3) trở thành b 2 2  t2 + > 4t t2 − ⇔ t4 − 4t3 + 2t2 + 4t + = ⇔ t2 − 2t − > (luôn đúng)  a =  √  và các hoán vị Suy (1) chứng minh Đẳng thức xảy c = + b Nếu b = ⇒ (3) đúng Xét b 6= ta đặt t = Ví dụ 6.5 (Hojoo Lee) Cho các số thực không âm a, b, c thỏa mãn ab + bc + ca = Chứng minh 1 + + ≥ a+b b+c c+a Giả sử c = max{a, b, c} Đặt f (a, b, c) = 1 + + a+b b+c c+a Nếu c = 0, ta có ab = nên a + b ≥ và 1 1 + + = +a+b a+b a b a+b a + b 3(a + b) = + + ≥1+ = a+b 4 2 f (a, b, c) = 50 (51) PHƯƠNG PHÁP DỒN BIẾN Xét   ,0 P = f (a, b, c) − f a + b, a+b      = + a+b 1 1    − + +a+b+  1 − ab − ab   a+b a+b+ b+ c+ a+b a+b a+b   1 = (a + b) + −1− 2 1+a 1+b + (a + b)2 a (2 − 2ab − ab(a + b)2 ) = (a + b) (1 + a2 )(1 + b2 )(1 + (a + b)2 ) Ta có − 2ab = 2(1 − ab) = 2(bc + ac) = 2c(a + b) ≥ ab(a + b)2 Suy P ≥ 0, từ đó ta có f (a, b, c) ≥ f (t, ,0) ≥ t III Bài tập Bài 6.1 Xét các số thực dương a, b, c thỏa mãn a + b + c = a) Tìm giá trị nhỏ biểu thức P = abc + 12 ab + bc + ca b) Chứng minh số nguyên k nhỏ cho abc + k k ≥1+ ab + bc + ca với a, b, c thỏa mãn điều kiện trên là k = 10 Bài 6.2 (Nguyễn Văn Quý) Cho các số thực dương a,b,c thỏa mãn điều kiện a + b + c = Chứng minh √ √ √ 3a2 + 4bc + + 3b2 + 4ca + + 3c2 + 4ab + ≥ 12 Bài 6.3 (Liu Quan Bao) Cho a, b, c, d là các số thực dương thỏa mãn điều kiện a + b + c = Chứng minh √ √ √ √ − 6ab + a2 + b2 + − 6bc + b2 + c2 + − 6ca + c2 + a2 ≥ Bài 6.4 số thực dương a,b,c thỏa mãn r (Yi Chang) Cho các r r điều kiện a + b + c = Chứng minh 7 + 3bc + (b − c)2 + + 3ca + (c − a)2 + + 3ab + (a − b)2 ≤ 12 12 12 Bài 6.5 (Võ Quốc số thực dương a,b,c thỏa pBá Cẩn) Cho các p p mãn điều kiện a + b + c = Chứng minh 2(a2 + b2 ) + 21c + 2(b2 + c2 ) + 21a + 2(c2 + a2 ) + 21b ≥ 15 Bài 6.6 (Phạm Thanh Tùng) Cho các số thực dương a,b,c có tổng Chứng minh p √ √ √ 3a2 − a + + 3b2 − b + + 3c2 − c + ≤ 6(a2 + b2 + c2 ) + 51 (52) PHƯƠNG PHÁP DỒN BIẾN Bài 6.7 (Phạm Kim Hùng) Cho các số thực a,b,c thỏa mãn điều kiện a + b + c = Chứng minh √ √ √ a + b − 2ab + b + c − 2bc + c + a − 2ca ≥ Để kết thúc bài viết, chúng tôi xin giới thiệu bài toán chặt và khó Liu Quan Ban Ngoài lời giải dồn biến mà chúng tôi biết và giới thiệu đây chúng tôi chưa thấy có lời giải nào khác Bài 6.8 (Liu Quan Bao) Cho các số thực a, b,c thỏa mãn điều kiện a + b + c = Chứng minh √ √ √ 1p 2a2 − a + + 2b2 − b + + 2c2 − c + ≥ 21(a2 + b2 + c2 ) + 99 Bài 6.9 Cho các số thực dương a, b, c có tích Chứng minh 1 13 25 + + + ≥ a b c a+b+c Bài 6.10 Cho các số thực dương a, b, c thỏa mãn a2 + b2 + c2 = Chứng minh 2(a + b − c) + abc ≤ 10 Bài 6.11 Cho các số thực dương a, b, c thỏa a + b + c = Chứng minh  2(a4 + b4 + c4 ) + 36 ≥ a3 + b3 + c3 + 3abc Bài 6.12 Cho các số thực a, b, c > Chứng minh √ 2(a2 + b2 + c2 ) + a2 b2 c2 ≥ (a + b + c)2 Bài 6.13 Cho các số thực a, b, c ≥ −3 thỏa mãn a + b + c = Chứng minh 1 1 1 + + ≥ + + a2 b2 c2 a b c Bài 6.14 Cho các số thực a, b, c thỏa mãn a2 + b2 + c2 = Chứng minh 2(a + b + c) − abc ≤ 10 Bài 6.15 (Iran 1996) Cho a, b, c là số dương Chứng minh :   1 (ab + bc + ca) ≥ + + (a + b) (b + c) (c + a) 52 (53) Chương Các phương pháp chứng minh bất đẳng thức đại 53 (54) PHƯƠNG PHÁP P, Q, R §1 Phương pháp p, q, r I Lý thuyết Bất đẳng thức Schur Cho các số thực không âm x,y,z và số thực dương r Khi đó, ta có bất đẳng thức sau xr (x − y)(x − z) + y r (y − x)(y − z) + z r (z − x)(z − y) ≥ Đẳng thức xảy x = y = z z = 0,x = y và các hoán vị Hệ a) Xét r = ta có các dạng sau • x3 + y + z + 3xyz ≥ xy(x + y) + yz(y + z) + zx(z + x) • 4(x3 + y + z ) + 15xyz ≥ (x + y + z)3 • xyz ≥ (x + y − z)(y + z − x)(z + x − y) 9xyz • x2 + y + z + ≥ 2(xy + yz + zx) x+y+z • (x + y + z)3 + 9xyz ≥ 4(x + y + z)(xy + yz + zx) b) r = ta có các dạng sau • x4 + y + z + xyz(x + y + z) ≥ xy(x2 + y ) + yz(y + z ) + zx(z + x2 ) • 6xyz(x + y + z) ≥ [2(xy + yz + zx) − (x2 + y + z )] (x2 + y + z + xy + yz + zx) Một số biểu diễn đa thức đối xứng ba biến qua p, q, r Cho các số thực a, b, c Đặt p = a + b + c, p = ab + bc + ca và r = abc Khi đó ta có các biểu diễn sau • ab(a + b) + bc(b + c) + ca(c + a) = pq − 3r • (a + b)(b + c)(c + a) = pq − r • ab(a2 + b2 ) + bc( b2 + c2 ) + ca(c2 + a2 ) = p2 q − 2q − pr • (a + b)(a + c) + (b + c)(b + a) + (c + a)(c + b) = p2 + q • a2 + b2 + c2 = p2 − 2q • a3 + b3 + c3 = p3 − 3pq + 3r • a4 + b4 + c4 = p4 − 4p2 q + 2q + 4pr • a2 b2 + b2 c2 + c2 a2 = q − 2pr • a3 b3 + b3 c3 + c3 a3 = q − 3pqr + 3r2 • a4 b4 + b4 c4 + c4 a4 = q − 4pq r + 2p2 r2 + 4qr2 54 (55) PHƯƠNG PHÁP P, Q, R Một số đánh giá p, q, r Dựa vào các bất đẳng thức ba biến và bất đẳng thức Schur ta có các đánh giá sau • (a + b + c)2 ≥ 3(ab + bc + ca) ⇒ p2 ≥ 3q • (a + b + c)3 ≥ 27abc ⇒ p3 ≥ 27r • (ab + bc + ca)2 ≥ 3abc(a + b + c) ⇒ q ≥ 3pr • (a + b + c)(ab + bc + ca) ≥ 9abc ⇒ pq ≥ 9r • p3 + 9r ≥ 4pq (BĐT Schur với r = 1) • p4 + 4q + 6pr ≥ 5p2 q (BĐT Schur với r = 2)   p(4q − p2 ) (BĐT Schur với r = 1) • r ≥ max 0,   (4q − p2 )(p2 − q) (BĐT Schur với r = 2) • r ≥ max 0, 6p II Một số ví dụ Ví dụ 1.1 Cho các số thực dương a, b, c thỏa mãn a2 + b2 + c2 = Chứng minh 5(a + b + c) + ≥ 18 abc Ta có p2 − 2q = và bất đẳng thức cần chứng minh trở thành 5p + Ta có q ≥ 3rq ⇒ ≥ 18 r (1) 9p 36p ≥ = r q (p − 3)2 Do đó V T (1) ≥ 5p + 36p (p2 − 3)2 Nên ta chứng minh 5p + 36p ⇔ 5p5 − 18p4 − 30p3 + 108p2 + 81p − 162 ≥ (p2 − 3)2 ⇔ (p − 3)2 (5p3 + 12p2 − 3p − 18) ≥ Ta có < p2 ≤ 3(a2 + b2 + c2 ) = ⇒ √ < p ≤ ⇒ 5p3 + 12p2 − 3p − 18 > 0, nên (2) luôn đúng 55 (2) (56) PHƯƠNG PHÁP P, Q, R Ví dụ 1.2 Cho các số thực dương a, b, c thỏa mãn ab + bc + ca = Chứng minh 1 + + ≥ a+b b+c c+a Bất đẳng thức cần chứng minh tương đương với (a + b)(b + c) + (b + c)(c + a) + (c + a)(a + b) ≥ (a + b)(b + c)(c + a) 2 p +1 p +q ≥ ⇔ ≥ ⇔ 2p2 − 5p + 5r + ≥ ⇔ pq − r p−r Nếu p ≥ thì 2p2 − 5p + + 5r = (p − 2)(2p − 1) + 5r ≥ 0, suy (1) đúng √ 4pq − p3 4p − p3 Xét ≤ p < ta có r ≥ = Nên để chứng minh (1) ta chứng minh 9 2p2 − 5p + + 4p − p3 ≥ ⇔ (p − 2)(5p2 − 8p + 9) ≤ (luôn đúng) Ví dụ 1.3 Chứng minh x, y, z > thì (xy + yz + zx)( 1 + + )≥ 2 (x + y) (y + z) (z + x) Ta có (x + y)2 (y + z)2 + (y + z)2 (z + x)2 + (z + x)2 (x + y)2 = ((x + y)(y + z) + (y + z)(z + x) + (z + x)(x + y))2 − 4(x + y)(y + z)(z + x)(x + y + z) = (p2 + q)2 − 4p(pq − r) Do đó bất đẳng thức cần chứng minh tương đương với (p2 + q)2 − 4p(pq − r) ) ≥ (pq − r)2 4 2 ⇔ 4p q − 17p q + 4q + 34pqr − 9r2 ≥ ⇔ 3pq(p3 − 4pq + 9r) + q(p4 − 5p2 q + 4q + 6pr) + r(pq − 9r) ≥ q( Bất đẳng thức cuối đúng nên ta có đpcm III Bài tập Bài 1.1 Cho các số dương a, b, c thỏa abc = Chứng minh : 1+ ≥ a+b+c ab + bc + ca 56 (1) (57) PHƯƠNG PHÁP P, Q, R Bài 1.2 Cho các số thực dương a, b, c thỏa mãn ab + bc + ca + 6abc = Chứng minh a + b + c + 3abc ≥ Bài 1.3 Cho a, b, c là các số thực không âm thỏa mãn ab + bc + ca = 3.Chứng minh rằng: a3 + b3 + c3 + 7abc ≥ 10 Bài 1.4 Cho a, b, c > thỏa a + b + c = 3.Chứng minh rằng:   12 1 3+ ≥5 + + abc a b c Bài 1.5 Cho a, b, c là các số thực dương thỏa mãn a2 + b2 + c2 = 3.Chứng minh rằng: 1 + + ≥ 2−a 2−b 2−c Bài 1.6 Cho a, b, c là các số thực không âm thỏa mãn a + b + c = 3.Chứng minh rằng: 1 + + ≤ − ab − bc − ca Bài 1.7 Cho các số thực không âm x, y, z thỏa mãn xy + yz + zx + xyz = Chứng minh x2 + y + z + 5xyz ≥ 57 (58) PHƯƠNG PHÁP SỬ DỤNG TIẾP TUYẾN VÀ CÁT TUYẾN §2 Phương pháp sử dụng tiếp tuyến và cát tuyến I Lý thuyết Hàm lồi - Dấu hiệu hàm lồi Định nghĩa Cho hàm số y = f (x) liên tục [a; b] và có đồ thị là (C) Khi đó ta có hai điểm A(a; f (a)), B(b; f (b)) nằm trên đồ thị (C) i) Đồ thị (C) gọi là lồi trên (a; b) tiếp tuyến điểm nằm trên cung AB luôn nằm phía trên đồ thị (C) ii) Đồ thị (C) gọi là lõm trên (a; b) tiếp tuyến điểm nằm trên cung AB luôn nằm phía đồ thị (C) y y M B A x O A x O B Đồ thị hàm số lõm Đồ thị hàm số lồi M Định lí (Dấu hiệu hàm lồi, lõm) Cho hàm số y = f (x) có đạo hàm cấp hai liên tục trên (a; b) Khi đó • Nếu f 00 (x) > ∀x ∈ (a; b) thì đồ thị hàm số lõm trên (a; b) • Nếu f 00 (x) < ∀x ∈ (a; b) thì đồ thị hàm số lồi trên (a; b) Bất đẳng thức tiếp tuyến - Bất đẳng thức cát tuyến Định lí (Bất đẳng thức tiếp tuyến) Cho hàm số y = f (x) liên tục và có đạo hàm đến cấp hai trên [a;b] Khi đó ta có • Nếu f 00 (x) ≥ ∀x ∈ [a; b] thì f (x) ≥ f (x0 )(x − x0 ) + f (x0 ) ∀x0 ∈ [a; b] • Nếu f 00 (x) ≤ ∀x ∈ [a; b] thì f (x) ≤ f (x0 )(x − x0 ) + f (x0 ) ∀x0 ∈ [a; b] Đẳng thức hai bất đẳng thức trên xảy x = x0 Chứng minh: Ta chứng minh trường hợp thứ nhất, trường hợp thứ hai chứng minh tương tự Xét hàm số g(x) = f (x) − f (x0 )(x − x0 ) − f (x0 ), x ∈ [a; b] Ta có : g (x) = f (x) − f (x0 ) ⇒ g 00 (x) = f 00 (x) ≥ ∀x ∈ [a; b] Suy g (x) = ⇔ x = x0 và g (x) đổi dấu từ − sang + x qua x0 nên ta có : g(x) ≥ g(x0 ) = ∀x ∈ [a; b] Định lí (Bất đẳng thức cát tuyến) Cho hàm số y = f (x) liên tục và có đạo hàm đến cấp hai trên [a; b] Khi đó 58 (59) PHƯƠNG PHÁP SỬ DỤNG TIẾP TUYẾN VÀ CÁT TUYẾN • Nếu f 00 (x) ≥ ∀x ∈ [a; b] thì f (x) ≥ f (a) − f (b) (x − a) + f (a) ∀x0 ∈ [a; b] a−b • Nếu f 00 (x) ≤ ∀x ∈ [a; b] thì f (x) ≤ f (a) − f (b) (x − a) + f (a) ∀x0 ∈ [a; b] a−b Đẳng thức các bất đẳng thức trên có và x = a x = b II Các ví dụ minh họa Ví dụ 2.1 Cho các số thực dương a,b,c thỏa a + b + c = Chứng minh √ Xét hàm số f (x) = √ b c a +√ +√ ≤√ a2 + b2 + c2 + 10 x với x ∈ (0; 1) Ta có: +1 x2 3x ⇒ f 00 (x) = − q < ∀x ∈ (0; 1) f (x) = q (x2 + 1)3 (x2 + 1)5 Nên ta có: 1 f (a) ≤ f ( )(a − ) + f ( ) 3 1 f (b) ≤ f ( )(b − ) + f ( ) 3 1 f (c) ≤ f ( )(c − ) + f ( ) 3 Suy :   1 f (a) + f (b) + f (c) ≤ f (a + b + c − 1) + 3f ( ) = √ 3 10 Đẳng thức xảy a = b = c = Ví dụ 2.2 Cho các số thực dương a, b, c thỏa : a2 + b2 + c2 = Chứng minh √ Xét hàm số : f (x) = √ 1 +√ +√ ≥ 1 + 8a + 8b + 8b √ , < a ≤ Ta có : + 8a f (x) = − q (1 + 8x)3 48 ⇒ f 00 (x) = q (1 + 8x)5 Nên ta có : f (a) ≥ f (1)(a − 1) + f (1) f (b) ≥ f (1)(b − 1) + f (1) f (c) ≥ f (1)(c − 1) + f (1) 59 √ > ∀x ∈ (− ; 3] (60) PHƯƠNG PHÁP SỬ DỤNG TIẾP TUYẾN VÀ CÁT TUYẾN Suy f (a) + f (b) + f (c) ≥ f (1)(a + b + c − 3) + 3f (1) (∗) Mặt khác (a + b + c)2 ≤ 3(a2 + b2 + c2 ) = ⇒ −3 ≤ a + b + c ≤ ⇒ a + b + c − ≤ < nên từ (*) ta suy : f (a) + f (b) + f (c) ≥ 3f (1) = 27 Nhận xét Dấu hiệu giúp chúng ta nhận phương pháp trên là bất đẳng thức cần chứng minh có dạng f (a1 ) + f (a2 ) + · · · + f (an ) ≥ k và f (1) = − f (a1 ) + f (a2 ) + · · · + f (an ) ≤ k, đó (i = 1, ,n) là các số thực cho trước Trong số trường hợp BĐT chưa có dạng trên, ta phải thực số phép biến đổi đưa dạng trên.Chúng ta cần chú ý số dấu hiệu sau • Nếu bất đẳng thức có dạng f (a1 ) · f (a2 ) · · · f (an ) ≥ k thì ta lấy ln hai vế • Nếu bất đẳng thức cần chứng minh đồng bậc thì ta có thể chuẩn hóa Tùy thuộc vào bài toán mà ta lựa chọn cách chuẩn hóa phù hợp Ví dụ 2.3 Cho các số thực dương a,b,c thỏa : a + b + c = Tìm GTLN biểu thức :  b  c  a √ √ √ P = a + + a2 b + + b2 c + + c2 Ta có :     √ √ √ ln P = b ln(a + + a2 ) + c ln b + + b2 + a ln c + + c2 √  Xét hàm số : f (x) = ln x + + x2 , < x < Ta có : −x < 0, ∀x ∈ (0; 1) f (x) = √ ⇒ f 00 (x) = q x2 + (1 + x ) Suy : f (a) ≤ f (1) (a − 1) + f (1) = f (1)a + f (1) − f (1) Do đó bf (a) ≤ f (1)ab + [f (1) − f (1)] b Tương tự cf (b) ≤ f (1)cb + [f (1) − f (1)] c và af (c) ≤ f (1)ac + [f (1) − f (1)] a Công các bất đẳng thức theo vế ta ln P ≤ f (1) (ab + bc + ca − (a + b + c)) + f (1)(a + b + c) ≤ ln(1 + √ 2) (Do ab + bc + ca ≤ =√a + b + c) √ Suy ln P ≤ ln(1 + 2)√⇒ P ≤ (1 + 2)3 Đẳng thức xảy ⇔ a = b = c = Vậy GTLN P = (1 + 2)3 60 (61) PHƯƠNG PHÁP SỬ DỤNG TIẾP TUYẾN VÀ CÁT TUYẾN Ví dụ 2.4 Cho x,y > thỏa x+y+z = Tìm GTNN biểu thức P = x−y +y −z +z −x Áp dụng bất đẳng thức AM-GM, ta có : P ≥ √ 3 xy y z z x Đặt A = xy y z z x ⇒ ln A = y ln x + z ln y + x ln z Vì hàm số f (t) = ln t có f 00 (t) = − < Suy t    1 x− + f ( ) = 3x − − ln ln x ≤ f 3 Do đó ln A ≤ y(3x − − ln 3) + z(3y − − ln 3) + x(3z − − ln 3) = 3(xy + yz + zx) − − ln ≤ (x + y + z)2 − − ln = −3 ln √ 1 Suy A ≤ ⇒ P ≥ 3 Đẳng thức xảy ⇔ x = y = z = 3 √ Vậy GTNN P = 3 Ví dụ 2.5 Cho a,b,c ≥ thỏa a + b + c = Tìm GTNN biểu thức P = aa + bb + cc Xét hàm số f (t) = tt , ≤ t ≤ Ta có : ln f (t) = t ln t lấy đạo hàm hai vế ta f (t) = (1 + ln t)f (t) ⇒ ln f (t) = ln f (t) + ln (ln t + 1) f 00 (t) f (t) 1 ⇒ = + = + ln t + f (t) f (t) t(ln t + 1) t(ln t + 1)   1 00 ⇒ f (t) = (1 + ln t)f (t) + ln t + > ∀t ∈ [ ; 1] t(1 + ln t)   Vì a,b,c ∈ ; nên áp dụng bất đẳng thức tiếp tuyến, ta có : 2 2 f (a) ≥ f ( )(a − ) + f ( ) 3 2 f (b) ≥ f ( )(b − ) + f ( ) 3 2 f (c) ≥ f ( )(c − ) + f ( ) 3 Cộng ba bất đẳng thức trên ta có : r 2 f (a) + f (b) + f (c) ≥ f ( ) (a + b + c − 2) + 3f ( ) = 3 r Vậy GTNN P = 3 đạt ⇔ a = b = c = 61 (62) PHƯƠNG PHÁP SỬ DỤNG TIẾP TUYẾN VÀ CÁT TUYẾN Ví dụ 2.6 Cho tam giác ABC có góc không nhỏ tan 2π Chứng minh : √ A B C + tan + tan ≥ − 2 2π π Không tính tổng quát, ta giả sử A ≥ >B≥C⇒C≤  π có Hàm số f (x) = tan x, x ∈ 0;  π f 00 (x) > ∀x ∈ 0; Áp dụng BĐT tiếp tuyến, ta có A π A π π f ( ) ≥ f ( )( − ) + f ( ) 3 π B π π B f ( ) ≥ f ( )( − ) + f ( ) 12 12 12 C π C π π f ( ) ≥ f ( )( − ) + f ( ) 12 12 12 Suy       h   i  A 2π  A B C A+B+C π π π π f +f +f ≥ f ( )−f ( ) − +f ( ) − 2 12 12 2 π  π +f + 2f 12 π  π A π A+B+C π Do f − f0 > 0; − ≥ và = nên ta có : 12 2       π π  √ A B C f + 2f = − +f +f ≥f 2 12 Đẳng thức xảy ⇔ A = 2π π ; B = C = và các hoán vị Ví dụ 2.7 Cho các số thực không âm a,b,c thỏa max {a,b,c} ≥ và a + b + c = Tìm GTNN biểu thức : P = √ + 3a2 + √ + 3b2 + √ + 3c2 Không tính tổng quát, ta giả sử a = max {a,b,c} ⇒ a ≥ ,c ≤ √ Xét hàm số f (x) = + 3x2 , x ∈ (0; 1) có 2x f (x) = q (1 + 3x2 )2 − 2x2 ⇒ f 00 (x) = q 62 (1 + 3x2 )5 > ∀x ∈ (0; 1) (63) PHƯƠNG PHÁP SỬ DỤNG TIẾP TUYẾN VÀ CÁT TUYẾN Áp dụng bất đẳng thức tiếp tuyến, ta có :      3 f (a) ≥ f a− +f 4      1 f (b) ≥ f b− +f 8      1 f (c) ≥ f c− +f 8 Cộng các bất đẳng thức trên ta có            3 0 −f a− +f + 2f f (a) + f (b) + f (c) ≥ f 4 √     √ 3 172 + 67 ≥f + 2f = Đẳng thức xảy ⇔ a = ; b = c = và các hoán vị √ √4 172 + 67 Vậy P = Nhận xét Trong số trường hợp đồ thị hàm số y = f (x) có khoảng lồi, lõm trên [a; b] ta có đánh giá : f (x) ≥ f (x0 )(x − x0 ) + f (x0 ) ,x0 ∈ (a; b) Ví dụ 2.8 Cho a,b,c ∈ R và a + b + c = Chứng minh : a4 + b4 + c4 ≥ 2(a3 + b3 + c3 ) Bất đẳng thức đã cho tương đương với    a4 − 2a3 + b4 − 2b3 + c4 − 2c3 ≥ ⇔ f (a) + f (b) + f (c) ≥ Trong đó f (x) = x4 − 2x3 Ta thấy f 00 (x) = 12x2 − 12x nên đồ thị hàm số f có khoảng lồi và khoảng lõm đó ta không thể áp dụng BĐT tiếp tuyến Tuy nhiên ta có thể đánh giá f (x) qua tiếp tuyến nó điểm có hoành độ x = (vì đẳng thức xảy a = b = c = 2) Ta có tiếp tuyến đồ thị hàm số y = f (x) điểm có hoành độ x = là: y = 8x − 16  f (x) − (8x − 16) = x4 − 2x3 − 8x + 16 = (x − 2)2 x2 − 2x + ≥ ∀x ∈ R Suy f (a) + f (b) + f (c) ≥ (a + b + c) − 48 = Vậy bài toán chứng minh Ví dụ 2.9 (Ba Lan 1996) Cho a,b,c ≥ − a2 và a + b + c = Chứng minh rằng: a b c + + ≤ +1 b +1 c +1 10 63 (64) PHƯƠNG PHÁP SỬ DỤNG TIẾP TUYẾN VÀ CÁT TUYẾN Ta thấy đẳng thức xảy a = b = c = và bất đẳng thức đã cho có dạng: f (a)+f (b)+f (c) ≤ x đó f (x) = với x ∈ [− ; ] 10 x +1 36x + Tiếp tuyến đồ thị hàm số y = f (x) điểm có hoành độ x = là : y = 50 Ta có: 36x + 36x + x (3x − 1)2 (4x + 3) = ≥ ∀x ∈ [− − f (x) = − ; ] 50 50 x +1 50(x2 + 1) Vậy a2 a b c 36(a + b + c) + 9 + + ≤ = +1 b +1 c +1 50 10 Bài toán chứng minh Ví dụ 2.10 Cho a,b,c là độ dài ba cạnh tam giác Chứng minh :   1 1 + + + ≥4 + + a b c a+b+c a+b b+c c+a Không làm tính tổng quát ta giả sử , đó Bđt đã cho trở thành 5a − 5a − 5c − + + ≤ a − a2 b − b2 c − c2 Vì a,b,c là độ dài ba cạnh tam giác và a + b + c = suy a,b,c ∈ (0; ) Ta có (3a − 1)2 (2a − 1) 5a − − (18a − 3) = ≤ ∀a ∈ (0; ) 2 a−a a−a 5a − 1 ≤ 18a − ∀a ∈ (0; ) a−a Ta có hai bất đẳng thức tương tự Cộng các bất đẳng thức này lại với ta có Suy 5a − 5a − 5c − + + ≤ 18(a + b + c) − = a − a2 b − b2 c − c2 Bài toán chứng minh Đẳng thức xảy khia = b = c = Ví dụ 2.11 (Trung Quốc 2005) Cho a,b,c > và a + b + c = Chứng minh rằng:  10 a3 + b3 + c3 ) − 9(a5 + b5 + c5 ≥ Giả sử a ≥ b ≥ c Xét hàm số f (x) = 10x3 − 9x4 , x ∈ (0; 1) có f (x) = 30x2 − 45x4 ⇒ f 00 (x) = 60x − 180x3 r 00 Suy f (x) = ⇔ x = x0 = đồng thời f 00 (x) > ∀x ∈ (0; x0 ) và f 00 (x) < ∀x ∈ (x0 ; 1) 64 (65) PHƯƠNG PHÁP SỬ DỤNG TIẾP TUYẾN VÀ CÁT TUYẾN • Nếu a < x0 Áp dụng bất đẳng thức tiếp tuyến ,ta có:      1 f (a) ≥ f a− +f 3      1 f (b) ≥ f b− +f 3      1 f (c) ≥ f c− +f 3 Suy     1 f (a) + f (b) + f (c) ≥ f (a + b + c − 1) + 3f = 3 • Nếu a > x0 Áp dụng bất đẳng thức tiếp tuyến và cát tuyến ta có: f (1) − f (x0 ) (a − 1) + f (1) > f (1) = 1 − x0 f (b) ≥ f (0) (b − 0) + f (0) = f (c) ≥ f (0) (c − 0) + f (0) = f (a) ≥ Do đó f (a) + f (b) + f (c) > Vậy bài toán chứng minh Ví dụ 2.12 Cho ∆ABC nhọn Tìm GTLN biểu thức F = sin A · sin2 B · sin3 C Ta có ln F = ln sin A + ln sin B + ln sin C π Xét hàm số f (x) = ln sin x, x ∈ (0; ), ta có  π f (x) = cot x ⇒ f 00 (x) = − ∀x ∈ 0; sin x Áp dụng bất đẳng thức tiếp tuyến với ∆M N P nhọn, ta có : f (A) ≤ f (M ) (A − M ) + f (M ) = (A − M ) cot M + ln sin M f (B) ≤ f (N ) (B − N ) + f (N ) = (B − N ) cot N + ln sin N f (C) ≤ f (P ) (C − P ) + f (P ) = (C − P ) cot P + ln sin P Suy tan M · f (A) + tan N · f (B) + tan P · f (C) ≥ tan M · ln sin M + tan N · ln sin N + tan P · ln sin P Chọn ba góc M, N, P cho : tan M tan N tan P = = = k ⇒ tan M = k; tan N = 2k; tan P = 3k Mặt khác : tan M + tan N + tan P = tan M tan N tan P , suy tan M 6k = 6k ⇒ k = ⇒ sin M = √ = √ ; sin N = √ ; sin P = √ 2 10 + tan M 65 (66) PHƯƠNG PHÁP SỬ DỤNG TIẾP TUYẾN VÀ CÁT TUYẾN Do đó, ta có 27 f (A) + f (B) + f (C) ≤ ln √ + ln √ + ln √ = ln √ , 10 25 27 √ Đẳng thức xảy ⇔ A = M ; B = N ; C = P 25 27 Vậy GTLN F = √ 25 hay F ≤ III Bài tập Bài 2.1 (Albania 2002) Cho a,b,c > Chứng minh : √ √ 1 1+ √ (a + b2 + c2 )( + + ) ≥ a + b + c + a2 + b2 + c2 a b c 3 π Bài 2.2 Cho n số thực x1 ,x2 , ,xn thuộc khoảng (0; ) thỏa : tan x1 + tan x2 + · · · + tan xn ≤ n Chứng minh sin x1 · sin x2 · · · sin xn ≤ √ 2n Bài 2.3 Cho các số thực a,b,c > thoả mãna + b + c = Chứng minh : a b c + + ≥ + bc + ac + ab 10 Bài 2.4 Cho a,b,c > Chứng minh : (b + c − a)2 (c + a − b)2 (a + b − c)2 + + ≥ 2 (b + c) + a2 (c + a) + b2 (a + b) + c2 Bài 2.5 Cho a, b, c > −1 và a + b + c = Tìm giá trị nhỏ S = a3 + b3 + c3 + 5(a2 + b2 + c2 ) Bài 2.6 Cho a + b + c = −6 với a, b, c < −1 Tìm giá trị lớn S= Bài 2.7 Cho a, b, c > − a2 a b c + + +a+1 b +b+1 c +c+1 và 4(ab + bc + ca) + a + b + c ≥ 15 Tính giá trị nhỏ biểu thức S = a3 + b3 + c3 + 2(a2 b + b2 c + c2 a) 66 (67) PHƯƠNG PHÁP SỬ DỤNG TIẾP TUYẾN VÀ CÁT TUYẾN   Bài 2.8 Cho a, b, c ∈ 0, và −2(ab + bc + ca) + 3(a + b + c) = Tìm giá trị nhỏ S = 2(a3 + b3 + c3 ) − 3(a2 b + b2 c + c2 a) Bài 2.9 Cho các số dương a, b, c thỏa mãn ab + bc + ca = Tìm giá trị nhỏ S = 5(a3 + b3 + c3 ) + 2(a2 b + b2 c + c2 a)  1√ Bài 2.10 Cho a, b, c ∈ , và ab + bc + ca + = 4(a + b + c) Tìm giá trị nhỏ   1 b a c T = 2+ 2+ 2− + + a b c a c b  Bài 2.11 Cho các số dương a, b, c thỏa mãn 3(ab + bc + ca) − (a + b + c) ≥ Tính giá trị nhỏ A = a5 + b5 + c5 + 3(a4 b + b4 c + c4 a) Bài 2.12 Cho a, b, c ∈ [0,4] thỏa mãn ab + bc + ca ≥ Tìm giá trị nhỏ √ √ √ B = (a2 + b2 + c2 ) + b 3a + + c 3b + + a 3c + Bài 2.13 Cho tam giác ABC nhọn Tìm GTNN biểu thức : F = tan A + tan B + tan C Bài 2.14 Cho x, y, z > thỏa x + y + z = Tìm GTNN : p √ P = x3 + + y + + z 67 (68) Chương Một số chuyên đề §1 Ứng dụng điều kiện có nghiệm phương trình bậc ba chứng minh bất đẳng thức I Lý thuyết Mở đầu Định lí Vi-ét đảo phương trình bậc hai phát biểu sau: Định lí Nếu hai số a,b có tổng là S và tích là P thì hai số đó là hai nghiệm phương trình x2 − Sx + P = (3.1) Phương trình (3.1) có nghiệm và ∆ = S − 4P ≥ hay S ≥ 4P Mà S = a + b,P = ab nên ta có điều kiện để tồn hai số a,b (tức là phương trình (3.1) có nghiệm) là : (a + b)2 ≥ 4ab Đây chính là bất đẳng thức quen thuộc Nếu a, b ≥ thì ta thu bất đẳng thức AM-GM Tương tự định lí Vi-ét đảo phương trình bậc ba sau: Đặt m = a + b + c,n = ab + bc + ca,p = abc Khi đó, a, b, c là nghiệm phương trình x3 − mx2 + nx − p = (3.2) Ta tìm điều kiện để phương trình (3.2) có ba nghiệm (có thể trùng nhau) Một số kết Đặt: x = y + m2 9mn − 2m3 − 27p m ;α= − n; β = Từ (3.2) ta thu phương trình 3 27 y − αy + β = (3.3) Số nghiệm (3.3) chính là số giao điểm đồ thị (C) : f (y) = y − αy + β với trục hoành Ta có: f (y) = 3y − α • Nếu α < thì f (y) > 0, ∀y nên phương trình (3.3) có đúng nghiệm • Nếu α = thì phương trình (3.3) có nghiệm bội ba 68 (69) ỨNG DỤNG ĐIỀU KIỆN CÓ NGHIỆM CỦA PHƯƠNG TRÌNH BẬC BA TRONG CHỨNG MINH BẤT ĐẲNG THỨC r r α α ; y2 = , đó • Nếu α > thì f (y) = có hai nghiệm y1 = − 3 r r 2α α 2α α f (y1 ) = + β, f (y2 ) = − + β 3 3 Suy f (y1 ) f (y2 ) = β − 4α3 27β − 4α3 = 27 27 Do đó, phương trình (3.3) có ba nghiệm và khi: f (y1 ) f (y2 ) ≤ ⇔ 4α3 − 27β ≥ Hay là: 9mn − 27p − 2m q ≤ (m2 − 3n)3 Kết Cho các số thực a, b, c Đặt a + b + c = m, ab + bc + ca = n, abc = p Khi đó, ta có đánh giá sau: q 9mn − 27p − 2m3 ≤ (m2 − 3n)3 (3.4) Với a, b, c ≥ 0, đặt a + b + c = 3u, ab + bc + ca = 3v và abc = w3 Vì q (a + b + c) ≥ (ab + bc + ca) ≥ (abc)2 nên ta có u ≥ v ≥ w Khi đó (3.4) trở thành q · 3u · 3v − 27w3 − · 27u3 ≤ (9u2 − 9v )3 Hay q ≤ (u2 − v )3 3uv − w − 2u (3.5) Chia hai vế (3.5) cho u3 ta có  v 2 u −  w 3 u s  v 2 3 −2 ≤2 1− u Hay là s −2 1−  v 2  u ≤3  v 2 u −  w 3 u s −2≤2 1−  v 2 3 u Suy  v 2 u s s  v 2 3  w 3  v 2  v 2 3 −2 1− −2≤ ≤3 +2 1− − u u u u Kết 2: Cho các số thực dương a,b,c Đặt a + b + c = 3u, ab + bc + ca = 3v và abc = w3 với u,v,w là các số thực dương Khi đó u ≥ v ≥ w và s s  v 2  v 2 3  w 3  v 2  v 2 3 −2 1− −2≤ ≤3 +2 1− − (3.6) u u u u u 69 (70) ỨNG DỤNG ĐIỀU KIỆN CÓ NGHIỆM CỦA PHƯƠNG TRÌNH BẬC BA TRONG CHỨNG MINH BẤT ĐẲNG THỨC II Ví dụ minh họa Ví dụ 1.1 Cho các số thực a, b, c thỏa mãn a + b + c = Chứng minh 3  a2 + b2 + c2 ≥ 24 a3 + b3 + c3 − Ta có m = nên (3.4) trở thành  2 q 27 3 |27p| ≤ (−3n) ⇔ −n ≥ p = p − + 9p − ≥ 9p − Hay là − (ab + bc + ca)3 ≥ 9abc − Mặt khác a + b + c = nên ta có a3 + b3 + c3 = 3abc và − (ab + bc + ca) = a2 + b2 + c2 Do vậy, ta có a2 + b2 + c2 3  ≥ 24 a3 + b3 + c3 − Vậy bài toán chứng minh Ví dụ 1.2 Cho các số thực a, b, c có tổng −1 Tìm giá trị nhỏ biểu thức P = 2abc + (ab + bc + ca)2 Ta có m = −1 nên áp dụng (3.4) ta có q q 2 − |9n + 27p − 2| ≤ (1 − 3n) ⇒ p ≥ − n + (1 − 3n)3 27 27 Do đó Đặt t = q q 2 27P ≥ −18n + − (1 − 3n) + 27n = (3n − 1) − (1 − 3n)3 + √ − 3n,t ≥ ta có  27P ≥ 3t4 − 4t3 + = (t − 1)2 3t2 − t + ≥ Do đó P ≥ Đẳng thức xảy a = −1, b = c = và các hoán vị Vậy P = Ví dụ 1.3 Cho các số thực a,b,c thỏa mãn a2 + b2 + c2 = 3.Chứng minh (abc − 2) ≤ (a + b + c) (ab + bc + ca) ≤ (abc + 2) Ta có a2 + b2 + c2 = nên (a + b + c)2 = (ab + bc + ca) + hay n = Từ (3.4) ta suy  m3 − s 9−m 3    ≤ 9mn − 27p ≤ m3 + 70 s m2 − 2 9−m 3   (71) ỨNG DỤNG ĐIỀU KIỆN CÓ NGHIỆM CỦA PHƯƠNG TRÌNH BẬC BA TRONG CHỨNG MINH BẤT ĐẲNG THỨC Ta chứng minh s 3 3  2 − m2 − m2 m + ≤ 27 ⇔ ≤ 27 − m3 2 Khai triển và biến đổi ta  (m − 3)2 m4 + 6m3 + 24m2 + 42m + 63 ≥ Bất đẳng thức này hiển nhiên đúng m4 + 6m3 + 24m2 + 42m + 63 = m2 + 3m + 2 + m2 + 14 > Chứng minh tương tự, ta có s m − − m2 3 ≥ −27 Do ta có bất đẳng thức −54 ≤ 9mn − 27p ≤ 54 ⇔ 3p − ≤ mn ≤ 3p + Hay là (abc − 2) ≤ (a + b + c) (ab + bc + ca) ≤ (abc + 2) Bài toán chứng minh Ví dụ 1.4 Cho các số thực a, b, c thỏa mãn abc = Tìm giá trị lớn biểu thức (a + b + c)3 + (ab + bc + ca)3 P = (a + b + c)2 (ab + bc + ca)2 + 27 Ta có p = nên từ (3.4) ta 9mn − 27 − 2m q ≤ (m2 − 3n)3 Bình phương hai vế và rút gọn ta thu  (mn)2 + 18mn ≥ m3 + n3 + 27 Mặt khác 18mn ≤ (mn)2 + 81 nên ta có   (mn)2 + 81 ≥ m3 + n3 + 27 ⇔ m2 n2 + 27 ≥ m3 + n3 Do đó P = ( Đẳng thức xảy m3 + n3 ≤ 2 m n + 27 abc = , chẳng hạn a = b = c = (ab + bc + ca) (a + b + c) = Vậy max P = Ví dụ 1.5 Cho các số thực a,b,c thỏa mãn ab + bc + ca = Tìm giá trị lớn biểu thức abc(a + b + c)3 + 27 P = (a + b + c + 3abc)2 71 (72) ỨNG DỤNG ĐIỀU KIỆN CÓ NGHIỆM CỦA PHƯƠNG TRÌNH BẬC BA TRONG CHỨNG MINH BẤT ĐẲNG THỨC Ta có n = nên từ (3.4), suy q 27m − 27p − 2m ≤ (m2 − 9)3 Bình phương hai vế và rút gọn ta thu 27p2 + 4m3 p + 108 ≤ 54mp + 9m2 , hay 108p2 + 4m3 p + 108 ≤ (m + 3p)2 Suy 4m3 p + 108 ≤ (m + 3p)2 ⇒ P = m3 p + 27 ≤ (m + 3p)   abc = √ √ Đẳng thức xảy a + b + c = ±2 , chẳng hạn ta chọn a = 0,b = c =   ab + bc + ca = Vậy max P = Ví dụ 1.6 Cho các số thực a,b,c thoả  a2 + b2 + c2 = (ab + bc + ca) Chứng minh rằng: √ (a + b + c)2 + 27 abc + ≥ Ta có  2 m2 − 2n = 5n ⇒ n = m2 Khi đó (3.4) trở thành s r 3 p 2 2 |27p| ≤ ⇒ 27 p ≤ m ⇒ m ≥ 27 m − m 27 Mặt khác p +1 2 ≥ nên p2 ≥ − (p + 1) , p m2 ≥ −27 p + suy hay √ (a + b + c)2 + 27 abc + ≥ Bài toán chứng minh Ví dụ 1.7 Cho các số thực a,b,c thoả a2 + b2 + c2 = ab + bc + ca + Chứng minh rằng: (a + b + c)2 ≤ + (ab + bc + ca)2 + 18abc 72 (73) ỨNG DỤNG ĐIỀU KIỆN CÓ NGHIỆM CỦA PHƯƠNG TRÌNH BẬC BA TRONG CHỨNG MINH BẤT ĐẲNG THỨC Ta có (a + b + c)2 = (ab + bc + ca) + nên m2 = 3n + Khi đó (2) trở thành: 9m m2 − − 27p − 2m3 ≤ ⇒ 27p ≥ m3 − 3m − Đặt T = (a + b + c)2 − (ab + bc + ca)2 − 18abc, ta cần chứng minh T ≤ 3T = 3m2 − 9n2 − 54p 2  ≤ 3m2 − m2 − − m3 − 3m − = −m4 − 2m3 + 5m2 + 6m + 2 = − m2 + m − + 12 ≤ 12 Suy T ≤ Bài toán chứng minh Ví dụ 1.8 (Iran MO 2014, vòng 2) Cho các số thực không âm x,y,z thỏa mãn điều kiện: x2 + y + z = 2(xy + yz + zx) √ x+y+z ≥ 2xyz Chứng minh rằng: Nếu x = y = z = thì bất đẳng thức cần chứng minh hiển nhiên đúng Ta xét x + y + z > Bất đẳng thức cần chứng minh tương đương với  w 3 √ u ≥ 2.w ⇔ ≤ u Áp dụng (3.6) ta cần chứng minh  w 3 u ≤3  v 2 u s 1− +2  v 2 3 u − Mà x2 + y + z = (xy + yz + zx) nên 2 9u = 4.3v ⇒ Do đó  w 3 u ≤3  v 2 u  v 2 u s 1− +2 =  v 2 3 u −2= Bài toán chứng minh Ví dụ 1.9 Cho các số thực không âm a,b,c Chứng minh  a4 + b4 + c4 3abc 2 + ≥ a + b2 + c2 ab + bc + ca a + b + c Ta có a4 + b4 + c4 = 81u4 − 108u2 v + 18v + 12uw3 , và a2 + b2 + c2 = 9u2 − 6v 73 (74) ỨNG DỤNG ĐIỀU KIỆN CÓ NGHIỆM CỦA PHƯƠNG TRÌNH BẬC BA TRONG CHỨNG MINH BẤT ĐẲNG THỨC Nên bất đẳng thức cần chứng minh trở thành  81u4 − 108u2 v + 18v + 12uw3 3w3 2 ≥ 9u − 6v + 3v 3u 27 − 36x + 6x2 + 4y + y + 4x ≥ ⇔ x  v 2  w 3 Trong đó x = và y = u u Theo (3.6), ta có q y ≥ 3x − (1 − x)3 − vàx ≤ nên (3.7)   27 27 − 36 + 10x + y + ≥ − 36 + 10x + 5y V T (3.7) = x x x   q 27 ≥ − 36 + 10x + 3x − (1 − x) − x q 25x2 − 52x + 27 − 10 (1 − x)2 + = x q ≥ (1 − x) (27 − 25x) − 10 (1 − x)3 + q ≥ (1 − x) (1 + 25(1 − x)) − 10 (1 − x)3 + q √ ≥ (1 − x) 10 − x − 10 (1 − x)3 + = Bài toán chứng minh Đẳng thức xảy a = b = c III Bài tập Bài 1.1 Cho các số thực a,b,c không đồng thời thỏa a + b + c = Tìm giá trị lớn biểu thức: 13a2 b2 c2 − 2abc − P = (a2 + b2 + c2 )3 Bài 1.2 Cho các số thực a,b,c có tổng Tìm giá trị nhỏ biểu thức 5 P = a2 + b2 + c2 − 32 (ab + bc + ca) a2 b2 c2 − |abc| Bài 1.3 Cho các số thực a,b,c thoả a2 + b2 + c2 = 2(ab + bc + ca) Tìm giá trị nhỏ biểu thức: P = abc (a + b + c)3 + (abc)4 Bài 1.4 Cho các số thực a,b,c thoả a2 + b2 + c2 = ab + bc + ca + Tìm giá trị nhỏ biểu thức: P = 18 (ab + bc + ca)2 − (ab + bc + ca) (a + b + c − 48) + 9abc Bài 1.5 Cho các số thực dương a,b,c thoả (a + b + c)3 = 32abc Tìm giá trị lớn nhất, giá trị nhỏ biểu thức: a4 + b4 + c4 P = (a + b + c)4 74 (75) BÀI TOÁN TÌM HẰNG SỐ TỐT NHẤT TRONG BẤT ĐẲNG THỨC §2 Bài toán tìm số tốt bất đẳng thức I Lý thuyết Trong chuyên đề nyaf ta giải bài toán: Tìm số k lớn (nhỏ nhất) để BĐT luôn đúng với giả thiết nào đó các biến Để giải dạng toán này, ta thường giải theo hai hướng sau: Hướng 1: • Bước 1: Chọn giá trị đặc biệt các biến đánh giá trực tiếp các biến để điều kiện cần k • Bước 2: Chứng minh bất đẳng thức đã cho đúng với giá trị k ( lớn nhất, nhỏ nhất) vừa tìm Hướng 2: Giả sử ta cần tìm k nhỏ để bất đẳng thức f (a1 ,a2 , ,an ) ≤ k luôn đúng với a1 , a2 , , an ∈ D Ta tìm giá trị lớn M f (a1 ,a2 , ,an ) với a1 , a2 , , an ∈ D Khi đó kmin = M II Ví dụ minh họa Ví dụ 2.1 Tìm số k lớn cho bất đẳng thức sau luôn đúng q q q a + k|b − c|α + b + k|c − a|α + c + k|a − b|α ≤ 2, với α ≥ và a,b,c là các số thực không âm thỏa mãn a + b + c = Cho a = b = 0,c = 1,α = Ta có √ 1+ √ k+ k≤1⇔0≤k≤ Ta chứng minh bất đẳng thức sau đúng q q q α α 4a + |b − c| + 4b + |c − a| + 4c + |a − b|α ≤ đúng với α ≥ và a,b,c ≥ thỏa a + b + c = Không tính tổng quát ta giả sử a ≥ b ≥ c Ta có ≤ |a − b| , |b − c| , |c − a| ≤ 1, α ≥ nên ta có p p p 4a + |b − c| + 4b + |c − a| + 4c + |a − b| √ √ 1p = 4(4a + b − c) + 4b + a − c + 4c + a − b + 4a + b − c + 4b + a − c + 4c + a − b ≤ + + 2 2 (a + b + c) + 8a + 7b + 5c + = ≤ = 4 V T (3.8) ≤ Vậy kmax = 75 (3.8) (76) BÀI TOÁN TÌM HẰNG SỐ TỐT NHẤT TRONG BẤT ĐẲNG THỨC Ví dụ 2.2 Tìm số k nhỏ cho bất đẳng thức a3 + b3 + c3 + kabc ≤  k+3 a (b + c) + b2 (c + a) + c2 (a + b) đúng với a,b,c là độ dài ba cạnh tam giác   3+ 1 n Cho c = 1, a = b = + ta có: k ≥ → Ta chứng minh bất đẳng thức 2 n 1+ n   a3 + b3 + c3 + 9abc ≤ a2 (b + c) + b2 (c + a) + c2 (a + b) Giả sử a = max {a,b,c}, ta có   a3 + b3 + c3 + 9abc − a2 (b + c) + b2 (c + a) + c2 (a + b) = (a − b − c) (a − b) (a − c) + (b − c)2 (b + c − 3a) ≤ Vậy kmax = Ví dụ 2.3 Cho a, b, c > Tìm số k lớn cho bất đẳng thức sau đúng   a + b2 + c2 a b c + + −3≥k −1 b c a ab + bc + ca Cho a = 1, b = c3 6= 1, ta có k≤ (c3 + c2 + 1) (c3 + 3c2 + 2c + 1) → c2 (c4 + 2c3 + 2c2 + c + 1) Ta chứng minh bất đẳng thức a b c + + −3≥ b c a   a2 + b2 + c2 −1 , ab + bc + ca hay a2 + b2 + c2 a b c + + ≥ +2 b c a ab+ bc + ca  a b c ⇔ (ab + bc + ca) + + ≥ (a + b + c)2 b c a Bất đẳng thức cuối dễ dàng chứng minh cách áp dụng bất đẳng thức Cauchy – Schwarz Ví dụ 2.4 Tìm số thực dương k lớn để bất đẳng thức sau đúng với số thực dương x,y,z thỏa mãn điều kiện xyz = : x y z k k √ + + +p ≥ + 3 xy + yz + zx + xy + yz + zx2 76 (77) BÀI TOÁN TÌM HẰNG SỐ TỐT NHẤT TRONG BẤT ĐẲNG THỨC Do bất đẳng thức đã cho đúng với x,y,z > thỏa mãn xyz = nên nó đúng x = n; y = 1; z = với n > Khi đó ta có n 2n k k + +r ≥ +√ ; ∀n > n + 2n 3 2n + n Cho n → +∞ ta √ k 3 2≥ +√ ⇔k≤ 2 √ 3 thỏa mãn yêu cầu bài toán, tức là Ta chứng minh k = r 2x 2y 2z + + + ≥ xy + yz + zx + xy + yz + zx2 Do x, y, z > thỏa mãn xyz = nên tồn a,b,c > thỏa mãn x = c a b ; y = ; z = Khi đó a b c bất đẳng thức cần chứng minh trở thành 2b 2c 2a + + + b+c c+a a+b Ta có r a3 3abc ≥ + b + c3 r 2a 2b 2c 3abc + + + b+c c+a a+b a3 + b3 + c3 (a + b + c) 9abc ≥ + q ab + bc + ca 3 (3abc)2 (a3 + b3 + c3 ) (a + b + c)2 9abc ≥ + ab + bc + ca a + b + c3 + 6abc Ta cần chứng minh (a + b + c)2 9abc −3≥1− 3 ab + bc + ca a + b + c3 + 6abc a+b+c ⇔ ≥ ab + bc + ca a + b3 + c3 + 6abc √ 3 Bất đẳng thức này hiển nhiên đúng theo Schur Vậy bất đẳng thức đã cho đúng k = Đẳng thức xảy x = y = z = x → +∞; y = 1; z = x √ 3 Tóm lại giá trị k tốt cần tìm là k = √ Ví dụ 2.5 (VN TST 2012) Chứng minh C = 10 24 là số lớn cho có 17 số thực dương a1 ,a2 , ,a17 thỏa các điều kiện ( a21 + a22 + · · · + a217 = 24 a31 + · · · + a317 + a1 + · · · + a17 < C thì với ≤ i ≤ j ≤ k ≤ 17 ta có , aj , ak là độ dài ba cạnh tam giác Trước hết ta có bổ đề sau 77 (78) BÀI TOÁN TÌM HẰNG SỐ TỐT NHẤT TRONG BẤT ĐẲNG THỨC Bổ đề Cho số nguyên n ≥ Giả sử n số dương a1 ,a2 , ,an thỏa mãn bất đẳng thức  2 (n − 1) a41 + a42 + + a4n < a21 + a22 + · · · + a2n Hãy chứng minh ba số bất kì , aj , ak (1 ≤ i < j < k ≤ n) là độ dài các cạnh tam giác Chứng minh • Với n = thì ta có 2  a41 + a42 + a43 < a21 + a22 + a23 ⇔ (a1 + a2 + a3 )(a2 + a3 − a1 )(a3 + a1 − a2 )(a1 + a2 − a3 ) > 0, suy a1 ,a2 ,a3 là độ dài cạnh tam giác • Với n > 3, không tính tổng quát ta chứng minh a1 , a2 , a3 là độ dài ba cạnh tam giác Ta có  2 (n − 1) a41 + a42 + · · · + a4n < a21 + a22 + · · · + a2n  2 √ a21 + a22 + a23 2 √ = + a4 + · · · + an  " # 2 2 ) + a + a (a ≤ 2 + 1| + ·{z · · + 1} + a44 + · · · + a4n n−3 # " (a21 + a22 + a23 ) 4 + a4 + · · · + an = (n − 1) Suy  2 a41 + a42 + a43 < a21 + a22 + a23 , đó a1 ,a2 ,a3 là độ dài ba cạnh tam giác Bổ đề chứng minh Trở lại bài toán Đặt xi = √ ,i = 1,17, đó các số dương x1 ,x2 , x17 thỏa 24 ( x21 + x22 + · · · + x217 =  24 x31 + x32 + · · · + x317 + x1 + · · · + x17 < 10 Để chứng minh , aj , ak là độ dài ba cạnh tam giác, ta cần chứng minh xi , xj , xk là độ dài ba cạnh tam giác • Ta chứng minh bài toán đúng với C = 10 Ta tìm số thực dương a thỏa :  a 24x3 + x , ∀x ∈ (0; 1) 10 16x4 + (a − 1)x2 < Vì có bất đẳng thức (1) thì ta suy 16x4i + (a − 1)x2i <  a 24x3i + xi , ∀i = 1,17 10 78 (1) (79) BÀI TOÁN TÌM HẰNG SỐ TỐT NHẤT TRONG BẤT ĐẲNG THỨC Do đó 16 17 X x4i + (a − 1) i=1 17 X 17  a X < 24x3i + xi 10 i=1 x2i i=1 Hay 16 17 X x4i + (a − 1) < a ⇒ 16 i=1 17 X x4i < = i=1 17 X !2 x2i , i=1 đó theo bổ đề ta có đpcm Ta tìm a để (1) đúng? Ta viết lại (1) sau 16x3 + (a − 1)x −  a 24x2 + < 10 (2) Vì x ∈ (0; 1) nên ta chọn a cho vế trái (2) có thừa số x − hay 16 + a − − 5a = ⇒ a = 10 Khi đó (2) trở thành:  16x3 − 24x2 + 9x − < ⇔ (x − 1) 16x2 − 8x + < (3) Rõ ràng (3) đúng với x ∈ (0; 1) Từ đó, ta có đpcm • Ta chứng minh C = 10 là số lớn Giả sử tồn số C > 10 cho với 17 số thực dương x1 , x2 , , x17 thỏa mãn ( x21 + x22 + · · · + x217 =  24 x31 + x32 + · · · + x317 + x1 + · · · + x17 < C thì xi , xj , xk là độ dài ba cạnh tam giác với ≤ i < j < k ≤ 17 Ta xét 17 số thực dương r r a − a2 1 x1 = , x = − a, x3 = a, x4 = · · · = x17 = + ,0<a< 16 16 14 16 Ta có x21 + x22 + · · · + x217 = 1, và x1 − x3 = − a = s −a 2 r < − a = x2 ⇒ x1 > x2 + x3 , 16 hay x1 , x2 , x3 không là độ dài ba cạnh tam giác Ta có S(a) = 24 17 X x3i + 17 X xi i=1 i=1   s s 3  1 a−a  = 24  + − a + a3 + 14 + 16 16 14 r r 1 a − a2 + + − a + a + 14 +  16 16 14 s  s   r r 3 1 1 1 + + → 24  + + 14 + 14 = 10 a → 0+ 16 16 16 16 79 (80) BÀI TOÁN TÌM HẰNG SỐ TỐT NHẤT TRONG BẤT ĐẲNG THỨC  Do tính liên tục S, nên tồn a0 ∈ , r − a0 , a , 16 r 0; 16  cho S(a0 ) < C a0 − a20 + , , 16 14 r a0 − a20 + 16 14 ! thỏa mãn các điều kiện bài toán, (x1 ,x2 ,x3 ) không tạo thành cạnh tam giác Vậy bài toán chứng minh Ví dụ 2.6 Tìm số thực k lớn cho bất đẳng thức sau luôn đúng với a,b là các số thực dương phân biệt thỏa mãn ab = a2 + b2 − p ≥ k 2(a + b) − Bài toán chuyển tìm GTNN biểu thức a2 + b2 − p P = , 2(a + b) − với a,b >p và ab = Đặt t = (a + b) > 2, ta có P = t4 − 16 (t + 2) (t2 + 4) = >8 (t − 2) t > Giả sử tồn k > thỏa bài toán Cho t = + , suy n #   " 2 1 k≤ + → (vô lí) 4+ 2+ n n Vậy kmax = Ví dụ 2.7 Tìm số dương k lớn cho bất đẳng thức    a2 − b2 b2 − c2 c2 − a2 ≤ k (a + b + c)6 đúng với a,b,c ≥ Giả sử a ≥ b ≥ c, ta có       a2 − b2 b2 − c2 c2 − a2 = a2 − b2 b2 − c2 a2 − c2 q  2 2 ≤ a − b a b = (a + b) (a − b)2 ab.ab.ab.ab v ! u u (a − b)2 + 4ab ≤ (a + b) t = Từ đó ta có kmax = (a + b)6 √ ≤ √ (a + b + c)6 25 25 √ 25 80 (81) BÀI TOÁN TÌM HẰNG SỐ TỐT NHẤT TRONG BẤT ĐẲNG THỨC Ví dụ 2.8 (IMO 2006) Tìm số M nhỏ cho với số thực a,b,c ta có  2   ab a2 − b2 + bc b2 − c2 + ca c2 − a2 ≤ M a2 + b2 + c2 Bằng biến đổi đơn giản, ta có    ab a2 − b2 + bc b2 − c2 + ca c2 − a2 = |(b − c) (a − b) (a − c) (a + b + c)| Bài toán trở thành: Tìm M nhỏ để  |(b − c) (a − b) (a − c) (a + b + c)| ≤ M a2 + b2 + c2 Giả sử a = max {a,b,c} , ta có a2 + b2 + c2 2  2 = (a + b + c)2 + (b − c)2 + (a − b) (a − c) h √ i2 ≥ 2 |(a + b + c) (b − c)| + |(a − b) (a − c)| √ ≥ 16 |(b − c) (a − b) (a − c) (a + b + c)| Do đó, ta có √ 2 2 |(b − c) (a − b) (a − c) (a + b + c)| ≤ a + b2 + c2 32 √ √ 3 Có thể chọn b = thì a = + ; c=1− để đẳng thức xảy 2 √ Vậy M = 32 Ví dụ 2.9 (Tổng quát IMO 2004) Với số nguyên dương n ≥ 3, tìm số dương k = k(n) lớn cho n số thực dương t1 , t2 , , tn thỏa mãn   1 + + ··· + <k (t1 + t2 + · · · + tn ) t1 t2 tn thì ti , tj , tk là độ dài ba cạnh tam giác với i, j, k thỏa mãn ≤ i < j < k ≤ n Giả sử tồn ba số chẳng hạn t1 , t2 , t3 thỏa mãn t1 + t2 ≤ t3 Khi đó       1 t1 t2 t1 + t2 1 (t1 + t2 + t3 ) + + =3+ + + + t3 + t1 t2 t3 t2 t1 t3 t1 t2 t1 + t2 4t3 ≥3+2+ + t3 t1 + t2 s  4 t3 t1 + t2 ≥5+5 ≥ 10 t3 t1 + t2 81 (1) (82) BÀI TOÁN TÌM HẰNG SỐ TỐT NHẤT TRONG BẤT ĐẲNG THỨC Xét n ≥ 4, ta có ! n !   n X X1 1 ti = (t1 + t2 + t3 ) + + + t t t t i i=1 i=1 ! n !    n n  X X1 X 1 1 + + ti + ti + (t1 + t2 + t3 ) + t t t t t i i=4 i=4 i i=4 s   1 ≥ 10 + 2(n − 3) (t1 + t2 + t3 ) + + + (n − 3)2 t1 t2 t3  2 √ √ ≥ 10 + 10(n − 3) + (n − 3)2 = n + 10 − √ 10t1 Đẳng thức xảy t1 = t2 , t3 = 2t1 , ti = , ∀i = 4, ,n √ 2 Từ đó suy kmax = n + 10 − Nhận xét a) Nội dung bài toán IMO 2004 sau: Cho số nguyên n ≥ Giả sử t1 ,t2 , ,tn là các số thực dương cho   1 + + ··· + < n2 + (t1 + t2 + · · · + tn ) t1 t2 tn Chứng minh ti , tj , tk là độ dài ba cạnh tam giác với ≤ i < j < k ≤ n b) Ta có thể tổng quát bài toán trên theo hướng khác sau: Cho các số nguyên dương n, k với n ≥ k ≥ và giả sử n số thực dương t1 , t2 , , tn thỏa mãn    2 √ 1 + + ··· + (t1 + t2 + · · · + tn ) < n + 2k − 4k + − k t1 t2 tn Chứng minh số k số ti1 , ti2 , ,tik nhỏ tổng k − số còn lại với ≤ i1 < · · · < ik III Bài tập Bài 2.1 Tìm giá trị lớn k để bất đẳng thức sau đúng với giá trị a, b, c: a4 + b4 + c4 + abc (a + b + c) ≥ k (ab + bc + ca)2 Bài 2.2 Tìm số thực k lớn cho với số thực a, b, c không âm và thỏa mãn điều kiện a + b + c = 1, ta luôn có b c a + + ≥ + 9bc + k(b − c) + 9ca + k(c − a) + 9ab + k(a − b) Bài 2.3 (Turkey National Olympiad Second Round 2013) Tìm số thực M lớn cho với a, b, c dương, ta luôn có  a3 + b3 + c3 − 3abc ≥ M ab2 + bc2 + ca2 − 3abc 82 (83) BÀI TOÁN TÌM HẰNG SỐ TỐT NHẤT TRONG BẤT ĐẲNG THỨC Bài 2.4 Cho các số dương x, y, z Tìm số k lớn cho: x y z x+y+z + + + 3k ≥ (k + 1) · √ xyz y z x Bài 2.5 Cho các số thực không âm a, b, c thỏa mãn ab + bc + ca = a + b + c Tìm số thực k lớn cho ta luôn có bất đẳng thức:   1 + + ≥ k · (a + b + c + 1) (a + b + c) a+b b+c c+a Bài 2.6 (Phạm Kim Hùng) Tìm số thực dương k lớn cho bất đẳng thức sau đúng với số dương a, b, c:   1 (ab + bc + ca) (a + b + c) + + +k· ≥ + k a b c a + b2 + c2 Bài 2.7 (Kiểm tra Trường Hè Lê Qúy Đôn năm 2014) Tìm số k nhỏ cho với số thực dương a, b, c thỏa mãn a + b + c = thì ta luôn có bất đẳng thức:  k a4 + b4 + c4 − ≥ a3 + b3 + c3 + 3abc − Bài 2.8 Với các số không âm a, b, c thỏa mãn a + b + c = ab + bc + ca Tìm số thực k lớn cho a + b + c + k · abc ≥ + k Bài 2.9 Cho a, b, c là các số thực dương Tìm số k lớn cho bất đẳng thức sau đúng: ab + bc + ca k a3 + b3 + c3 +k· + ≥ (a + b) (b + c) (c + a) (a + b + c) Bài 2.10 (Vietnamese Team Selection Test 2009) Tìm tất các số k cho bất đẳng thức sau đúng với số dương a, b, c :      3 a b c +k +k +k ≥ k+ b+c c+a a+b Bài 2.11 (VN TST – 2013) Tìm số nguyên dương k lớn để bất đẳng thức sau đúng với số thực dương a, b, c thỏa mãn điều kiện abc = 1: 1 k k + + + ≥3+ a b c a+b+c+1 Bài 2.12 Xét các số dương a, b, c thỏa mãn abc = Tìm số k lớn cho   1 2 a + b + c + 3k ≥ (k + 1) + + a b c 83 (84) BÀI TOÁN TÌM HẰNG SỐ TỐT NHẤT TRONG BẤT ĐẲNG THỨC Bài 2.13 Xét các số dương a, b, c thỏa mãn a + b + c = Tìm k lớn cho  1 + + − ≥ k a2 + b2 + c2 − a b c Bài 2.14 (Tạp chí Pi - P47 – 7/2017) Tìm số thực k bé cho với ba số thực a, b, c mà abc ≥ 0, ta luôn có  abc + k · (a − b)2 + (b − c)2 + (c − a)2 + ≥ a + b + c Bài 2.15 (Tạp chí Pi - P68 – 9/2017) Tìm số thực k nhỏ cho với số thực dương x, y, z mà {xy,yz,zx} ≥ 1, ta luôn có p (x2 + 1) (y + 1) (z + 1) ≤  x+y+z 2 + k Bài 2.16 (Tạp chí Pi - P61 – 9/2017) Tìm số thực k nhỏ cho với số thực a, b, c là độ dài ba cạnh tam giác, ta luôn có b c ab + bc + ca a + + +k· ≤ + k 2 b+c c+a a+b a +b +c Bài 2.17 (P5 – 2/2017) Tìm √ số dương k nhỏ cho với số thực dương x, y, z thỏa √ √ mãn điều kiện xy + yz + zx = 1, ta luôn có 1 9k + + + ≤ x+y+k y+z+k z+x+k Bài 2.18 (BMO 2012) Tìm số thực k lớn để a,b,c khác thỏa mãn + a 1 + ≤ 3, ta có bất đẳng thức b c       a + b2 + c2 b + a2 + c2 c + a2 + b2 ≥ k Bài 2.19 Cho a1 ,a2 , ,a5 là các số thực bất kì có tổng Tìm số c = c(n) lớn cho bất đẳng thức sau đúng: c n X i=1 |ai | ≤ X |ai − aj | 1≤i<j≤n Bài 2.20 Tìm số M lớn cho với n nguyên dương tồn các số a1 ,a2 , ,an > vàP b1 ,b2 , ,bn > đồng thời thỏa mãn các điều kiện; i) nk=1 bk = 1; 2bk ≥ bk−1 + bk+1 , ∀k = 2, ,n; P ii) a2k ≤ + ki=1 bi , k = 1,n; iii) an = M 84 (85) BÀI TOÁN TÌM HẰNG SỐ TỐT NHẤT TRONG BẤT ĐẲNG THỨC ĐÁP SỐ VÀ HƯỚNG DẪN GIẢI 85 (86) Chương Các bất đẳng thức cổ điển §1 Bất đẳng thức AM-GM Câu 1.1 a) Bất đẳng thức cần chứng minh tương đương với s s 1.1.1 abc + ≤ (1 + a) (1 + b) (1 + c) (1 + a) (1 + b) (1 + c) Đặt : s 1.1.1 abc T = + (1 + a) (1 + b) (1 + c) (1 + a) (1 + b) (1 + c)     1 1 a b c + + + + + T ≤ 1+a 1+b 1+c 1+a 1+b 1+c   a+1 b+1 c+1 T ≤ + + = = 1+a 1+b 1+c s Dấu đẳng thức xảy a = b = c ≥ b) Ta có   a b  c 1+ 1+ 1+ b c a a b c a c b =2+ + + + + +  b c  a  c b a   a+b b+c c+a +1 + +1 + +1 −1 = c a b   1 = (a + b + c) + + −1 a b c 3(a + b + c) 2(a + b + c) √ √ ≥ −1≥ + 3 abc abc  Câu 1.2 Bất đẳng thức cần chứng minh tương đương với r a1 a2 · · · an p + n ≤ n (1 + a1 )(1 + a2 ) · · · (1 + an ) (1 + a1 )(1 + a2 ) · · · (1 + an ) Áp dụng bất đẳng thức AM-GM ta có 1 1X 1 X V T (1) ≤ + = n i=1 + n i=1 + 86 (1) (87) BẤT ĐẲNG THỨC AM-GM Bài toán chứng minh  3 a+b+c Câu 1.3 Ta có abc ≤ = 27 Khi đó      3 1 1 1+ 1+ ≥ 1+ √ 1+ ≥ 64 a b c abc Suy (1 + a) (1 + b) (1 + c) ≥ 64abc Câu 1.4 Bất đẳng thức cần chứng minh tương đương với s r a a · · · a b1 b2 · · · bn n n + n ≤ (a1 + b1 )(a2 + b2 ) · · · (an + bn ) (a1 + b1 )(a2 + b2 ) · · · (an + bn ) (1) Áp dụng bất đẳng thức AM-GM ta có   r a a · · · a a a n n n ≤ + ··· + (a1 + b1 )(a2 + b2 ) · · · (an + bn ) n a1 + b1 an + bn s   b1 bn b1 b2 · · · bn n ≤ + ··· + (a1 + b1 )(a2 + b2 ) · · · (an + bn ) n a1 + b1 an + bn Cộng hai bất đẳng thức trên theo vế ta có đpcm Câu 1.5 Vì αi là các số hữu tỉ dương và n P αi = nên tồn các số nguyên dương N,k1 ,k2 , · · · , kn i=1 ki Áp dụng bất đẳng thức AM-GM cho N số, ta có N a1 + a1 + · · · + a1 + · · · + an + an + · · · + an k1 kn | {z } | {z } n X k1 số kn số ≥ a1n · · · ann = aα1 · · · aαnn αi · = N i=1 cho αi = Bất đẳng thức chứng minh Câu 1.6 Chuẩn hóa a1 + a2 + · · · + an = n, ta cần chứng minh ak1 + ak2 + · · · akn ≥ n (1) Áp dụng bất đẳng thức AM − GM cho k số gồm k − số và aki ta có aki + k − ≥ kai ⇒ n X aki + n(k − 1) ≥ k i=1 n X i=1 = kn ⇒ n X aki ≥ n i=1 Vậy (1) đúng, hay bài toán chứng minh Câu 1.7 Áp dụng bất đẳng thức 1 + ≥ ta có x y x+y 1 + ≥ = a + 3b b + 2c + a (a + 3b) + (b + 2c + a) a + 2b + c 1 + ≥ b + 3c 2a + b + c a + b + 2c 1 + ≥ c + 3a a + 2b + c 2a + b + c Cộng các bất đẳng thức trên theo vế ta có đpcm 87 (88) BẤT ĐẲNG THỨC AM-GM Câu 1.8 Áp dụng bất đẳng thức Cô si cho hai số ta có √ 4 √ √ √ √ √ 1 4 a + b ≥ ab ⇒ a + b ≥ 16 ab và + ≥ √ a b ab Suy √ 4  1  √ √ a+ b + ≥ 16 ab √ = 32 a b ab Dẫn tới 1 √ 4 ≤ √ 32 a+ 4b  1 + a b  Tương tự: 1 ≤  √ √ 32 b+ 4c  1 + b c  1 , √ ≤ √ 32 ( c + a)  1 + c a  Cộng các bất đẳng thức trên ta có 1 ≤ + √ 4 +  √  √ √ 4 √ √ 16 ( c + a) a+ 4b b+ 4c 1  1 + + a b c  Mặt khác, theo giả thiết ta có ab + bc + ca ≤ 3abc nên suy 1 + + ≤ a b c Suy 1 (đpcm) √ 4 +  √ 4 + √ √ ≤ √ √ 4 16 ( c + a) a+ 4b b+ 4c Câu 1.9 Bất đẳng thức cần chứng minh tương đương với 2b 2c 2a 2b 2c 2a + + ≥4− − − b + 2c c + 2a a +  2b  b + 2a c + 2b  a+ 2c  1 1 1 ⇔a + +b + +c + ≥ b + 2c b + 2a c + 2a c + 2b a + 2b a + 2c Áp dụng bất đẳng thức 1 + ≥ ta có x y x+y 1 + ≥ = b + 2c b + 2a 2a + 2b + 2c a+b+c Suy  a 1 + b + 2c b + 2a  ≥ 2a a+b+c Tương tự:  b 1 + c + 2a c + 2b  2b ≥ ,c a+b+c  1 + a + 2b a + 2c  ≥ 2c a+b+c Cộng các bất đẳng thức trên ta có       1 1 1 a + +b + +c + ≥ (đpcm) b + 2c b + 2a c + 2a c + 2b a + 2b a + 2c 88 (89) BẤT ĐẲNG THỨC AM-GM Câu 1.10 Áp dụng AM – GM, ta có  (1 + x + − x + x2 )2 (2 + x2 ) + x3 = (1 + x) − x + x2 ≤ = 4 Tương tự p + y3 ≥ 2 √ ; ≥ + y2 + z2 + z3 Vậy P =√ 1 2 +p +√ ≥ + + 2 2+x 2+y + z2 + x3 + z3 + y3 Áp dụng Cauchy – Swarzt, ta được: P ≥ 18 ≥ x2 + y + z + Dấu ‘=’ xảy x = y = z = Vậy GTNN biểu thức là P = Câu 1.11 Ta có: a a (1 + b2 ) − ab2 ab2 ab2 ab = = a − ≥ a − = a − + b2 + b2 + b2 2b Do đó: b c a + + ≥ a + b + c − (ab + bc + ca) 2 1+b 1+c 1+a Mà: ab + bc + ca ≤ (a + b + c)2 = 3 Nên suy ra: a b c 3 + + ≥ − = + b2 + c2 + a2 2 Câu 1.12 Ta có: a2 a (a + 2b2 ) − 2ab2 2ab2 2ab2 √ √ √ = = a − ≥ a − = a − b a a + 2b2 a + 2b2 a + b2 + b2 2ab Suy ra: √ √  a2 b2 c2 √ + + ≥a+b+c− √ a.b + b.c + c.a a + 2b2 b + 2c2 c + 2a2 Mặt khác: ab + bc + ca ≤ Và √ (a + b + c)2 = p √ √ √ √ √ ab b + bc c + ca a ≤ (ab + bc + ca) (a + b + c) ≤ √ 2 Vậy: a2 b2 c2 3 + + ≥ − = 2 a + 2b b + 2c c + 2a 4 89 (90) BẤT ĐẲNG THỨC AM-GM Câu 1.13 ax + ay ≥ 2axy Đẳng thức xảy x = y √ by + cz ≥ √bcyz.Đẳng thức xảy by = cz cz + bx2 ≥ cbzx Đẳng thức xảy cz = bx2 Bây ta chọn a, b, c cho :    a + b =  a =   2c = ⇔ b=2   √   c = a = bc Suy ra: x2 + y ≥ 2xy Đẳng thức xảy x = y 1 2y + z ≥ 2yz.Đẳng thức xảy 2y = z 2 2 2 z + 2x ≥ 2zx Đẳng thức xảy z = 2x2 2 Cộng vế theo vế ta : 3x2 + 3y + z ≥ (xy + yz + zx) ⇒ 3x2 + 3y + z ≥ 10 (đpcm)  x=y      (  2y = z x=y=1 Đẳng thức xảy : ⇔  z=2  z = 2x      xy + yz + zx = Câu 1.14 Áp dụng bất đẳng thức Cô si cho số thực dương ta có s a a + 2b b + 2c a3 a + 2b b + 2c + + ≥33 = a (a + 2b) (b + 2c) 27 27 (a + 2b) (b + 2c) 27 27 Tương tự: b3 b + 2c c + 2a + + ≥ b, (b + 2c) (c + 2a) 27 27 và c3 c + 2a a + 2b + + ≥ c (c + 2a) (a + 2b) 27 27 Cộng ba bất đẳng thức trên ta có b3 c3 2(a + b + c) a+b+c a3 + + + ≥ (a + 2b) (b + 2c) (b + 2c) (c + 2a) (c + 2a) (a + 2b) Suy a3 b3 c3 a+b+c + + ≥ (a + 2b) (b + 2c) (b + 2c) (c + 2a) (c + 2a) (a + 2b) Đẳng thức xảy a = b = c Câu 1.15 Ta thấy đẳng thức xảy a = b = c = a4 Khi đó = ,b = 1,c + = nên áp dụng bất đẳng thức Cô si cho số ta b (c + 2) s a4 b b c+2 a4 b b c+2 4 + + + ≥ + = a b2 (c + 2) 3 b2 (c + 2) 3 90 (91) BẤT ĐẲNG THỨC AM-GM Tương tự: b4 2b a + c4 2a b + + + ≥ b, + + ≥ c c2 (a + 2) a2 (b + 2) Cộng các bất đẳng thức trên ta có a4 b4 c4 7(a + b + c) + (a + b + c) + + + ≥ b2 (c + 2) c2 (a + 2) a2 (b + 2) Hay a4 b4 c4 5(a + b + c) − + + ≥ b (c + 2) c (a + 2) a (b + 2) √ Mà a + b + c ≥ 3 abc = nên ta có a4 b4 c4 + + ≥ (đpcm) b2 (c + 2) c2 (a + 2) a2 (b + 2) p √ √ Câu 1.16 Áp dụng bất đẳng thức x + y ≤ (x + y), ta có : r r r √ ! √ √ ! √ √ ! √ a+b b+c c+a a b b c c b √ +√ √ +√ √ +√ +√ +√ + + ≥√ c a b c c a a a a 2 √    √   √  a 1 b 1 c 1 √ +√ +√ √ +√ +√ √ +√ =√ c a c a 2 b b 1 + ≥ , ta có : x y x+y √ √ √  √   √   √  a 1 b c 2c 2a 2b 1 √ √ +√ +√ √ +√ +√ √ +√ √ +√ √ ≥√ √ +√ c a c a a+ c 2 b b b+ c a+ b Áp dụng bất đẳng thức p √ √ Áp dụng bất đẳng thức x + y ≤ (x + y), ta có : √ √ √ √ √ √ 2b 2a 2c 2b 2c 2a √ √ +√ √ ≥p +p +p √ +√ a+ c b+ c a+ b (b + c) (a + c) (a + b) ! r r r c b a =2 + + a+b a+c b+c Câu 1.17 Áp dụng bất đẳng thức AM-GM ta có s a b+2 a4 b + 2 + ≥2 = a2 b+2 b+2 Tương tự: b4 c+2 c4 a+2 + ≥ b2 , + ≥ c2 c+2 a+2 Cộng ba bất đẳng thức trên ta có  a4 b4 c4 a+b+c+6 2 + + + ≥ a + b2 + c2 = b+2 c+2 a+2 Suy a4 b4 c4 12 − (a + b + c) + + ≥ b+2 c+2 a+2 91 (92) BẤT ĐẲNG THỨC AM-GM Mặt khác: a+b+c≤ p (a2 + b2 + c2 ) = nên suy b4 c4 12 − a4 + + ≥ = (đpcm) b+2 c+2 a+2 Đẳng thức xảy a = b = c = Câu 1.18 Bất đẳng thức cần chứng minh tương đương với     4 +1 +1 + ≥ (a + b + c)2 2 3−a 3−b 3−c Từ đề bài, ta suy a2 ,b2 ,c2 < Áp dụng bất đẳng thức Cô si ta có: r  4 + − a2 ≥ (3 − a2 ) = 4, 3−a − a2 suy + ≥ a2 + 2 3−a Tương tự: 4 + ≥ b + 2, + ≥ c2 + 2 3−b 3−c Do đó  +1 − a2  +1 − b2  +1 − c2  ≥ a2 +  b2 +   c2 + Mặt khác: (a + b + c)2 =  a+b √ √ + 1.c 2 " (a + b)2 ≤3 +1 #  c2 + Ta chứng minh " #   (a + b)2 + ≤ a2 + b2 + Khai triển và rút gọn, bất đẳng thức (*) trở thành 2a2 b2 + a2 + b2 + ≥ 6ab Bất đẳng thức này hiển nhiên đúng vì:  a2 b2 + ≥ 4ab, a2 + b2 ≥ 2ab Vậy bài toán chứng minh p Câu 1.19 Ta có: b + c ≤ 2(b2 + c2 ) Suy a2 a2 + b2 + c2 √ − √ b2 + c2 ≥p b+c 2(b2 + c2 ) Tương tự: b2 a2 + b2 + c2 √ c2 a2 + b2 + c2 √ ≥p − √ c2 + a2 ; ≥p − √ a2 + b2 c+a a+b 2 2(c2 + a2 ) 2(a2 + b2 ) 92 (∗) (93) BẤT ĐẲNG THỨC AM-GM Suy a2 + b + c2 √ VT ≥ Ta có:    √ √ 1 1 √ 2 2 2 √ a +b + b +c + c +a +√ +√ −√ a2 + b2 b2 + c2 c2 + a2 1 √ √ √ +√ +√ ≥√ 2 2 2 2 a +b b +c c +a a + b + b + c2 + c2 + a2 và a2 + b2 + c2 ≥ Suy  √ √ √ 2 2 2 √ a +b + b +c + c +a VT ≥ 2 Đặt t= Suy 2 √ √ √ a + b2 + b2 + c2 + c2 + a2 √ a2 + b2 + √ b2 + c2 √ √ √ √ √ √ √  2 + c +a ≥ ab + bc + ca ≥ abc √ t ≥ abc nên từ giả thiết ta suy 54 √ √ √ − abc t = a2 + b2 + b2 + c2 + c2 + a2 = √ ≥√ − t 2 108 √ ⇔ t3 + 108t − 378 ≥   √ √ √  ⇔ t − t + 2t + 126 ≥ ⇔ t ≥ √ 3 Suy V T ≥ √ = 2  ( a = b = c a=b=c √ Đẳng thức xảy và ⇔ a = b = c = − a3 ⇔ 3 2a = √ a3 + 6a − = Câu 1.20 Nhận thấy đẳng thức xảy a = b = c = và = + nên ta có đánh giá   a2 + 2b2 + = a2 + b2 + b2 + + ≥ 2ab + 2b + Do đó: a2 1 ≤ + 2b + ab + b + Suy ra: 1 1 + + ≤ 2 2 a + 2b + b + 2c + c + 2a +  1 + + ab + b + bc + c + ca + a +  Vậy ta cần chứng minh: 1 + + ≤ ab + b + bc + c + ca + a + Bất đẳng thức này hiển nhiên đúng vì đó là đẳng thức Để chứng minh ta thay c = vào vế ab trái và biến đổi ta có đpcm Câu 1.21 Ta có X √ a3 + b ≤ X 1 X p √ =√ 2 a3 b s    1 X 3 · ≤ √ + =√ a b a b 2 93 (94) BẤT ĐẲNG THỨC AM-GM Câu 1.22 Ta có 2ab + a2 + b2 + c2 + ab + bc + ca (a + b)2 + (c + a) (c + b) ab + ≥ = 2 Suy VT ≥   (a + c)(b + c) (a + b)(a + c) (b + a)(b + c) 3+ ≥ + + (a + b)2 (b + c)2 (c + a)2 Câu 1.23 Áp dụng bất đẳng thức AM-GM ta có s a+b+c=a+ b+c b+c (b + c)2 + ≥3 a , 2 suy s 2a b+c 2 ≥ 3a a+b+c Chứng minh tương tự, ta có s s 3 2 2b 3b 2c 3c 3 ≥ và ≥ c+a a+b+c a+b a+b+c Cộng ba bất đẳng thức trên theo vế ta có đpcm Câu 1.24 Bài toán này có thể chứng minh cách sử dụng đánh giá sau: r 3 a2 + b2 a + b ≤ a+b a2 + b2 2ab Chú ý rằng: =a+b− a+b a+b Như ta phải chứng minh:   bc ca ab + + + a + b + c ≥ a+b b+c c+a Áp dụng bất đẳng thức AM-GM với abc = 1,ta có ngay: 2ab a+b 2bc b+c 2ca c+a + + + + + ≥ a+b b+c c+a Vậy ta có điều phải chứng minh.Đẳng thức xảy a = b = c = Câu 1.25 Ta có 13a2 b2 c2 − 2abc − 27a2 b2 c2 − (abc + 2)2 27a2 b2 c2 = ≤ (a2 + b2 + c2 )3 2(a2 + b2 + c2 )3 2(a2 + b2 + c2 )3 Ta chứng minh: 27a2 b2 c2 a2 b2 c2 ≤ ⇔ 3 ≤ 2 2 2 54 2(a + b + c ) (a + b + c ) Vì a + b + c = nên ba số a,b,c có hai số cùng dấu, ta giả sử hai số đó là a,b Khi đó ab ≥ nên " #    (a + b) (a + b) 3 a2 + b2 + c2 = a2 + b2 + (a + b)2 = a2 + b2 + + 2  27 27 ≥ a + b2 (a + b)2 (a + b)2 ≥ 2ab.4ab.c2 = 54a2 b2 c2 4 Suy a2 b2 c2 (đpcm) ≤ 2 54 (a + b + c ) 94 (95) BẤT ĐẲNG THỨC AM-GM Câu 1.26 Không√mất tính tổng quát, ta giả sử c = {a,b,c} Nếu a ≥ b suy (a − b) (b − c) (c − a) ≤ nên bất đẳng thức cần chứng minh luôn đúng Nếu a ≤ b, ta có: √ √ (a − b) (b − c) (c − a) = (b − a) (b − c) (a − c) √ √ q ≤ (b − a) ba = 3 (a − b)2 2ab.2ab v !3 u √ u (a − b)2 + 2ab + 2ab t ≤3 3 q q = (a + b)3 ≤ (a + b + c)3 (đpcm)  ( c = c=0  √  ⇔ Đẳng thức xảy và a = ± b (a − b)2 = 2ab Câu 1.27 Trước hết ta có bất đẳng thức √ 2 1 + ≥p x y x2 + y Không tính tổng quát, ta giả sử a > b > c Khi đó P = ≥ = ≥ = 2 + + +√ a−b b−c a−c ab + bc + ca + +√ a−b+b−c a−c ab + bc + ca 10 10 + √ a − c ab + bc + ca √ 20 q (a − c)2 + 4(ab + bc + ca) √ √ 20 20 q =p (1 − b) (1 + 3b) (a + c) + 4b (a + c) Mặt khác  2 − 3b + + 3b (1 − b) (1 + 3b) = (3 − 3b) (1 + 3b) ≤ = 3   b = √ √ Do đó P ≥ 10 Đẳng thức xảy Vậy P = 10  a = ,c = Câu 1.28 Ta có X a+ b 2   X X Xa X 1 ≥ a+ b+ = ab + + + b c c ab P P P Pa P = a và ab + ≥ a ta có đpcm ab c Cách 2: Ta có r r r 2 b b a a · abc 2 =3 = 3a a + + ≥3 a · · =3 b c b c bc bc Áp dụng 95 (96) BẤT ĐẲNG THỨC AM-GM Tương tự b2 + c + ≥ 3b, c a c2 + a + ≥ 3c a b và Kết hợp với r a b c b c a + + ≥3 · · = b c a b c a ta có đpcm ≥ + a nên b+1 2       2 5 a + 2b + b + 2a + ≥ + a + b a+1 b+1 2 2 Câu 1.29 Ta có b + 2a + và  + a 2  + b 2  = 25 15 + (a + b) + ab ≥ 16 4 nên ta có đpcm Câu 1.30 Ta có p 2a + b + c = a + b + a + c ≥ (a + b) (a + c) Suy 1 ≤ (a + b) (a + c) (2a + b + c) Do đó a+b+c VT ≤ (a + b) (b + c) (c + a) Mặt khác ta có: (a + b) (b + c) (c + a) ≥ 8(a + b + c)(ab + bc + ca), và (ab + bc + ca)2 ≥ 3abc (a + b + c) Từ giả thiết, ta suy ab + bc + ca = abc (a + b + c) nên ab + bc + ca ≥ Suy a+b+c (a + b + c)(ab + bc + ca) = ≤ = (a + b)(b + c)(c + a) (a + b)(b + c)(c + a) ab + bc + ca 8 Từ đó, suy đpcm Câu 1.31 Áp dụng bất đẳng thức Bunhia ta có: 2 √ 2  √  p a2 + b2 = a a · a + b b · b ≤ a3 + b3 (a + b) ≤ a3 + b3 (a2 + b2 ) q q √  3 3 2 2 ⇒ (a + b ) ≤ a + b ⇒ a + b ≤ (a3 + b3 )2 c3 c3 a3 + b3 + c3 √ √ q √ q √ ⇒ ≥ = − a3 + b3 3 a +b (a3 + b3 )2 (a3 + b3 )2 96 (1) (97) BẤT ĐẲNG THỨC AM-GM Ta có hai bất đẳng thức tương tự b3 a3 + b3 + c3 √ 3 √ p c + a3 , ≥ −√ 3 2 3 c +a (c + a ) 3 3 a a +b +c √ 3 √ √ p b + c3 ≥ − 3 2 b +c (b3 + c3 )2 (2) (3) Cộng ba bất đẳng thức (1), (2) và (3), ta    1 1  P ≥ √ a3 + b3 + c3  q +q +q 2 3 (a3 + b3 ) (a3 + b3 ) (a3 + b3 )  √ √ √ 3 + b3 + b3 + c3 + c3 + a3 −√ a √ √ √ Đặt x = a3 + b3 , y = b3 + c3 , z = c3 + a3 Suy ra:      1 1 3 P ≥ √ x +y +z − (x + y + z) + + x2 y z 2 Mặt khác áp dụng bất đẳng thức AM-GM ta có được:    1 2 x +y +z + + ≥9 x2 y z 1 ⇒ 2+ 2+ ≥ x y z x + y2 + z2    1 x3 + y + z 3 3 + + ≥ ⇒ x +y +z x2 y z x2 + y + z   x + y3 + z3 ⇒P ≥ √ − (x + y + z) x2 + y + z Áp dụng bất đẳng thức Cauchy-Schwarz ta có:   2 (x + y + z)2 ≤ x2 + y + z ⇒ (x + y + z)2 x2 + y + z ≤ x2 + y + z mà  √ √ 2 √ √ √ √ = x x x + y y y + z z z ≤ (x + y + z) x3 + y + z   ⇒ (x + y + z)2 x2 + y + z ≤ (x + y + z) x3 + y + z x3 + y + z ⇒ ≥ (x + y + z) 2 x +y +z x2 + y + z 2 Do đó P ≥ Vì t= √ a3 + √ b3 √  √ √ √ 3 3 3 3 3 (x + y + z) = a +b + b +c + c +a + √ b3 + c3 √ √ √ √ √ √ √  3 3 3 + c +a ≥ ab + bc + ca ≥ abc t 54 Nên từ giả thiết ta suy ra: Suy abc ≤ 3≤t+ t ⇔ t3 + 54t − 162 ≥ 54 97 (∗) (98) BẤT ĐẲNG THỨC AM-GM Vì hàm số f (t) = t3 + 54t − 162 là hàm đồng biến và f (2)f (3) < nên f (t) có nghiệm x = m ∈ (2; 3) ( a=b=c √ √ ⇔ a = b = c = α Suy (∗) ⇔ t ≥ m ⇒ P ≥ 32.m Đẳng thức xảy ⇔ a3 + 2a − = √ Trong đó α là√nghiệm phương trình X + 3 2X − = và α > Vậy P = 32.m (đpcm) Câu 1.32 Ta chứng minh a + bc ≤ (1) b2 + c2 < nên tương đương a2 ≤ (2 − bc)2 Do Do bc ≤ a2 ≤ − (b2 + c2 ) và (2 − bc)2 − (3 − b2 − c2 ) = (bc − 1)2 + (b − c)2 ≥ nên (1) đúng p Ta chứng minh (4 − a2 )(4 − c2 ) ≥ ac + 2b (2)  3 a + b2 + c2 Ta có abc ≤ ≤ nên (4 − a2 )(4 − c2 ) − (ac + 2b)2 = 16 − 4(a2 + b2 + c2 + abc) ≥ Suy (2) đúng Ta chứng minh a2 + b2 + c2 ≥ ab + b2 c + c2 a (3) Ta có a2 + (ab + c2 )2 ≥ a(ab + c2 ) = a2 b + c2 a (4) và  p abc2 1 2 2 2 (4 − a )b + (4 − c )c ≥ bc (4 − a )(4 − b ) ≥ bc + 2 (5) Cộng (4) và (5) theo vế suy (3) chứng minh Quay trở lại bài toán Từ (1) ta có a2 b + ab2 c ≤ 2ab, b2 c + abc2 ≤ 2bc, c2 a + a2 bc ≤ 2ca Suy 2(ab + bc + ca) ≥ a2 b + b2 c + c2 a + abc(a + b + c) ⇔(a + b + c)2 ≥ a2 + b2 + c2 + abc(a + b + c) + a2 b + b2 c + c2 a Từ (3) và (6) ta có (a + b + c)2 ≥ abc(a + b + c) + 2(a2 b + b2 c + c2 a) ⇔(a + b + c)(a + b + c − abc) ≥ 2(a2 b + b2 c + c2 a) Câu 1.33 Không tính tổng quát, giả sử a = max{a; b; c} Khi đó b + c + ≤ c + a + và b + c + ≤ a + b + b c c b ≤ ; ≤ Dấu xảy a = b = c Suy c+a+1 b+c+1 a+b+1 b+c+1 Mặt khác theo BĐT AM-GM ta có  3 1−b+1−c+b+c+1 (1 − b) (1 − c) (b + c + 1) ≤ =1 Suy (1 − a) (1 − b) (1 − c) ≤ Do đó P ≤ 1−a b+c+1 a b c 1−a + + + =1 b+c+1 b+c+1 b+c+1 b+c+1 Dấu = xảy a = b = c = 98 (6) (99) BẤT ĐẲNG THỨC AM-GM Câu 1.34 √ √ √ • Cách 1: Theo BĐT AM-GM ta có: a + b ≥ ab; b + c ≥ 2 bc; c + a ≥ 2 ca 3a + 2b + c 3a + 2b + c 1 √ Ta có: P = ≤ √ + + √ = (a + b)(b + c)(c + a) ab ac bc ab.2 bc.2 ca 5 Lại có: 3bc + 4ac + 5ab ≤ 6abc ⇔ + + ≤ Đặt S = + + a b c a b c Ta có     1 24 30 40 16 25 24 30 40 1 S = 2+ + + + + =9 + + + + 16 + + a b c ab ac bc a c b c ab ac bc   18 32 24 30 40 24 48 72 ⇒S ≥ + + + + = + + = 24 + + ac bc ab ac bc ab ac bc ab ac bc S2 + + ≤ ≤ ab ac bc 24 3 ⇒P ≤ = 16 Dấu xảy a = b = c = Vậy giá trị lớn P a = b = c = 5 • Cách 2: 3bc + 4ac + 5ab ≤ 6abc ⇔ + + ≤ Đặt S = + + a b c a b c Ta có:     16 25 24 30 40 1 1 24 30 40 S = 2+ + + + + =9 + + + + + 16 + 2 a b c ab ac bc a2 c2 b c ab ac bc   18 32 24 30 40 24 48 72 ⇒ S2 ≥ + + + + = + + = 24 + + ac bc ab ac bc ab ac bc ab ac bc 1 4 ≥ ≥ ≥ 2; 2; ab a + c)2  (a + b) (b + c) ac  bc   12 + ⇒ S ≥ 24 = 96 + + + (a + c)2 (c+ b)2 (a + b) (c + b)2  (a + b)  (a + c) 1 1 ⇒ S ≥ 96 + +2 + (b + c) (a + c) (b + c)2  (a + b)  + ⇒ S ≥ 96  (a + b)(b + c) (a + c)(b + c)  3a + 2b + c S ≥ 192 + = 192 (a + b)(b + c) (a + c)(b + c) (a + b)(b + c)(c + a) S2 36 P ≤ = = 192 192 16 Dấu xảy a = b = c = Vậy giá trị lớn P a = b = c = p Câu 1.35 Biến đổi tương đương ta chứng minh được: a+b+c ≤ (a2 + b2 + c2 ), ∀a, b,c > Dấu ” = ” xảy và a = b = c Ta có v ! u u 1 V T ≤ t3 p +p +√ z + 2x3 + x3 + 2y + y + 2z + v s u   u 1 t ≤ 3 + + x3 + 2y + y + 2z + z + 2x3 + ⇒ 99 (100) BẤT ĐẲNG THỨC AM-GM Theo bất đẳng thức TTBC-TBN ta có: x3 + 2y + = (x3 + y + 1) + (y + + 1) + ≥ 3xy + 3y + = 3(xy + y + 1) Tương tự: y + 2z + ≥ 3(yz + z + 1) z + 2x3 + ≥ 3(xz + x + 1) Do đó   1 + + VT ≤9 xy + y + yz + z + zx + x +   xy =9 + + xy + y + xy(yz + z + 1) zx + x + = (do xyz = 1) √ Hay V T ≤ Dấu ” = ” xảy và x = y = z = √       3+1 1 1 1 y2 z z z Câu 1.36 Ta có P + = + + + x + + + + + + 3x 3x 3x y2 z3 9 Áp dụng Bất đẳng thức AM-GM, s  ta có: 3 x 1 4 = (1) + + + x >4 3x 3x 3x 3x 3 √ y + > (2) y 3 √ z z z + + + > (3) z3 9 9 √ √ √ √ 4 3+9 3+1 > + + hay P > Từ (1), (2) và (3) ta có P + 3 9 Dấu đẳng thức xảy và khi:  x3    =   3x     x=1   1 =y  √ y2 ⇔ y=   √   z   z= =    z3   x3 + y + z = 2√3 + √ √ √ 3+9 Vậy P = (x,y,z) = (1, 3, 3) Câu 1.37 Ta có: a6 b6 + b6 c6 + c6 a6 = a5 b5 ab + b5 c5 bc + c5 a5 ca a5 + b5 + + + b5 + c5 + + + c5 + a5 + + + ≤ a5 b5 + b5 c5 + c5 a5 5 5 5 5 5 5 5 = (a b (a + b + 3) + b c (b + c + 3) + c a (c + a + 3)) Đặt x = a5 ,y = b5 ,z = c5 Ta có: x + y + z = 3;với x,y,z > Cần chứng minh: xy(x + y + 3) + yz(y + z + 3) + zx(z + x + 3) ≤ 15 hay xy(x + y) + yz(y + z) + zx(z + x) + 3(xy + yz + zx) ≤ 15 ⇔ (xy + yz + zx)(x + y + z + 3) − 3xyz ≤ 15 ⇔ 2(xy + yz + zx) − xyz ≤ Mặt khác; xyz ≥ (3 − 2x)(3 − 2y)(3 − 2z) ⇒ 9xyz ≥ 12(xy + yz + zx) − 27 ⇒ xyz ≥ (xy + yz + zx) − 3 Ta có: VT(1) ≤ 2(xy + yz + zx) − (xy + yz + zx) + = (xy + yz + zx) + ≤ 3 Vậy bài toán chứng minh 100 (1) (101) BẤT ĐẲNG THỨC AM-GM   1 ; ; không thỏa mãn 2 Câu 1.38 Với k = thì   4 Với k = thì ; ; không thỏa mãn 5 Ta chứng minh với k = thì bất đẳng thức đúng hay x3 y z (x3 + y + z ) ≤ Không tính tổng quá ta có thể giả sử z nhỏ suy z ≤ Ta có x3 + y = (x + y)3 − 3xy(x + y) = (3 − z)3 − 3xy(x + y) Khi đó bất đẳng thức cần chứng minh tương đương (3 − z)3 + z ≤ x3 y z + 3xy(x + y) ⇔ 3z − 9z + ≤ x3 y z + x2 y + xy s x3 y 3 Ta có 3 + x2 y + xy ≥ 3 3 = , cần chứng minh xy z xy z z 3z − 9z + ≤ ⇔ 3(z − 1)2 ≤ z Câu 1.39 Theo bất đẳng thức AM-GM, ta có: r r p a+c a+c a + b + 2(a + c) = (a + b) + + 2 v ! u r r u a+c (a + b)(a + c) t ≥ (a + b) =3 2 Suy  3 ≤ p 27(a + b)(a + c) a + b + 2(a + c) Tương tự với hai biểu thức còn lại Do đó: X cyc  a+b+ p 2(a + c) 3 ≤ X sym 4(a + b + c) = 27(a + b)(a + c) 27(a + b)(b + c)(c + a) Hơn nữa, ta thấy với a, b, c dương: 9(a + b)(b + c)(c + a) − 8(a + b + c)(ab + bc + ca) = X a(b − c)2 ≥ sym ⇒ (a + b)(b + c)(c + a) ≥ (a + b + c)(ab + bc + ca) Do đó: X cyc  a+b+ p 3 ≤ 6(ab + bc + ca) 2(a + c) (1) Mặt khác, ta có: (ab + ca + ca)2 ≥ 3abc(a + b + c)nên theo giả thiết: 16(a + b + c) ≥ 1 ab + bc + ca 3(a + b + c) + + = ≥ ⇒ ab + bc + ca ≥ a b c abc ab + bc + ca 16 Từ (1) và (2), suy ra: 1 p p p + + ≤ (a + b + 2(a + c)) (b + c + 2(b + a)) (c + a + 2(c + b)) 101 (2) (102) BẤT ĐẲNG THỨC AM-GM Ta có đpcm  a,b,c >    ⇒ Đẳng thức xảy dấu tất các bất đẳng thức trên xảy hay: a = b = c  1  16(a + b + c) = + + a b c a=b=c= Câu 1.40 Gọi P là vế trái bất đẳng thức cần chứng minh Không tính tổng quát, ta giả sử a + b + c = Áp dụng bất đẳng thức AM-GM ta có : √ a2   a 2a a +√ + a a2 + 8bc ≥ 3a ⇔ √ + a a2 + 8bc ≥ 3a 2 + 8bc a + 8bc a + 8bc Tương tự : √ b2   2b 2c + b b2 + 8ca ≥ 3b ; √ + c c2 + 8ab ≥ 3c + 8ca c + 8ab Cộng ba bất đẳng thức trên lại với ta : 2P + a3 + b3 + c3 + 24abc ≥ Mặt khác ta lại có : = (a + b + c)3 = a3 + b3 + c3 + (a + b) (b + c) (c + a) ≥ a3 + b3 + c3 + 24abc Suy :  2P ≥ − a3 + b3 + c3 + 24abc ≥ − = ⇒ P ≥ đpcm Câu 1.41 Áp dụng bất đẳng thức AM-GM ta có a3 b + 2b = a3 b + b + b ≥ 3ab, b3 c + 2c ≥ 3bc, c3 a + 2a ≥ 3ca Suy a3 b + b3 c + c3 a + 2(a + b + c) ≥ 3(ab + bc + ca), hay a3 b + b3 c + c3 a + ≥ 3(ab + bc + ca) Mặt khác ab + bc + ca ≤ (a + b + c)2 = 3, nên ta có a3 b + b3 c + c3 a + ≥ 4(ab + bc + ca) Câu 1.42 Áp dụng bất đẳng thức AM-GM ta có r     a4 a a b a 2a b a a= ≤ + + + = + + abcd b b c d b c d Tương tự: b≤  2b c b + + c d a  ,c≤  2c d c + + d a b 102  ,d≤  2d a d + + a b c  (103) BẤT ĐẲNG THỨC AM-GM Suy     a b c d b c d a a+b+c+d≤ + + + + + + + b c d a a b c d   b c d a ≤ (a + b + c + d) + + + + 4 a b c d Nên a+b+c+d≤ b c d a + + + a b c d Câu 1.43 Ta {a,b,c} Q giả sử a = Ta có V T = (a + b) (a − ab + b2 ) Mà (a + b) (b + c) (c + a) = (a + b + c) (ab + bc + ca) − 3abc ≤ (a + b + c) (ab + bc + ca) = (ab + bc + ca) Q Suy V T ≤ (ab + bc + ca) (a − ab + b2 ) Do a = {a,b,c} nên a2 − ab + b2 ≤ b2 và c2 − ca + a2 ≤ c2 Tiếp theo ta chứng minh  b2 c2 (ab + bc + ca) b2 − bc + c2 ≤ (∗) Áp dụng bất đẳng thức AM-GM ta có 2 b c (ab + bc + ca) b − bc + c  bc + bc + ab + bc + ca + b2 − bc + c2 ≤ 4 = b2 + c2 + 2bc + ab + ca  4 Lại có b2 + c2 + 2bc + ca + ab = (b + c)2 + a (b + c) = (2 − a)2 + a (2 − a) = −2a + ≤ Từ đó, ta có đpcm Câu 1.44 Xét x2 − yz, y − zx, z − xy là các số thực dương Áp dụng bất đẳng thức AM-GM ta có  3 x + y + z − (xy + yz + zx) P ≤ Mặt khác x2 + y + z + 2(xy + yz + zx) = (x + y + z)2 ≥ 0, suy xy + yz + zx ≥ − Do đó P ≤ Xét x2 − yz > > y − zx, z − xy,khi đó áp dụng bất đẳng thức AM-GM ta có 2  P = (x − yz)(xz − y )(xy − z ) ≤ 103 xy + xz + x2 − yz − y − z 3 (104) BẤT ĐẲNG THỨC AM-GM Mặt khác x2 + xy + xz − yz − y − z = x2 + xy + xz + yz − (y + z)2 ≤ x2 + xy + xz + yz = (x2 + y + z ) − (x − 2y)2 − (x − 2z)2 3 ≤ (x2 + y + z ) = 2 √ √ 2 ,y=− , z = Suy P ≤ Đẳng thức xảy chẳng hạn x = 2 Vậy GTLN P bằng: Câu 1.45 Ta có x2 y x2 y (x + y) ≥ 4xy ⇒ = ≤ xy(x + y) 1−z x+y Do đó, V T ≤ (xy(x + y) + yz(y + z) + zx(z + x)) + 3xyz Tiếp theo ta chứng minh 2(x + y + z)3 xy(x + y) + yz(y + z) + zx(z + x) + 12xyz ≤ = 3 (1) Ta có (1) tương đương với 2(x3 + y + z ) + (xy(x + y) + yz(y + z) + zx(z + x)) ≥ 24xyz Vì √ √ √ 2(x3 + y + z ) ≥ 2xy xy + 2yz yz + 2zx zx, nên √ √ √ √ V T (2) ≥ 8(xy xy + yz zy + zx zx) ≥ 24 xyz ≥ 24xyz √ (Do xyz ≤ nên xyz ≥ xyz.) Câu 1.46 Áp dụng bất đẳng thức AM – GM, ta có s (b + 2c)a2 a2 (b + 2c)a2 2a2 a2 + ≥2 = b + 2c b + 2c Tương tự b2 (c + 2a)b2 2b2 c2 (a + 2b)c2 2c2 + ≥ , + ≥ c + 2a a + 2b Suy ra: a2 b2 c2 F = + + b + 2c c + 2a a + 2b  1  2 ≥ a + b2 + c2 − a2 (b + 2c) + b2 (c + 2a) + c2 (a + 2b) (1) Lại áp dụng AM – GM, ta có a2 c + b2 a + c2 b ≤ a3 + a3 + c3 b3 + b3 + a3 c3 + c3 + b3 + + = a3 + b3 + c3 (2) 3 Từ (1) và (2) suy ra:  2 a + b2 + c2 −  2 ≥ a + b2 + c2 − F ≥ (a + b + c) (a2 + b2 + c2 ) p a + b2 + c2 (a2 + b2 + c2 ) 104 (2) (105) BẤT ĐẲNG THỨC AM-GM Đặt t = p (a2 + b2 + c2 ) , từ giả thiết ta có:   2 25 a2 + b2 + c2 − 48 = a4 + b4 + c4 ≥ a2 + b2 + c2  2 ⇒ a2 + b2 + c2 − 25 a2 + b2 + c2 + 48 ≤ 16 ⇒ ≤ a2 + b2 + c2 ≤ Do đó với t ∈ [3; 4] F ≥ t2 − t3 = f (t) 27 (3) Mà f (t) = f (3) = (4) Từ (3) và (4) suy F ≥ t∈[3;4] Vậy F = xảy a = b = c = Câu 1.47 Bất đẳng thức cần chứng minh tương đương với X √ √  p 5a2 + 4bc − bc > 3(a2 + b2 + c2 )   X a p √ ⇔ √  ≥ 3(a2 + b2 + c2 ) 5a2 + 4bc + bc (1) Áp dụng đt AM-GM ta có p 8a2 + 3b2 + 3c2 + 4bc 2 3(a + b + c )(5a bc) ≤ p √ 2 2 2 3(a + b + c ).2 bc ≤ a + b + c + 3bc Do đó p 3(a2 + b2 c2 ) + √ 5a2 √  10a2 + 5(b + c)2 + 4bc + bc ≤ ≤ 5(a2 + b2 + c2 ) Suy V T (1) ≥ X a2 = 2 5(a + b + c ) Câu 1.48 Gọi P là vết trái Ta có a2 b (1 + a + b + c) a2 bc 4a2 b = = a2 b + 1+a+b 1+a+b 1+a+b Tương tự 4b2 c ab2 c 4c2 a abc2 = b2 c + , = c2 a + 1+b+c 1+b+c 1+c+a 1+c+a Do đó 4P = X X X a2 b a = a2 b + abc 1+a+b 1+a+b Mặt khác X X a (1 + a + b + c) X a ac = =3+ 1+a+b 1+a+b 1+a+b Lại có ac 3ac = ≤ 1+a+b (2a + b) + (2b + c) Nên X  2ac ac + 2a + b 2b + c ac ≤ (a + b + c) = 1+a+b Suy 4P ≤ a2 b + b2 c + c2 a + abc ≤ ⇒ P ≤ 105  (106) BẤT ĐẲNG THỨC AM-GM Câu 1.49 Ký hiệu P là vế trái bất đẳng thức cần chứng minh theo yêu cầu đề bài Tiếp theo, có thể giải bài đã theo các cách sau: Cách Vì a, b, c > nên ta có: a2 + bc P =p a(b + c).(a2 + bc) b2 + ca +p b(c + a).(b2 + ca) c2 + ab +p c(a + b).(c2 + ab) Áp dụng bất đẳng thức Cauchy cho hai số thực dương a(b + c) và (a2 + bc), ta được: p ab + ac + a2 + bc (a + b)(a + c) a(b + c) (a2 + bc) ≤ = 2 (a2 + bc) a2 + bc ≥ Suy p (a + b)(a + c) a(b + c) (a2 + bc) Chứng minh tương tự, ta được: (1.1) b2 + ac (b2 + ca) ≥ (b + c)(a + c) b(c + a) (b2 + ca) (c2 + ab) c2 + ab p ≥ (c + a)(c + b) c(a + b) (c2 + ab) p Cộng vế theo vế, các bất đẳng thức (1.1), (1.2) và (1.3), ta được:   a2 + bc b2 + ca c2 + ab P ≥2 + + (a + b)(a + c) (b + c)(b + a) (c + a)(c + b) (1.2) (1.3) (1.4) Ta chứng minh: a2 + bc b2 + ca c2 + ab + + ≥ (a + b)(a + c) (b + c)(b + a) (c + a)(c + b) (1.5) Thật vậy, vì a, b, c > nên      (1.5) ⇔ a2 + bc (b + c) + b2 + ca (a + c) + c2 + ab (a + b) ≥ 3(a + b)(b + c)(c + a)   ⇔ a2 b + ab2 + b2 c + bc2 + c2 a + ca2 ≥ a2 b + ab2 + b2 c + bc2 + c2 a + ca2 + 2abc ⇔ a2 b + ab2 + b2 c + bc2 + c2 a + ca2 ≥ 6abc    ⇔ a b2 + c2 − 2abc + b c2 + a2 − 2ac + c a2 + b2 − 2ab ≥ ⇔ a(b − c)2 + b(c − a)2 + c(a − b)2 ≥ (1.6) Hiển nhiên, (1.6) là bất đẳng thức đúng Vì thế, (1.5) chứng minh Từ (1.4) và (1.5), suy P ≥ Từ các chứng minh trên ta thấy, đẳng thức xảy và dấu xảy đồng thời (1.1), (1.2), (1.3) và (1.6) Dễ thấy, điều cuối cùng có và a = b = c Cách Vì a, b, c > nên ta có: p p p (a2 + bc) (ab + ac) (b2 + ca) (bc + ba) (c2 + ab) (ca + cb) P = + + (2.1) a(b + c) b(c + a) c(a + b)  √  √ √  Áp dụng bất đẳng thức Bunhiacopxki cho hai số a, bc và ab, ac , ta được:  √ √ √ 2  a2 + bc (ab + ac) ≥ a ab + bc ac = ab (a + c)2 106 (107) BẤT ĐẲNG THỨC AM-GM √ Suy (a2 + bc) (ab + ac) ≥ ab.(a + c) Bằng cách tương tự, ta chứng minh được: q √ (b2 + ac) (bc + ba) ≥ bc.(b + a) q √ (c2 + ab) (ca + cb) ≥ ca.(c + b) q Từ (2.1), (2.2), (2.3) và (2.4), suy √ √ √ ab(a + c) bc(b + a) ca(c + b) P ≥ + + a(b + c) b(c + a) c(a + b) Ký hiệu Q là vế phải (2.5) Theo bất đẳng thức Cauchy cho số thực dương, ta có: s√ √ √ ab(a + c) bc(b + a) ca(c + b) + + = Q≥ a(b + c) b(c + a) c(a + b) (2.2) (2.3) (2.4) (2.5) (2.6) Từ (2.5) và (2.6), suy P ≥ Từ các chứng minh trên ta thấy, đẳng thức xảy và dấu xảy đồng thời (2.2),(2.3), (2.4) và (2.6) Dễ thấy, điều cuối cùng có và a = b = c Cách Theo bất đẳng thức Cauchy cho số thực dương, ta có: s a2 + bc b2 + ca c2 + ab P ≥36 (3.1) a(b + c) b(c + a) c(a + b) Tiếp theo, ta chứng minh: a2 + bc b2 + ca c2 + ab ≥ a(b + c) b(c + a) c(a + b) (3.2) Thật vậy, a, b, c > nên (3.2) ⇔ a2 + bc  c2 + ab ≥ abc(a + b)(b + c)(c + a)  √   √  Áp dụng bất đẳng thức Bunhiacopxki cho hai số a, bc và b, bc , ta được:  b2 + ca a2 + bc   (3.3)  b2 + bc ≥ (ab + bc)2 Hay  b a2 + bc (b + c) ≥ b2 (a + c)2 (3.4) Chứng minh tương tự, ta có:  c b2 + ca (c + a) ≥ c2 (b + a)2  a c2 + ab (a + b) ≥ a2 (c + b)2 (3.5) (3.6) Vì các vế (3.4), (3.5) và (3.6) dương nên nhân các bất đẳng thức đó với nhau, vế theo vế, chia vế bất đẳng thức thu cho abc(a + b)(b + c)(c + a), ta nhận bất đẳng thức (3.3) Vì thế, (3.2) chứng minh Từ (3.1) và (3.2), hiển nhiên suy P ≥ Từ các chứng minh trên ta thấy, đẳng thức xảy và dấu xảy đồng thời (3.1), (3.4), (3.5) và (3.6) Dễ thấy, điều cuối cùng có và a = b = c 107 (108) BẤT ĐẲNG THỨC AM-GM Câu 1.50 Áp dụng Cauchy ta có:   1 (a1 + a2 + · · · + a2018 ) = (a1 + a2 + · · · + a2018 ) + + ··· + a1 a2 a2018 √ 2018 ≥ 2018 2018 a1 a2 · · · a2018 · 2018 = 20182 √ a1 a2 · · · a2018 Hay a1 + a2 + · · · + a2018 ≥ 2018   a2018 2017 a22 + a33 + 2018 + 2017 + + ··· + − + + ··· + Ta có P = a1 + 2018 2018 Tiếp tục sử dụng Cauchy và sử dụng (1) ta thu a1 + a22 + a33 + a2018 + 2017 + + · · · + 2018 ≥ a1 + a2 + · · · + a2018 ≥ 2018 2018 Dấu đẳng thức xảy a1 = a2= · · · = a2018 =  2017 1 Từ (2) và (3) suy P ≥ 2018 − + + ··· + = + + + ··· + 2018 2018 108 (1) (2) (3) (109) BẤT ĐẲNG THỨC CAUCHY-SCHWARZ §2 Bất đẳng thức Cauchy-Schwarz Câu 2.1 Bình phương hai vế và rút gọn, ta có q (a21 + a22 + · · · + a2n ) (b21 + b22 + · · · + b2n ) ≥ (a1 b1 + a2 b2 + · · · + an bn ) Đây chính là bất đẳng thức Cauchy-Schwarz dạng đa thức Đẳng thức xảy = kbi Câu 2.2 Áp dụng bất đẳng thức Cauchy – Schwarz ta có     a2 + b2 + = a2 + 1 + b2 ≥ (a + b)2 Tương tự, ta có (b2 + 1) (c2 + 1) ≥ (b + c)2 , (c2 + 1) (a2 + 1) ≥ (a + c)2 Nhân ba bất đẳng thức trên theo vế ta    a2 + b2 + c2 + ≥ (a + b) (b + c) (c + a) Đẳng thức xảy a = b = c = Câu 2.3 Áp dụng bất đẳng thức Cauchy – Schwarz ta có   a2 + b2 + 1 + + c2 ≥ (a + b + c)2 = hay là a2 + b2 + ≥ c2 +2 Tương tự b2 + c2 + ≥ a2 9 và c2 + a2 + ≥ +2 b +2 Cộng các bất đẳng thức trên theo vế, ta   2 2 a +b +c +3≥9 1 + + 2 a +2 b +2 c +2  Đẳng thức xảy a = b = c = Câu 2.4 Áp dụng bất đẳng thức Cauchy – Schwarz ta có √ √ √ √ √ √ V T = a · a3 + 8abc + b · b3 + 8abc + c · c3 + 8abc p ≤ (a + b + c) (a3 + b3 + c3 + 24abc) Mặt khác (a + b + c)3 = a3 + b3 + c3 + (a + b) (b + c) (c + a) ≥ a3 + b3 + c3 + 24abc Suy VT ≤ q (a + b + c) (a + b + c)3 = (a + b + c)2 = 1 Bài toán chứng minh Đẳng thức xảy a = b = c = 109 (110) BẤT ĐẲNG THỨC CAUCHY-SCHWARZ Câu 2.5 Áp dụng bất đẳng thức Cauchy-Schwarz ta có a4 (a2 + b2 + c2 ) ≥ a(b + 2c) a(b + 2c) + b(c + 2a) + c(a + 2b) 2 2 a2 + b2 + c2 (a + b + c ) ≥ = = 3(ab + bc + ca) VT = X Câu 2.6 Áp dụng bất đẳng thức Cauchy-Schwarz ta có a4 a b2 + c2 + (a2 + b2 + c2 )2 √ √ ≥ √ a b2 + c2 + + b c2 + a2 + + c a2 + b2 + (a2 + b2 + c2 )2 ≥p (a2 + b2 + c2 )(2a2 + 2b2 + 2c2 + 21) √ t t =√ , t = a2 + b2 + c2 ≥ 2t + 21 VT = X √ Ta chứng minh √ t t √ ≥ ⇔ t3 ≥ 2t + 21 ⇔ (t − 3)(t2 + 3t + 9) ≥ (đúng) 2t + 21 Câu 2.7 Áp dụng bất đẳng thức Cauchy-Schwarz ta có: (a + b + c)2 = 4a2 + b2 + c2 2a + (a2 + b2 ) + (a2 + c2 ) a2 b2 c2 b2 c2 ≤ 2+ + = + + 2a a + b2 a2 + c2 a2 + b2 a2 + c2 Tương tự: a2 c2 ≤ + + 2 2 2 a + 4b + c a +b c +b và a2 b2 ≤ + + a2 + b2 + 4c2 a2 + c2 b2 + c2 Cộng bất đẳng thức trên theo vế ta có đpcm Câu 2.8 Bất đẳng thức cần chứng minh tương đương với       3 1− + 1− + 1− ≤ 2a + 2b + 2c + a2 b2 c2 ⇔ + + ≤ 2a + 2b + 2c + (1) Áp dụng bất đẳng thức Cauchy-Schwarz ta có 25 25 = 2 3(2a + 3) 6a + (a + b + c)2 (2 + + 1)2 = 2(2a2 + bc) + 2a(a + b + c) + a2 + b2 + c2 2 ≤ + + 2a + bc a(a + b + c) a + b2 + c2 110 (111) BẤT ĐẲNG THỨC CAUCHY-SCHWARZ Suy a2 ≤ 2a2 + 25  V T (1) ≤ 25  2a2 2a a2 + + 2a2 + bc a + b + c a2 + b2 + c2  Suy  2a2 2b2 2c2 + + +3 2a2 + bc 2b2 + ca 2c2 + ab Ta chứng minh a2 b2 c2 + + ≤1 2a2 + bc 2b2 + ca 2c2 + ab bc ca ab ⇔ + + ≥ (2) 2a + bc 2b + ca 2c + ab Áp dụng bất đẳng thức Cauchy-Schwarz ta có V T (2) ≥ (ab + bc + ca)2 (ab + bc + ca)2 = = 2abc(a + b + c) + a2 b2 + b2 c2 + c2 a2 (ab + bc + ca)2 Vậy bài toán chứng minh Câu 2.9 Bất đẳng thức cần chứng minh tương đương với       1 + 1− + 1− ≤ 1− a +1 b +1 c +1 2 2 a b c ⇔ + + ≤ a +1 b +1 c +1 4b2 4c2 4a2 + + ≤ ⇔ 3a + ab + bc + ca 3b + ab + bc + ca 3c + ab + bc + ca Ta có 4a2 (a + a)2 = 3a2 + ab + bc + ca (2a2 + bc) + a(a + b + c) ≤ 2a2 a a + + bc a + b + c Suy a2 b2 c2 V T (1) ≤ + + + ≤ + = 2a + bc 2b2 + ca 2c2 + ab Câu 2.10 Ta có ab 2ab =1+ = − ab a + b2 + c2 − ab a2 + b2 + 2c2 + (a − b)2 (a + b) ≤1+ 2(a2 + b2 + 2c2 )   a2 b2 + ≤1+ a2 + c2 b + c2 Tương tự và ≤1+ − bc  ≤1+ − ca  b2 c2 + b2 + a2 c2 + a2 c2 a2 + c2 + b2 a2 + b2 Công ba bất đẳng thức trên theo vế ta có đpcm 111   (1) (112) BẤT ĐẲNG THỨC CAUCHY-SCHWARZ Câu 2.11 Ta có VT = x2 + y2 y2 z2 + z2 x2 + + 3yz + + 3zx + + 3xy 5 y x z + + + x + y + 3yz y + z + 3zx z + x2 + 3xy Áp dụng bất đẳng thức Schwarz ta được:  2 1 √ +√ +√ y x z 4.9 VT ≥ + 2 2 2 2(x + y + z ) + 3(xy + yz + zx) 2(x + y + z ) + 3(xy + yz + zx) 36 45 ≥ + 2 2 2 2(x + y + z ) + 3(xy + yz + zx) 2(x + y + z ) + 3(xy + yz + zx) 162 84 ≥ = 2 2 2(x + y + z ) + 3(xy + yz + zx) x + y + z + 27 Vậy bài toán chứng minh Câu 2.12 Áp dụng bất đẳng thức Cauchy-Schwarz ta có a b c a2 b2 c2 + + = + + b + 2c c + 2a a + 2b a(b + 2c) b(c + 2a) c(a + 2b) (a + b + c)2 (a + b + c)2 = ≥ ≥ a(b + 2c) + b(c + 2a) + c(a + 2b) 3(ab + bc + ca) Do đó  VT ≥ a b c + + b + 2c c + 2a a + 2b 2(a + b + c) 2 ≥ √ √ √ Câu 2.13 Đặt x = ab, y = bc, z = ca, ta có √ √ √ b a ba x2 c b y2 a c z2 √ √ = √ √ √ = , = , = √ √ 4yz − z 4c a − a b 4xz − x2 4a b − b c 4xy − y 4b c − c a 4b ca − ca Áp dụng bất đẳng thức Cauchy-Schwarz ta có (x + y + z)2 3(xy + yz + zx) VT ≥ ≥ = 2 4(xy + yz + zx) − (x + y + z ) 4(xy + yz + zx) − (xy + yz + zx) Câu 2.14 Đặt 1 = x, = y, = z ta có x2 + y + z ≥ và ta chứng minh bất đẳng thức a b c √ x3 x3 x3 + + ≥ 2 2 y +z y +z y +z Ta có: x3 x3 x3 x4 y4 z4 + + = + + y2 + z2 y2 + z2 y2 + z2 x (y + z ) y (z + x2 ) z (x2 + y ) (x2 + y + z ) ≥ z (y + z ) + y (z + x2 ) + z (x2 + y ) 112 (113) BẤT ĐẲNG THỨC CAUCHY-SCHWARZ Áp dụng bất đẳng thức AM-GM, ta có √ p p 2 2 2 x y +z = √ 2x (y + z ) (y + z ) ≤ x + y2 + z2 x2 + y + z √ p  p 2 y z + x2 = √ 2y (z + x2 ) (z + x2 ) ≤ x + y2 + z2 x2 + y + z √ p  p 2 2 2 2z (x + y ) (x + y ) ≤ x + y2 + z2 x2 + y + z z x +y = √ 2  Suy √ √ 3p x3 y3 z3 + z2 ≥ x + y + + ≥ y + z z + x2 x2 + y 2 Vậy bất đẳng thức chứng minh Đẳng thức xảy và chi a = b = c = √ Câu 2.15 Áp dụng bất đẳng thức Cauchy-Schwarz ta có 100 X (3S)2 = = ≤ = !2 ak+1 (a2k + 2ak+1 ak+2 ) k=1 100 X 100 X ≤ ! 100 X a2k+1 k=1 ! a2k + 2ak+1 ak+2 2 = 100 X ! a2k + 2ak+1 ak+2 2 k=1 a4k + 4a2k ak+1 ak+2 + 4a2k+1 a2k+2  k=1 k=1 ! 100 X a4k + 2a2k (a2k+1 + a2k+2 ) + 4a2k+1 a2k+2 k=1 100 X  a4k + 6a2k a2k+1 + 2a2k a2k+2 k=1 Mặt khác 100 X a4k + 2a2k a2k+1 + 2a2k a2k+2 ≤  k=1 và 100 X !2 a2k k=1 100 X a2k a2k+1 ≤ 50 X ! a22i−1 i=1 k=1 50 X ! a22j , j=1 nên ta có (3S)2 ≤ 100 X !2 a2k +4 50 X i=1 k=1 ! a22i−1 50 X ! a22j ≤1+ j=1 50 X a22i−1 + i=1 √ Suy S ≤ Câu 2.16 Bất đẳng thức cần chứng minh tương đương với X cyc 1 ≤ a(a + b + c) + 2(ab + bc + ca) ab + bc + ca ⇔ X cyc 2(ab + bc + ca) ≤2 a(a + b + c) + 2(ab + bc + ca) 113 50 X j=1 !2 a22j = (114) BẤT ĐẲNG THỨC CAUCHY-SCHWARZ Hay a(a + b + c) ≥ a(a + b + c) + 2(ab + bc + ca) X cyc Ta có V T (4) = (a + b + c) X cyc (4) a2 a2 (a + b + c) + 2a(ab + bc + ca) (a + b + c)3 =1 cyc [a (a + b + c) + 2a(ab + bc + ca)] ≥P Vậy bài toán chứng minh r r r b2 c2 a2 Câu 2.17 Đặt S = a + + b + + c + c a b Áp dụng bất đẳng thức Cauchy-Schwarz ta có 2    b2 b (3 + 1) + ≥ + c c Suy r a Do đó S≥  b2 3+ ≥ c   ab 3a + c ab bc ca + + 3(a + b + c) + c a b  Lại có ab bc ca + + ≥ a + b + c c a b Nên ta có S ≥ 2(a + b + c) Đặt x = ab, y = bc, z = ca, đó bất đẳng thức   ab bc ca + + ≥S c a b p p √ ⇔ 2(x2 + y + z ) ≥ x x2 + 3z + y y + 3x2 + z z + 3y (4) Áp dụng AM-GM ta có √ √ 1 x x2 + 3z = (2x) x2 + 3z ≤ (4x2 + x2 + 3z ) = (5x2 + 3z ) 4 Tương tự cho hai bđt còn lại và công bđt đó theo vế ta có đpcm Câu 2.18 Đặt a = + x2 , b = + y , c = + z Ta có x≤ Do đó x2 + a b c = ,y≤ ,z≤ 2 2   a b c VT ≥2 + + b + 2c c + 2a a + 2b (a + b + c)2 ≥2 ≥ a(b + 2c) + b(c + 2a) + c(a + 2b) 114 (115) BẤT ĐẲNG THỨC CAUCHY-SCHWARZ Câu 2.19 Theo bất đẳng thức Cauchy-Shwarz, ta có a p 4(b3 + c3 ) + c a2 c2 b b2 + = p + + c+a a+b a 4(b3 + c3 ) bc + ab bc + ca (a + b + c)2 ≥ p a 4(b3 + c3 ) + 2bc + a(b + c) Từ đó, để chứng minh bất đẳng thức bài ra, ta chứng minh (a + b + c)2 a p 4(b3 + c3 ) + 2bc + a(b + c) ≥ (1) với a, b, c > Vì a, b, c > nên p (1) ⇔ 2a2 + a(b + c) + 2(b2 − bc + c2 ) ≥ 3a 4(b3 + c3 ) (2) Vì b2 − bc + c2 > nên áp dụng bất đẳng thức Cauchy cho số thực dương 2a2 , a(b + c) và b2 − bc + c2 , ta p p 2a2 + a(b + c) + 2(b2 − bc + c2 ) ≥ 3 2a2 · a(b + c) · 2(b2 − bc + c2 ) = 3a 4(b3 + c3 ) Vậy (2) chứng minh; đó (1) chứng minh và vì bất đẳng thức bài toán chứng minh Dễ thấy, đẳng thức xảy và a = b = c Câu 2.20 Kí hiệu P là vế trái bất đẳng thức cần chứng minh theo yêu cầu đề bài • Nếu a = thì từ các ràng buộc a, b, c suy c > Do đó P = b + > c • Nếu b = thì từ các ràng buộc a, b, c suy c > Do đó P = 2c a 2c a + > + = c c + 2a a + c 2c + 2a • Nếu c = thì từ các ràngrbuộc a, b, c suy a,b > a b a b Do đó P = + ≥2· · = Dấu xảy và b = 2a b 4a b 4a • Xét a,b,c > Khi đó, theo bất đẳng thức Cauchy-Schwarz, ta có 4a2 b2 4c2 (2a + b + 2c)2 P = + + ≥ (1.1) 4ab + 4ac bc + 4ab 2c + 4ac 8ab + 8ac + bc + 2c2 (2a + b + 2c)2 Tiếp theo, ta chứng minh >1 (1.2) 8ab + 8ac + bc + 2c2 Thật vậy, ta có (1.2) ⇔ 4a2 + b2 + 4c2 + 4ab + 8ac + abc > 8ab + 8ac + bc + 2c2 ⇔ (2a − b)2 + (3b + 2c)c > Bất đẳng thức cuối cùng hiển nhiên đúng và vì (1.2) chứng minh Từ (1.1) và (1.2), hiển nhiên suy P > Vậy, tóm lại, bất đẳng thức đề bài chứng minh Dấu đẳng thức xảy và b = 2a và c = p Câu 2.21 Ta có a + b + c ≤ 3(a2 + b2 + c2 ) = Bất đẳng thức cần chứng minh tương đương với a b c + + ≥ (1) 2−a 2−b 2−c 115 (116) BẤT ĐẲNG THỨC CAUCHY-SCHWARZ Ta có V T (1) = b2 c2 (a + b + c)2 a2 + + ≥ a(2 − a) b(2 − b) c(2 − c) 2(a + b + c) − (a + b + c)2 ≥ ⇔ (a + b + c − 3)2 ≥ (bđt luôn đúng) Đẳng thức xảy 2(a + b + c) − và a = b = c = Ta chứng minh Câu 2.22 Để ý 1 − 2ab = 2− Do đó bất đẳng thức cần chứng minh tương đương − ab − ab với − 2ab − 2bc − 2cd − 2da + + + ≥ − ab − bc − cd − da Vì − 2ab = (a − b)2 + c2 + d2 ≥ nên ta có thể áp dụng bất đẳng thức Cauchy-Schwarz sau − 2ab − 2bc − 2cd − 2da + + + ≥ − ab − bc − cd − da [(1 − 2ab) + (1 − 2bc) + (1 − 2cd) + (1 − 2da)]2 ≥ (1 − 2ab)(1 − ab) + (1 − 2bc)(1 − bc) + (1 − 2cd)(1 − cd) + (1 − 2da)(1 − da) 4[2 − (a + c)(b + d)]2 = − 3(a + c)(b + d) + 2(a2 + c2 )(b2 + d2 ) Bài toán quy chứng minh   [2 − (a + c)(b + d)]2 ≥ − 3(a + c)(b + d) + 2(a2 + c2 )(b2 + d2 ) Bất đẳng thức này tương đương với − 6(a + c)(b + d) + 3(a + c)2 (b + d)2 − 4(a2 + c2 )(b2 + d2 ) ≥ 0, hay [1 − (a + c)(b + d)]2 + − 4(a2 + c2 )(b2 + d2 ) ≥ Ta có − 4(a2 + c2 )(b2 + d2 ) = (a2 + b2 + c2 + d2 )2 − 4(a2 + c2 )(b2 + d2 ) = (a2 + c2 − b2 − d2 )2 ≥ Nên bất đẳng thức cuối đúng Bài toán chứng minh 2a2 2b2 2c2 ,y = và z = với a,b,c > Thay vào bất bc ca ab đẳng thức đã cho, ta có thể viết nó dạng sau Câu 2.23 Từ giả thiết, ta có thể đặt x = a4 b4 c4 + + ≥ a4 + a2 bc + b2 c2 b4 + b2 ca + c2 a2 c4 + c2 ab + a2 b2 Sử dụng bất đẳng thức Cauchy- Schwarz, ta dễ thấy (a2 + b2 + c2 ) VT ≥ , (a + b4 + c4 ) + abc(a + b + c) + (a2 b2 + b2 c2 + c2 a2 ) suy ta cần chứng minh bất đẳng thức sau là đủ 2 a2 + b2 + c2 ≥ a4 + b4 + c4 + abc (a + b + c) + a2 b2 + b2 c2 + c2 a2 ⇔ a2 b2 + b2 c2 + c2 a2 ≥ abc(a + b + c) ⇔ (ab − ac)2 + (bc − ba)2 + (ca − cb)2 ≥ Bài toán chứng minh xong 116 (117) BẤT ĐẲNG THỨC CAUCHY-SCHWARZ 2a2 2b2 2c2 Câu 2.24 Từ giả thiết, ta có thể đặt x = ,y = và z = với a,b,c > Thay vào bất bc ca ab đẳng thức đã cho, ta có thể viết nó dạng sau a4 b4 c4 + + ≥ a4 + a2 bc + b2 c2 b4 + b2 ca + c2 a2 c4 + c2 ab + a2 b2 Sử dụng bất đẳng thức Cauchy Schwarz, ta dễ thấy VT ≥ (a2 + b2 + c2 ) , (a4 + b4 + c4 ) + abc(a + b + c) + (a2 b2 + b2 c2 + c2 a2 ) suy ta cần chứng minh bất đẳng thức sau là đủ 2 a2 + b2 + c2 ≥ a4 + b4 + c4 + abc (a + b + c) + a2 b2 + b2 c2 + c2 a2 , hay a2 b2 + b2 c2 + c2 a2 ≥ abc(a + b + c) ⇔ (ab − ac)2 + (bc − ba)2 + (ca − cb)2 ≥ Bài toán chứng minh xong Câu 2.25 Từ xyz = ⇒ x = với a2 b2 c2 ,y = ,z = Bất đẳng thức cần chứng minh tương đương bc ca ab a4 b4 c4 + + ≥ (a2 − bc)2 (b2 − ca)2 (c2 − ab)2 (1) Áp dụng bất đẳng thức Schwarz ta có: (a2 + b2 + c2 ) V T (1) ≥ (a2 − bc)2 + (b2 − ca)2 + (c2 − ab)2 Ta chứng minh: 2 2 (a2 + b2 + c2 )2 ≥ (a2 − bc)2 + b2 − ca + c2 − ab ⇔ (ab + bc + ca)2 ≥ Vậy bài toán chứng minh 1 ; b = ; c = ⇒ x ; y ; z > Bất đẳng thức cần chứng minh tương x y z đương với bất đẳng thức sau Câu 2.26 Đặt a = x y z p +p +p ≥ z (3x + y) x (3y + z) y (3z + x) Đặt vế trái là P , sử dụng bất đẳng thức C - S ta có P = x2 y2 z2 p + p + p x z (3x + y) y x (3y + z) z y (3z + x) (x + y + z)2 p p ≥ p x z (3x + y) + y x (3y + z) + z y (3z + x) p p p Đặt Q = x z (3x + y) + y x (3y + z) + z y (3z + x) Sử dụng bất đẳng thức Cauchy - Schwarz ta có √ p √ p √ p Q = x xz (3x + y) + y yx (3y + z) + z zy (3z + x) p ≤ (x + y + z) [xz (3x + y) + xy (3y + z) + zy (3z + x)] p = (x + y + z) (x2 z + y x + z y + xyz) 117 (118) BẤT ĐẲNG THỨC CAUCHY-SCHWARZ Mặt khác, ta chứng minh được: x2 z + y x + z y + xyz ≤ (x + y + z)3 Thật vậy, không 27 tính tổng quát ta giả sử y là số nằm x và z Khi đó (y − x) (y − z) ≤ ⇔ y + xz ≤ xy + yz ⇔ y x + x2 z ≤ x2 y + xyz x+z x+z ⇒ x2 z + y x + z y + xyz ≤ x2 y + z y + 2xyz = y (x + z)2 = 4.y 2 3  x+z x+z + y+  2  (x + y + z)3 ≤   = 27 Do đó Q ≤ (x + y + z)2 ⇒ P ≥ (đpcm) Đẳng thức xảy x = y = z Câu 2.27 Khi thay a, b, c −a, − b, − c thì bất đẳng thức không đổi, đó ta giả sử a ≤ ≤ b, c Khi đó ta thay a −a ta cần chứng minh BĐT (a2 − ab + b2 )(b2 + bc + c2 )(c2 − ca + a2 ) ≥ 3(bc − ab − ac)3 (1) với a, b, c ≥ Ta xét bc − ca − ab ≥ Do b2 + bc + c2 − (bc − ac − ab) = b2 + c2 + a(b + c) ≥ 0, nên ta chứng minh (a2 − ab + b2 )(c2 − ca + a2 ) ≥ 3(bc − ab − ac)2 (2) Ta có     4(a2 − ab + b2 ) · 4(c2 − ca + a2 ) = (a + b)2 + 3(b − c)2 · (c + a)2 + 3(c − a)2 ≥ [(a + b)(a + c) + 3(a − c)(a − b)]2 Ta chứng minh (a + b)(a + c) + 3(a − b)(a − c) ≥ 4(bc − ab − ac) Bất đẳng thức này đúng vì nó tương đương với ⇔ a(2a + b + c) ≥ Câu 2.28 Áp dụng bất đẳng thức Cauchy-Schwarz ta có VT = X (a2 + b2 + c2 )2 a4 P ≥ a2 (2b2 − bc + 2c2 ) a2 (2b2 − bc + 2c2 ) Ta chứng minh (a2 + b2 + c2 )2 ≥ X a2 (2b2 − bc + 2c2 ), hay X a4 + abc X a≥2 X a2 b2 Áp dụng bất đẳng thức Schur ta có X X X X a4 + abc a≥ ab(a2 + b2 ) ≥ a2 b2 , nên (1) đúng 118 (1) (119) BẤT ĐẲNG THỨC CAUCHY-SCHWARZ Câu 2.29 Áp dụng bất đẳng thức Cauchy-Schwarz, ta có  2 √ √ P = a(2 − bc) + 2(b + c) ≤ (a2 + 2) (2 − bc)2 + 2(b + c)2 = (a2 + 2)(b2 + 2)(c2 + 2)  Lại áp dụng bất đẳng thức AM-GM, ta có (a2 + 2)(b2 + 2)(c2 + 2) = 3(a2 + 2).2(b2 + 2).(c2 + 2)  3 3(a2 + 2) + 2(b2 + 2) + (c2 + 2) ≤ = 36 Từ đó suy P ≤ 36 Suy −6 ≤ P ≤ Mặt khác với a = 0, b = 1, c = thì P = 6; a = 0, b = 1, c = thì P = −6 Vậy Pmax = và Pmin = −6 Câu 2.30 Đặt P = b+c c+a a+b + + Áp dụng bất đẳng thức Cauchy 2 (a + b + 1) (b + c + 1) (c + a + 1)2 - Schwarz ta có  2(a + b + c)P ≥ a+b b+c c+a + + a+b+1 b+c+1 c+a+1 2 Tiếp theo ta chứng minh a+b b+c c+a + + ≥ 2, a+b+1 b+c+1 c+a+1 hay 1 + + ≤ a+b+1 b+c+1 c+a+1 Bất đẳng thức này đúng a3 1 1 + + ≤ 3 + b + abc b + c + abc c + a + abc abc Câu 2.31 Áp dụng bất đẳng thức Cauchy-Schwarz ta có (x4 + y)( z2 + 1) ≥ (x2 + z)2 ⇒ (x4 + y)(z + y) ≥ y(x2 + z)2 y Tương tự: (y + z)(x2 + z) ≥ z(y + x)2 (x4 + y)(z + y) ≥ y(x2 + z)2 (z + x)(y + x) ≥ x(z + y)2 Nhân các bất đẳng thức trên theo vế ta (x4 + y)(y + z)(z + x) ≥ xyz(x + y )(y + z )(z + x2 ) ≥ (x + y )(y + z )(z + x2 ) Câu 2.32 Áp dụng bất đẳng thức Cauchy-Schwarz ta có s  a b c VT ≤ + + · (a + b + c) 3a + b 3b + c 3c + a 119 (120) BẤT ĐẲNG THỨC CAUCHY-SCHWARZ Ta chứng minh b c a + + ≤ 3a + b 3b + c 3c + a X b ⇔ ≥ 3a + b cyc Ta có X cyc X b (a + b + c)2 b2 = ≥ 3a + b 3ab + b2 a2 + b2 + c2 + 3(ab + bc + ca) cyc Do đó, ta chứng minh (a + b + c)2 ≥ 2 a + b + c + 3(ab + bc + ca) 2 ⇔ a + b + c ≥ ab + bc + ca (đúng) Câu 2.33 Từ giả thiết có (1 − b2 )(1 − c2 ) ≥ (a − bc)2 Bất đẳng thức cần chứng minh tương đương với (1 − b6 )(1 − c6 ) ≥ (a3 − b3 c3 )2 , hay (1 − b2 )(1 − c2 )(1 + b2 + b4 )(1 + c2 + c4 ) ≥ (a − bc)2 (a2 + abc + b2 c2 )2 Như cần chứng minh (1 + b2 + b4 )(1 + c2 + c4 ) ≥ (a2 + abc + b2 c2 )2 Áp dụng bất đẳng thức Cauchy-Schwarz, ta có (1 + b2 + b4 )(1 + c2 + c4 ) ≥ (1 + |bc| + b2 c2 )2 ≥ (a2 + |abc| + b2 c2 )2 ≥ (a2 + abc + b2 c2 )2 Do −1 ≤ a ≤ Suy điều phải chứng minh Dấu xảy a = b = c = Câu 2.34 Ta có (a2 + b2 + c2 )  1 + 2+ 2 a b c   a b c + + b c a 2  b c a + + + a b c  2 a b c b c a ≥ + + + + + b c a a b c   a b c b c a ≥ + + + + + b c a a b c   b+c c+a a+b = + + a b c ≥ Suy  1 + 2+ 2 a b c  ≥ b+c c+a a+b + + a b c 120 2 ! (121) BẤT ĐẲNG THỨC CAUCHY-SCHWARZ Câu 2.35 Bất đẳng thức cần chứng minh tương đương với n−1  X k=1 2)x21 (n − + 2x1 + (n − 1)x1 + n−1 n−1 P −2 xk k=1 k=1 n−1 2 P (n − 1) xk + (n − 2)  2 n−1 P ≥1− xk k=1 ⇔ n−1 X k=1 2 n−1 P ((n − 1)xk + 1)2 ≥ (n − 1)((n − 1)x2k + 1) xk + n−1 2 P (n − 1) xk + k=1 k=1 Áp dụng bất đẳng thức Cauchy-Schwarz ta có n−1 X k=1 n−1 P ((n − 1)xk + 1)2 ≥ (n − 1)((n − 1)x2k + 1) 2 xk + k=1 n−1 P x2k +1 k=1 Nên ta chứng minh n−1 P 2 xk + k=1 n−1 P 2 n−1 P xk + , ≥ n−1 2 P xk + (n − 1) k=1 x2k + k=1 k=1 hay (n − 1) n−1 X !2 xk ≥ k=1 n−1 X x2k (1) k=1 Câu 2.36 Áp dụng bđt Cauchy-Schwarz ta có 1 (a + b + c)2 + + ≥ 2a2 + bc 2b2 + ac 2c2 + ab ab + bc + ac 4(a + b + c)2 ≥P (2a + bc)(b2 + c2 + 2bc) Ta chứng minh X (2a2 + bc)(b2 + c2 + 2bc) ≤ 4(a2 + b2 + c2 )(ab + bc + ca), hay X ab(a2 + b2 ) ≥ X a2 b2 Bất đẳng thức này đúng theo AM-GM s Câu 2.37 Áp dụng bất đẳng thức AM − GM ta có P ≥ · 3n a2 + bc b2 + ac c2 + ab · · a(b + c) b(a + c) c(a + b) (1) a2 + bc b2 + ac c2 + ab · · ≥ a(b + c) b(a + c) c(a + b) Thật vậy: Do a, b, c dương nên bất đẳng thức (1) đưa Ta chứng minh (a2 + bc)(b2 + ac)(c2 + ab) ≥ abc(a + b)(b + c)(c + a) (3) 121 (2) (122) BẤT ĐẲNG THỨC CAUCHY-SCHWARZ  √   √  Áp dụng bất đẳng thức Cauchy - Schwarz cho số a, bc và b, bc ta (a2 + bc)(b2 + bc) ≥ (ab + bc)2 hay b(a2 + bc)(b + c) ≥ b2 (a + c)2 (4) 2 Chứng minh tương tự ta có c(b + ac)(a + c) ≥ c (a + b) (5) a(c2 + ab)(a + b) ≥ a2 (c + b)2 (6) Vì các vế (4), (5), (6) dương Nhân vế với vế các bất đẳng thức đó với nhau, chia vế bất đẳng thức thu cho abc(a + b)(b + c)(c + a) ta (3), đó có (2), suy P ≥ Dấu xảy và dấu xảy đồng thời (1), (4), (5) và (6), đó a = b = c Vậy giá trị nhỏ P là √ Câu 2.38 a3 + = (a3 + + 1) + ≥ a3 · · + = 3a + Tương tự: b3 + ≥ 3b + 3; c3 + ≥ 3c + b3 + c3 + 3a + 3b + 3c + a3 + + + ≥ + + 3 3 a (b + c) b (c + a) c (a + b) a (b + c) b (c + a) c (a + b) Ta có: 3a + 3b + 3c + 3a(abc) + 3(abc)2 3b(abc) + 3(abc)2 3c(abc) + 3(abc)2 + + = + + a3 (b + c) b3 (c + a) c3 (a + b) a3 (b + c) b3 (c + a) c3 (a + b) 3(bc + b2 c2 ) 3(ca + c2 a2 ) 3(ab + a2 b2 ) + + = ab + bc bc + ca ca + ab Đặt x = bc, y = ca, z = ab; x,y,z > 0, xyz =   3a + 3b + 3c + x + x2 y + y z + z + + ≥3 + + a3 (b + c) b3 (c + a) c3 (a + b) y+z z+x x+y     x y z y2 z2 x =3 + + + + +3 y+z z+x x+y y+z z+x z+y y z x + + ≥ y+z z+x x+y √ 3 xyz x2 y2 z2 x+y+z + + ≥ ≥ = y+z z+x z+y 2 a3 + b3 + c3 + Vậy + + ≥ a (b + c) b3 (c + a) c3 (a + b) Dấu “=” xảy a = b = c = Câu 2.39 Biến đổi và áp dụng bất đẳng thức Cauchy - Schwartz: n X k=1 ak (bk + ak+1 ) = n X ak bk + k=1 n X ak ak+1 k=1 v ! n ! u n n X X u X 2 t ak bk + ≤ ak ak+1 k=1 k=1 k=1 v u n n X uX a2 + ak ak+1 =t k k=1 k=1 v !2 u n n u X X X t ak ak+1 = ak − aj + 1≤i<j≤n k=1 122 k=1 (123) BẤT ĐẲNG THỨC CAUCHY-SCHWARZ s = 1−2 X aj + 1≤i<j≤n Đặt x = X aj và y = 1≤i<j≤n n X n X ak ak+1 k=1 ak ak+1 ta có x > y k=1 Đồng thời có 1= n X k=1 !2 ak = n X a2k + 2x > y ⇒ y < k=1 Do (1) nên ta cần chứng minh: √ − 2x + y < ⇔ − 2x < − 2y + y ⇔ 2(y − x) < y (hiển nhiên đúng) 123 (1) (2) (124) MỘT SỐ BẤT ĐẲNG THỨC KHÁC §3 Một số bất đẳng thức khác Câu 3.1 Ta có r 2a 2b 2c 3abc + + + b+c c+a a+b a + b3 + c3 9abc (a + b + c) + q ≥ ab + bc + ca 3 (3abc)2 (a3 + b3 + c3 ) (a + b + c)2 9abc ≥ + ab + bc + ca a + b + c3 + 6abc Ta cần chứng minh (a + b + c)2 9abc −3≥1− 3 ab + bc + ca a + b + c3 + 6abc a+b+c ⇔ ≥ ab + bc + ca a + b3 + c3 + 6abc Bất đẳng thức này hiển nhiên đúng theo Schur Câu 3.2 Vì a + b + c = 1, nên ta có     ab bc ca ab bc ca + + = (a + b + c) + + a+b b+c c+a a+b b+c c+a   1 = ab + bc + ca + abc + + a+b b+c c+a 9abc ≥ ab + bc + ca + 2(a + b + c) = ab + bc + ca + abc Suy  ab bc ca + + a+b b+c c+a  + ≥ 2(ab + bc + ca) + 9abc + Do đó, ta chứng minh 9abc + ≥ 4(ab + bc + ca) Áp dụng bất đẳng thức Schur ta có a2 + b2 + c2 + 9abc ≥ 2(ab + bc + ca) a+b+c Suy + 9abc ≥ 4(ab + bc + ca), hay (1) chứng minh Câu 3.3 Bất đẳng thức cần chứng minh tương đương với Xp p (a + b + c) (a2 + bc)(a2 + b2 )(a2 + c2 ) ≥ (a2 + b2 )(a2 + c2 )(b2 + c2 )abc cyc Áp dụng bất đẳng thức Cauchy-Schwarz ta có (a2 + b2 )(a2 + c2 ) ≥ (a2 + bc)2 , 124 (1) (125) MỘT SỐ BẤT ĐẲNG THỨC KHÁC và theo AM-GM X p (a2 + b2 )(a2 + c2 )(b2 + c2 )abc ≤ c(a2 + b2 ) cyc Do đó, ta chứng minh (a + b + c) X (a2 + bc) ≥ cyc X c(a2 + b2 ), cyc Đây chính là bđt Schur Câu 3.4 Ta có V T = a2 b2 c2 + 2(a2 b2 + b2 c2 + c2 a2 ) + 4(a2 + b2 + c2 ) + Mặt khác a2 b2 + b2 c2 + c2 a2 + = a2 b2 + + b2 c2 + + c2 a2 + ≥ 2(ab + bc + ca) và √ 3abc a2 b2 c2 + = a2 b c2 + + ≥ a2 b2 c2 = √ abc 9abc ≥ ≥ 2(ab + bc + ca) − (a2 + b2 + c2 ) a+b+c Suy V T ≥ 2(ab + bc + ca) − (a2 + b2 + c2 ) + 2.2(ab + bc + ca) + 4(a2 + b2 + c2 ) = 6(ab + bc + ca) + 3(a2 + b2 + c2 ) ≥ 6(ab + bc + ca) + 3(ab + bc + ca) ≥ 9(ab + bc + ca) Bài toán chứng minh Câu 3.5 Gọi P là vế trái bất đẳng thức cần chứng minh Không tính tổng quát, ta giả sử a + b + c = Áp dụng bất đẳng thức AM-GM ta có s s (a + b)3 (a + b)3 ab(4a + 4b + c) + + ≥ (a + b) 8ab(4a + 4b + c) 8ab(4a + 4b + c) 27 Suy s (a + b)3 ab(4a + 4b + c) + ≥ (a + b) 8ab(4a + 4b + c) 54 Tương tự s và s (b + c)3 bc(4b + 4c + a) + ≥ (b + c) 8bc(4b + 4c + a) 54 (c + a)3 ca(4c + 4a + b) + ≥ (c + a) 8ca(4c + 4a + b) 54 Cộng ba bất đẳng thức trên ta có √ P + A ≥ B 2 Với [ab(4a + 4b + c) + bc(4b + 4c + a) + ca(4c + 4a + b)] 54 = [4ab(a + b) + 4bc(b + c) + 4ca(c + a) + 3abc] 54 1 = [4(a + b + c)(ab + bc + ca) − 9abc] ≤ (a + b + c)3 = 54 54 A= 125 (126) MỘT SỐ BẤT ĐẲNG THỨC KHÁC và B = (a + b + c) = Suy √ √ P ≥ − = ⇒ P ≥ 2 2 2 Bài toán chứng minh Câu 3.6 Sử dụng bất đẳng thức AM-GM,ta có: 12(a2 + b2 + c2 ) + 6abc + 48 − 30(a + b + c) = 12(a2 + b2 + c2 ) + 3(2abc + 1) + 45 − 5.2.3(a + b + c) √ ≥ 12(a2 + b2 + c2 ) + a2 b2 c2 + 45 − 5.((a + b + c)2 + 9) 9abc = 7(a2 + b2 + c2 ) + √ − 10(ab + bc + ca) abc 27 ≥ 7(a2 + b2 + c2 ) + − 10(ab + bc + ca) a+b+c Mặt khác sử dụng bất đẳng thức Schur, ≥ 4(ab + bc + ca) − (a + b + c)2 = 2(ab + bc + ca) − (a2 + b2 + c2 ) a+b+c Do đó 27 − 10(ab + bc + ca) a+b+c ≥ 7(a2 + b2 + c2 ) + 6(ab + bc + ca) − 3(a2 + b2 + c2 ) − 10(ab + bc + 7(a2 + b2 + c2 ) + = 4(a2 + b2 + c2 − ab − bc − ca) ≥ Bất đẳng thức chứng minh 1 Câu 3.7 Đặt x = , y = , z = Bất đẳng thức cần chứng minh trở thành a b c x2 + y + z + ≥ 2(xy + yz + zx) ⇔ (x + y + z)(x2 + y + z + 3) ≥ 2(x + y + z)(xy + yz + zx) Hay x3 + y + z + 3(x + y + z) ≥ x2 (y + z) + y (z + x) + z (x + y) + (1) Ta có x3 + y + z + 3(x + y + z) ≥ x3 + y + z + = x3 + y + z + 3xyz + ≥ V P (1) Vậy bài toán chứng minh √ √ √ 3 Câu 3.8 Đặt x = a2 , y = b2 , z = c2 , ta có xyz = và bất đẳng thức cần chứng minh trở thành √  √ √ x3 + y + z + 3xyz ≥ xy xy + yz yz + zx zx (1) Áp dụng bất đăng thức Schur ta có x3 + y + z + 3xyz ≥ xy(x + y) + yz(y + z) + zx(z + x) √ √ √ ≥ 2xy xy + 2yz yz + 2zx zx Suy (1) đúng, hay bài toán chứng minh 126 (127) MỘT SỐ BẤT ĐẲNG THỨC KHÁC Câu 3.9 Áp dụng bất đẳng thức Holder với chú ý 3 4a + 9b + 36c  1 + + 1 + + = ta có 2 ≥ (a + b + c)3 = Từ đó ta có đpcm Câu 3.10 Đặt P là vế trái bất đẳng thức Áp dụng bất đẳng thức Holder ta có: X  P a(a + 2b) ≥ (a + b + c)4 Mặt khác X a(a + 2b) = (a + b + c)2 , nên ta có P ≥ (a + b + c)2 = ⇒ P ≥ Câu 3.11 Áp dụng bất đẳng thức Holder ta có  2 √ 3 √ √ a b c 3 2 (x + y + z ) + + a2 + b + c2 ≥ x y z Từ đó ta có đpcm Câu 3.12 Áp dụng bất đẳng thức Holder ta có  √ √ √ n−1 n−1 n−1 n n−1 n n n n n a + b + c (x + y + z ) ≥ (ax + by + cz)n = Suy xn + y n + z n ≥  √ n−1 an + √ bn + n−1 √ 1−n cn n−1 Câu 3.13  √ ab + bc + ca √ 1 (a + b + c) + + +4 2 ≥ + a b c a + b2 + c2   √ √ ab + bc + ca 1 + + ⇔(a + b + c) −9≥4 2−4 2 a b c a + b2 + c2 √ (a − b)2 + (a − c)2 + (b − c)2 (a − b)2 (b − c)2 (c − a)2 ⇔ + + ≥2 ab bc ca a2 + b2 + c2  Không tính tổng quát giả sử a ≥ b ≥ c ⇒ (*) (a − b)2 (a − c)2 = Áp dụng bất đẳng thức b c Chebyshev ta có:    (a − b)2 (a − c)2 + (b + c) ≥ (a − b)2 + (a − c)2 b c (a − b) (a − c)2 [(a − b)2 + (a − c)2 ] ⇒ + ≥ ab ac a(b + c)  Đẳng thức xảy và b = c Mặt khác theo bất đẳng thức AM − GM : √ 2(a2 + b2 + c2 ) ≥ 2a2 + (b + c)2 ≥ 2a(b + c) 127 (1) (128) MỘT SỐ BẤT ĐẲNG THỨC KHÁC √ 2 ≥ ⇒ a(b + c) a + b2 + c2 √ (a − b)2 + (a − c)2 [(a − b)2 + (a − c)2 ] ≥2 ⇒ a(b + c) a2 + b2 + c2 √ Đẳng thức xảy và a = b = c a = b + c √ a2 + b2 + c2 ≥ 3bc > 2bc (do a ≥ b ≥ c) √ (b − c)2 (b − c)2 ≥2 2 ⇒ bc a + b2 + c2 Từ (1),(2),(3) suy bất đẳng thức (*) đúng Suy điều phải chứng minh 128 (2) (3) (129) Chương Một số phương pháp chứng minh bất đẳng thức §1 Phương pháp quy nạp 129 (130) PHƯƠNG PHÁP PHÂN TÍCH BÌNH PHƯƠNG SOS §2 Phương pháp phân tích bình phương SOS 130 (131) PHƯƠNG PHÁP PHÂN TÍCH BÌNH PHƯƠNG SOS Câu 5.1 Bất đẳng thức cần chứng minh tương đương với   a3 + b3 + c3 − ab (a + b) − bc (b + c) − ca (c + a) − a3 + b3 + c3 − 3abc ≥ ⇔ (a + b) (a − b)2 + (b + c) (b − c)2 + (c + a) (c − a)2   − (a + b + c) (a − b)2 + (b − c)2 + (c − a)2 ≥ ⇔ (a + b − c) (a − b)2 + (b + c − a) (b − c)2 + (c + a − b) (c − a)2 ≥ ⇔ Sc (a − b)2 + Sa (b − c)2 + Sb (c − a)2 ≥ Không tính tổng quát giả sử a ≥ b ≥ c ⇒ Sb ≥ 0,Sc ≥ Ta có Sa + Sb = 2c ≥ Câu 5.2 Bất đẳng thức cần chứng minh tương đương với       ab − bc + ca bc − ca + ab ca − ab + bc + + ≥0 − − − b2 + c2 c2 + a2 a2 + b2 X (b + c)(2a − b − c) X (b + c)(a − b) X (b + c)(a − c) ⇔ ≥ ⇔ + ≥0 b2 + c2 b2 + c2 b2 + c2 X (b + c)(a − b) X (c + a)(b − a) ⇔ + ≥0 b + c2 c2 + a2 X (a − b)2 (ab + bc + ca − c2 ) ≥ (1) ⇔ (b2 + c2 )(c2 + a2 ) Mặt khác ab + bc + ca − c2 = (b − c)(c − a) + 2bc ≥ (b − c)(c − a), nên để chứng minh (1) ta chứng minh X (a − b)2 (b − c)(c − a) (b2 + c2 )(c2 + a2 ) ≥0⇔ X (a2 + b2 )(a − b)2 (b − c)(c − a) ≥ ⇔ (a − b)2 (b − c)2 (c − a)2 ≥ Bất đẳng thức cuối hiển nhiên đúng Câu 5.3 BĐT cần chứng minh tương đương với   a3 + b3 + c3 − ab (a + b) − bc (b + c) − ca (c + a) − a3 + b3 + c3 − 3abc p  p  p  ≥ ab (a2 + b2 ) − a − b + bc (b2 + c2 ) − b − c + ca (c2 + a2 ) − c − a ⇔ Sc (a − b)2 + Sa (b − c)2 + Sb (c − a)2 ≥ 0, với Sc = a + b − c − p 2ab , (a2 + b2 ) + a + b 2bc Sa = b + c − a − p , 2 (b + c2 ) + b + c 2ca Sb = c + a − b − p 2 (c + a2 ) + c + a Do a, b, c bình đẳng, không tính tổng quát giả sử a ≥ b ≥ c Ta có 2ab a ca Sc ≥ a + b − c − = + b − c ≥ 0, Sb ≥ c + a − b − ≥0 4b a+c 131 (132) PHƯƠNG PHÁP PHÂN TÍCH BÌNH PHƯƠNG SOS Ta cần chứng minh a b ! Sb + Sa = 2c − p −p (c2 + a2 ) + c + a (b2 + c2 ) + b + c   b bc bc a − = + ≥ ≥ 2b − (a + c) (b + c) a+c b+c Câu 5.4 Bất đẳng thức cần chứng minh tương đương với 4a2 + 2bc − (b2 + c2 ) 4b2 + 2ca − (c2 + a2 ) 4c2 + 2ab − (a2 + b2 ) + + ≥0 b2 + c2 c2 + a2 a2 + b2 (a2 − b2 ) + (a2 − c2 ) − (b − c)2 (b2 − c2 ) + (b2 − a2 ) − (c − a)2 ⇔ + b2 + c2 c2 + a2 2 2 2 (c − a ) + (c − b ) − (a − b) + ≥0 a2 + b2 ⇔ Sc (a − b)2 + Sa (b − c)2 + Sb (c − a)2 ≥ 0, với 2(a + b)2 − , Sc = 2 2 (b + c ) (c + a ) a + b2 2(c + a)2 Sb = − , 2 (b + c ) (b + a ) c + a2 2(b + c)2 Sa = − 2 (a + b ) (a + c ) b + c2 Giả sử a ≥ b ≥ c suy Sb ≥ 0,Sc ≥ 0, Sc ≥ Sb ≥ Sa Ta cần chứng minh Sa + Sb ≥ 0, hay       (c + a)2 c2 + a2 + (b + c)2 b2 + c2 ≥ b2 + c2 b2 + a2 + a2 + b2 a2 + c2 Ta thấy hệ số c2 vế trái là (c + a)2 + (b + c)2 lớn vế phải là (a2 + b2 ) nên ta cần chứng minh c = Hay  2 2 a4 + b4 ≥ a2 + b2 ⇔ a2 − b2 ≥ (đúng) Câu 5.5 Ta có : BĐT cần chứng minh tương đương với    a3 + b3 + c3 − 3abc + ab2 + bc2 + ca2 − a3 − b3 − c3 ≥ a2 b + b2 c + c2 a − a3 − b3 − c3   ⇔ (a + b + c) (a − b)2 + (b − c)2 + (c − a)2   − (2b + a) (a − b)2 + (2c + b) (b − c)2 + (2a + c) (c − a)2 + (2a + b) (a − b)2 + (2b + c) (b − c)2 + (2c + a) (c − a)2 ≥ ⇔ (2a − b + 2c) (a − b)2 + (2b − c + 2a) (b − c)2 + (2c − a + 2b) (c − a)2 ≥ ⇔ Sc (a − b)2 + Sa (b − c)2 + Sb (c − a)2 ≥ Nếu a ≥ b ≥ c ⇒ Sa ≥ 0,Sc ≥ 0, mặt khác Sa + 2Sb ≥ 0,Sc + 2Sb ≥ Theo tiêu chuẩn suy ĐPCM Nếu a ≤ b ≤ c ⇒ Sb ≥ 0,Sc ≥ Theo tiêu chuẩn ta cần chứng minh Sa + Sb ≥ ( hiển nhiên ) Suy điều phải chứng minh 132 (133) PHƯƠNG PHÁP PHÂN TÍCH BÌNH PHƯƠNG SOS Câu 5.6 Trước hết ta thấy :   X (x − y)2 1 (x + y + z) + + −9= , x y z xy   X y z (x − y)2 x + + −9= y+z z+x x+y (y + z)(z + x) Ta cần chứng minh :     X (x − y)2 X 3(x − y)2 1 x y z (x + y + z) + + ≥6 + + ⇔ ≥ x y z y+z z+x x+y xy (y + z)(z + x) với số thực x, y, z thuộc đoạn [1; 2] Đặt Sx = 3 − , Sy = − , yz (x + y)(x + z) zx (y + x)(y + z) − xy (z + x)(z + y) Bất đẳng thức đã cho viết dạng tương đương là: Sz = Sx (y − z)2 + Sy (z − x)2 + Sz (x − y)2 ≥ Không tính tổng quát, ta giả sử ≥ x ≥ y ≥ z ≥ Ta chứng minh Sx , Sy ≥ Thật vậy: Sx ≥ ⇔ x2 + xy + xz − 2yz ≥ 0, (đúng) Sy ≥ ⇔ y + yx + yz − 2zx ≥ x(y − z) + z(z + y − x) ≥ (do x,y,z ∈ [1; 2] nên y + z − x ≥ 0) - NếuSz ≥ , ta có đpcm - Nếu Sz < 0, ta chứng minh Sx+ 2Sz ≥ 0, Sy + 2Sz ≥ Khi đó dễ dàng thấy vì: (x − y)2 ≤ (y − z)2 + (z − x)2 và Sz < nên Sx (y − z)2 + Sy (z − x)2 + Sz (x − y)2 ≥ (Sx + 2Sz )(y − z)2 + (Sy + 2Sz )(z − x)2 ≥ Vậy trường hợp, ta luôn có đpcm Đẳng thức xảy x = y = z y = z = 1, x = và các hoán vị chúng Câu 5.8 a) Chứng minh 2 (a + b2 + c2 ) ≤ a3 + b3 + c3 , (1)   (1) ⇔ (a + b + c) a2 + b2 + c2 ≤ a3 + b3 + c3 ⇔ a2 b + a2 c + b2 c + b2 a + c2 a + c2 b ≤ a3 + b3 + c3  ⇔ (b + c) (b − c)2 + (c + a) (c − a)2 + (a + b) (a − b)2 ≥ Bất đẳng thức trên đúng nên (1) đúng Đẳng thức xảy a = b = c = (a + b2 + c2 ) − 3abc,   (2) ⇔ a3 + b3 + c3 ≤ (a + b + c) a2 + b2 + c2 − 9abc b) Chứng minh a3 + b3 + c3 ≤ ⇔ a3 + b3 + c3 + 9abc ≤ a2 b + a2 c + b2 c + b2 a + c2 a + c2 b ⇔ Sa (b − c)2 + Sb (c − a)2 + Sc (a − b)2 ≥ 0, (3) đó Sa = 3a − b − c; Sb = 3b − a − c; Sc = 3c − a − b Không tính tổng quát giả sử ≥ a ≥ b ≥ c 133 (2)  (134) PHƯƠNG PHÁP PHÂN TÍCH BÌNH PHƯƠNG SOS   Ta có V T (3) = Sa (b − c)2 + Sb (c − a)2 − (a − b)2 + (Sb + Sc ) (a − b)2 Mà Sa ≥ 0; Sb = 2(1 − a) + (b − c) ≥ 0; (c − a)2 − (a − b)2 ≥ 0;Sb + Sc = (1 − a) ≥ Suy V T (3) ≥ ⇒ (2) đúng Đẳng thức xảy a = b = c = ; a = 1,b = c = và các hoán vị 134 (135) PHƯƠNG PHÁP DỒN BIẾN §3 Phương pháp dồn biến Câu 6.1 a) Ta có bất đẳng thức sau với a, b, c > abc ≥ (a + b − c)(b + c − a)(c + a − b) Thật vậy, giả sử a ≥ b ≥ c, ta có abc − (a + b − c)(b + c − a)(c + a − b) = (a + b − c)(a − b)2 + c(a − c)(b − c) ≥ Do đó abc ≥ (3 − 2c)(3 − 2a)(3 − 2b) ⇔ abc ≥ 27 − 18(a + b + c) + 12(ab + bc + ca) − 8abc ⇔ 3abc ≥ 4(ab + bc + ca) − ⇔ abc ≥ (ab + bc + ca) − 3 Suy P = abc + 12 12 ≥ (ab + bc + ca) + − ≥ − = ab + bc + ca ab + bc + ca Dấu “=” xảy a = b = c = Vậy giá trị nhỏ P là b) Do bất đẳng thức đã cho đúng với a, b, c > thỏa  mãn a + b + c = nên nó đúng Khi đó với số a = b = x, c = − 2x với x ∈ 0;     1 k − ≥ − x (3 − 2x), ∀x ∈ 0; x2 + 2x(3 − 2x)   ⇔ k ≥ 3x(2 − x)(2x + 1), ∀x ∈ 0; 88 ta có k ≥ > Do k nguyên nên k ≥ 10 10 13 Với k = 10, ta cần chứng minh f (a,b,c) := abc + ≥ ab + bc + ca Không tính tổng quát, giả sử a ≥ b ≥ c, đó < c ≤   a+b a+b Ta chứng minh f (a, b, c) ≥ f , , c Thật vậy, điều này tương đương với 2   10 (a + b)2 10 − ≥c − ab (a + b)2 ab + bc + ca + bc + ca 10(a − b)2   ≥ c(a − b)2 ⇔ (a + b)2 (ab + bc + ca) + c(a + b) Với x = Điều này đúng c ≤ 1, ab + bc + ca ≤ và 3(a + b)(a + b + 4c) ≤ 135 (4a + 4b + 4c)2 = 36 (136) PHƯƠNG PHÁP DỒN BIẾN  Ta chứng minh f  a+b a+b 13 , , c ≥ Thật vậy, điều này tương đương với 2 c(3 − c)2 40 13 + ≥ (3 − c) + 4c(3 − c) c(3 − c) − 40 − 10(1 + c)(3 − c) + ≥0 ⇔ 3(1 + c)(3 − c)   10 c−4 ⇔ (c − 1) + ≥ 3(1 + c)(3 − c) Điều này đúng 3(4 − c)(1 + c)(3 − c) − 40 =3c3 − 18c2 + 15c − =3c(1 − c)(5 − c) − < · · − < 0, ∀c ∈ (0, 1] Vậy số nguyên k nhỏ cần tìm là k = 10 Câu 6.2 Từ điều kiện a + b + c = ta nghĩ đến việc dồn biến trung bình cộng Bài toán này ngoài trường hợp dấu xảy a = b = c = Còn xảy a = b = 0, c = các hoán vị số (0,0,3) Không tính tổng quát, giả sử c = max{a,b,c}, suy a ≤ c, b ≤ c và ≤ c ≤ 3, ≤ a + b ≤ Đặt √ √ √ f (a,b,c) = 3a2 + 4bc + + 3b2 + 4ca + + 3c2 + 4ab +   a+b a+b Ta chứng minh f (a,b,c) ≥ f , ,c 2 Bước 1: Ta chứng minh r √ √ 5(a − b)2 3a2 + 4bc + + 3b2 + 4ca + ≥ 3(a + b)2 + 8(a + b)c + 36 + Bình phương hai vế và sử dụng (x + y)2 = 2(x2 + y ) − (x − y)2 , ta 2(3a2 + 4bc + + 3b2 + 4ca + − ≥ 3(a + b)2 + 8(a + b)c + 36 + Tương đương với Ta có √ 3a2 + 4bc + − 2 √ 3b2 + 4ca + 5(a − b)2 2 √ (a − b)2 √ ≥ 3a + 4bc + − 3b2 + 4ca + √ √ 3a2 + 4bc + − 3b2 + 4ca + = √ 3a2 (a − b) [3(a + b) − 4c] √ + 4bc + + 3b2 + 4ca + Do đó, ta cần chứng minh [3(a + b) − 4c]2 ≥ √ √ 2 3a2 + 4bc + + 3b2 + 4ca + Suy √ 3a2 + 4bc + + √ 3b2 + 4ca + 136 2 ≥ [3(a + b) − 4c]2 (1) (137) PHƯƠNG PHÁP DỒN BIẾN Kết hợp với c = max{a,b,c} và bất đẳng thức Minkowski, ta có √ √ √ √ 3a2 + 4bc + + 3b2 + 4ca + ≥ 3a2 + 4b2 + + 3b2 + 4a2 + r r √ 2 √ 2 2 = 3a + (2b) + + 3b + (2a)2 + 32 p ≥ 3(a + b)2 + 4(a + b)2 + 36 p = 7(a + b)2 + 36 Từ ≤ c ≤ 3, ta có [3(a + b) − 4c]2 ≤ [3(a + b) − 4]2 = 9(a + b)2 − 24(a + b) + 16 Kết hợp với (1), ta cần chứng minh   7(a + b)2 + 36 ≥ 9(a + b)2 − 24(a + b) + 16 Tương đương với 11(a + b) [3 − (a + b)] + 15(a + b) + ≥ Bất đẳng thức cuối là đúng, vì ≤ a + b ≤ Bước giải hoàn toàn Bước Để hoàn thành bước dồn biến, ta chứng minh r 5(a − b)2 √ 3(a + b)2 + 8(a + b)c + 36 + + 3c + 4ab + p p ≥ 3(a + b)2 + 8(a + b)c + 36 + 3c2 + (a + b)2 + Bất đẳng thức trên tương đương với r 5(a − b)2 p 3(a + b)2 + 8(a + b)c + 36 + − 3(a + b)2 + 8(a + b)c + 36 p √ ≥ 3c2 + (a + b)2 + − 3c2 + 4ab + Nhân liên hợp và phân tích thành tổng bình phương, ta đưa chứng minh p √ 3c2 + (a + b)2 + + 3c2 + 4ab + r p 5(a − b)2 ≥ 3(a + b)2 + 8(a + b)c + 36 + + 3(a + b)2 + 8(a + b)c + 36 (i) Ta chứng minh r p 5(a − b)2 3c2 + (a + b)2 + ≥ 3(a + b)2 + 8(a + b)c + 36 + Thật vậy, ta có a ≤ c, b ≤ c và ≤ c ≤ Do đó, ta có r 5(a − b)2 3(a + b)2 + 8(a + b)c + 36 + p 2 = 12(a + 2ab + b ) + 32(a + b)c + 144 + 10(a2 − 2ab + b2 ) p = 22(a2 + b2 ) + 2ab + 32(a + b)c + 144 p ≤ 22(c2 + c2 ) + 2c2 + 32(a + b)c + 144 p = 46c2 + 32(a + b)c + 144 137 (2) (138) PHƯƠNG PHÁP DỒN BIẾN Do đó, ta cần chứng minh p p 3c2 + (a + b)2 + ≥ 46c2 + 32(a + b)c + 144 Bình phương hai vế và rút gọn, ta 29c2 + 25(a + b)2 + 81 − 32(a + b)c ≥ Từ a + b + c = 3, ta có 29c2 + 25(a + b)2 + 9(a + b + c)2 − 32(a + b)c ≥ hay 10c2 + 34(a + b)2 + 14c [2c − (a + b)] ≥ Bất đẳng thức cuối là đúng vì 2c − (a + b) = 2c − (3 − c) = 3c − = 3(c − 1) ≥ 0, ∀1 ≤ c ≤ (ii) Tiếp theo, ta chứng minh p √ 3c2 + 4ab + ≥ 3(a + b)2 + 8(a + b)c + 36 Thật vậy, từ c = max{a,b,c}, ta có p p 3(a + b)2 + 8(a + b)c + 36 ≤ 3(c + c)2 + 8(a + b)c + 36 p = 12c2 + 8(a + b)c + 36 Ta cần chứng minh p √ 3c2 + 4ab + ≥ 12c2 + 8(a + b)c + 36 Bình phương hai vế, ta   25(3c2 + 4ab + 9) ≥ 12c2 + 8(a + b)c + 36 Chú ý a + b + c = nên 8c2 + 100ab + 9(a + b)2 + 14c [2c − (a + b)] ≥ Bất đẳng thức cuối là đúng tương tự trường hợp trên Do đó, bất đẳng thức (2) chứng minh Bước giải hoàn toàn Bước Để hoàn thành lời giải, ta cần chứng minh   p p a+b a+b f , ,c = 3(a + b)2 + 8(a + b)c + 36 + 3c2 + (a + b)2 + ≥ 12 2 Từ a + b + c = 3, ta có p p 3(3 − c)2 + 8(3 − c)c + 36 + 3c2 + (3 − c)2 + ≥ 12 Suy √ 63 + 6c − 5c2 + √ 4c2 − 6c + 18 ≥ 12 Bình phương hai vế và khử hai lần, ta đưa (c − 1)2 (3 − c)(3c + 7) ≥ Bất đẳng thức cuối là đúng vì ta có ≤ c ≤ Bước giải hoàn toàn Vậy bài toán giải hoàn toàn Đẳng thức xảy và a = b = c = a = 3,b = c = b = 3,c = a = c = 3,a = b = 138 (139) PHƯƠNG PHÁP DỒN BIẾN Câu 6.3 Bài toán này ngoài trường hợp dấu  xảy  a = b = c = Còn xảy 3 a = b = , c = các hoán vị các số , ,0 Không tính tổng quát giả sử 2 c = min{a,b,c} Từ đây suy ≤ c ≤ Đặt √ √ √ f (a,b,c) = − 6ab + a2 + b2 + − 6bc + b2 + c2 + − 6ca + c2 + a2 Ta chứng minh  f (a,b,c) ≥ f  a+b a+b , ,c 2 Bước Ta chứng minh √ √ − 6bc + b2 + c2 + − 6ca + c2 + a2 ≥ r 36 − 12(a + b)c + (a + b)2 + 4c2 − (a − b)2 Bình phương hai vế và biến đổi, ta thu 2 √ 6(a − b)2 √ ≥ − 6bc + b2 + c2 − − 6ca + c2 + a2 Nhân liên hợp và phân tích tổng bình phương, ta đưa chứng minh [6c − (a + b)]2 ≥ √ √ 2 − 6bc + b2 + c2 + − 6ca + c2 + a2 hay √ − 6bc + b2 + c2 + √ − 6ca + c2 + a2 2 ≥ [6c − (a + b)]2 (3) Từ c = min{a,b,c} và a + b + c = 3, ta chứng minh p √ − 6bc + b2 + c2 ≥ b2 + (c + a)2 + (b − c)2 và p √ − 6ca + c2 + a2 ≥ a2 + (b + c)2 + (c − a)2 Cộng hai đánh giá trên lại và áp dụng thêm bất đẳng thức Minkowski, ta có p p b2 + (c + a)2 + (b − c)2 + a2 + (b + c)2 + (c − a)2 p ≥ (a + b)2 + (a + b + 2c)2 + (a − b)2 p ≥ (a + b)2 + (a + b + 2c)2 √ = 18 + 2c2 Ngoài ra, [6c − (a + b)]2 = [6c − (3 − c)]2 = 49c2 − 42c + Kết hợp với (3), ta đưa 6(18 + 2c2 ) ≥ 5(49c2 − 42c + 9) Bất đẳng thức cuối tương đương với −233c2 + 210c + 63 = (1 − c)(233c + 23) + 40 ≥ đúng vì c ≤ Bước giải hoàn toàn Bước Để hoàn thành bước dồn biến, ta cần chứng minh r √ (a − b)2 2 2 − 6ab + a + b + 36 − 12(a + b)c + (a + b) + 4c − p p 2 ≥ − (a + b) + 36 − 12(a + b)c + (a + b) + 4c 139 (140) PHƯƠNG PHÁP DỒN BIẾN Thật vậy, bất đẳng thức trên tương đương với p √ − 6ab + a2 + b2 − − (a + b)2 r p (a − b)2 ≥ 36 − 12(a + b)c + (a + b)2 + 4c2 − 36 − 12(a + b)c + (a + b)2 + 4c2 − Tiếp tục nhân liên hợp và phân tích tổng bình phương, ta cần chứng minh r p (a − b)2 10 36 − 12(a + b)c + (a + b)2 + 4c2 + 10 36 − 12(a + b)c + (a + b)2 + 4c2 − p √ 2 ≥ − 6ab + a + b + − (a + b) (4) (i) Ta chứng minh p √ 10 36 − 12(a + b)c + (a + b)2 + 4c2 ≥ − 6ab + a2 + b2 Thật vậy, từ a + b + c = và c = min{a,b,c}, suy a ≥ c, b ≥ c, ≤ c ≤ và a2 + b2 − 6ab = (a + b)2 − 2ab − 6ab = (a + b)2 − 8ab ≤ (a + b)2 − 8c2 Suy √ − 6ab + a2 + b2 ≤ p + (a + b)2 − 8c2 Do đó, ta cần chứng minh p p 10 36 − 12(a + b)c + (a + b)2 + 4c2 ≥ + (a + b)2 − 8c2 Từ a + b + c = 3, ta có p p 10 36 − 12(3 − c)c + (3 − c)2 + 4c2 ≥ + (3 − c)2 − 8c2 Khai triển và thu gọn, ta 1707c2 + 4194(1 − c) + 288 ≥ Bất đẳng thức cuối cùng là đúng vì ≤ c ≤ (ii) Tiếp theo, ta chứng minh r 10 36 − 12(a + b)c + (a + b)2 + 4c2 − Ta có: r 10 36 − 12(a + b)c + (a + b)2 + 4c2 (a − b)2 p ≥ − (a + b)2 (a − b)2 p − = 3600 − 1200(a + b)c + 80(a + b)2 + 400c2 + 80ab Ta cần chứng minh p p 3600 − 1200(a + b)c + 80(a + b)2 + 400c2 + 80ab ≥ − (a + b)2 Bình phương hai vế và biến đổi, ta thu 4086(1 − c) + 234 + 1681c2 + 80ab ≥ Bất đẳng thức cuối là đúng vì ≤ c ≤ và a,b là các số thực dương Do đó, bất đẳng thức (4) là chứng minh Bước giải hoàn toàn Bước Để hoàn thành lời giải, ta cần chứng minh   p p √ a+b a+b f , ,c = − (a + b)2 + 36 − 12(a + b)c + (a + b)2 + 4c2 ≥ 2 140 (141) PHƯƠNG PHÁP DỒN BIẾN Từ a + b + c = 3, ta có p p √ − (3 − c)2 + 36 − 12(3 − c)c + (3 − c)2 + 4c2 ≥ Suy √ √ √ 6c − c2 + 17c2 − 42c + 45 ≥ Lại bình phương hai vế và phân tích, ta thu c(c − 1)2 (10 − 3c) ≥ Bất đẳng thức cuối là đúng vì ≤ c ≤ 3 Dấu xảy và a = b = c = a = b = , c = b = 0, c = a = 2 Tương tự bài toán trên, đánh giá bước chính là chìa khóa để giải bài toán này Câu 6.4 Bài toán này ngoài trường hợp dấu xảy a = b = c = 1, còn xảy a = b = 0, c = các hoán vị số (0; 0; 3) Không tính tổng quát giả sử c = max{a,b,c}, suy a ≤ c, b ≤ c Từ a + b + c = 3, suy ≤ c ≤ kéo theo ≤ a + b ≤ Bất đẳng thức viết lại sau p p p √ 12 + 22ab + 7(a2 + b2 ) + 12 + 22bc + 7(b2 + c2 ) + 12 + 22ca + 7(c2 + a2 ) ≤ 12 Ta đặt p p p 12 + 22ab + 7(a2 + b2 ) + 12 + 22bc + 7(b2 + c2 ) + 12 + 22ca + 7(c2 + a2 )   a+b a+b ; ;c Ta chứng minh f (a; b; c) ≤ f 2 Bước Ta chứng minh p p 12 + 22bc + 7(b2 + c2 ) + 12 + 22ca + 7(c2 + a2 ) p ≤ 48 + 44(a + b)c + 7(a + b)2 + 28c2 + 2(a − b)2 f (a; b; c) = Bình phương hai vế và biến đổi, ta thu i2 hp p 2 2 12 + 22bc + 7(b + c ) − 12 + 22ca + 7(c + a ) 5(a − b) ≤ hay hp i2 p 2 2 12 + 22bc + 7(b + c ) + 12 + 22ca + 7(c + a ) ≤ [7(a + b) + 22c]2 (5) Áp dụng bất đẳng thức Cauchy-Schwar, ta có hp i2 p 2 2 12 + 22bc + 7(b + c ) + 12 + 22ca + 7(c + a )   ≤(1 + 1) 12 + 22bc + 7(b2 + c2 ) + 12 + 22ca + 7(c2 + a2 ) =48 + 44(a + b)c + 28c2 + 14(a2 + b2 ) Kết hợp với (5), ta cần chứng minh   48 + 44(a + b)c + 28c2 + 14(a2 + b2 ) ≤ [7(a + b) + 22c]2 Tương đương với (a + b)[88c − 21(a + b)] + 240(c2 − 1) + 104c2 + 140ab ≥ 141 (142) PHƯƠNG PHÁP DỒN BIẾN Bất đẳng thức cuối cùng đúng vì ≤ a + b ≤ và ≤ c ≤ Bước giải hoàn toàn Bước Để hoàn thành bước dồn biến ta cần chứng minh p p 12 + 22ab + 7(a2 + b2 ) + 48 + 44(a + b)c + 7(a + b)2 + 28c2 + 2(a − b)2 p p ≤ 12 + 9(a + b)2 + 48 + 44(a + b)c + 7(a + b)2 + 28c2 Thật bất đẳng thức trên tương đương với p p 48 + 44(a + b)c + 7(a + b)2 + 28c2 + 2(a − b)2 − 48 + 44(a + b)c + 7(a + b)2 + 28c2 p p ≤ 12 + 9(a + b)2 − 12 + 22ab + 7(a2 + b2 ).(6) Bằng cách nhân liên hợp và phân tích tổng bình phương, ta đưa chứng minh p p 48 + 44(a + b)c + 7(a + b)2 + 28c2 + 2(a − b)2 + 48 + 44(a + b)c + 7(a + b)2 + 28c2 p p ≥ 12 + 9(a + b)2 + 12 + 22ab + 7(a2 + b2 ).(7) Đầu tiên ta chứng minh p p 48 + 44(a + b)c + 7(a + b)2 + 28c2 + 2(a − b)2 ≥ 12 + 9(a + b)2 Bình phương hai vế và thu gọn ta thu 36 + 2(a + b)[22c − (a + b)] + 28c2 + 2(a − b)2 ≥ Bất đẳng thức cuối là đúng vì ≤ a + b ≤ và ≤ c ≤ Tiếp theo ta chứng minh p p 48 + 44(a + b)c + 7(a + b)2 + 28c2 ≥ 12 + 22ab + 7(a2 + b2 ) Bình phương hai vế và thu gọn ta thu 36 + 44(a + b)c + 28c2 − 8ab ≥ Vì c = max{a,b,c} nên 28c2 − 8ab ≥ và bất đẳng thức trên là đúng Do đó, bất đẳng thức (7) chứng minh Bước giải hoàn toàn Bước Để hoàn thành lời giải ta cần chứng minh   p p √ a+b a+b ; ; c = 12 + 9(a + b)2 + 48 + 44(a + b)c + 7(a + b)2 + 28c2 ≤ 12 f 2 Từ a + b + c = ta điều phải chứng minh trở thành √ √ √ 93 − 54c + 9c2 + 111 + 90c − 9c2 ≤ 12 Bình phương hai vế và thu gọn ta (c − 1)2 (c − 3)(c − 11) ≥ Bất đẳng thức cuối cùng đúng ≤ c ≤ Bước giải hoàn toàn hay bài toán chứng minh xong Dấu xảy và a = b = c = a = 3,b = c = b = 3,a = c = c = 3,a = b = Bước đánh giá bước là mấu chốt để ta giải bài toán 142 (143) PHƯƠNG PHÁP DỒN BIẾN Câu 6.5 Bài toán này ngoài trường hợp dấu  xảyra a = b = c = còn xảy 3 a = b = ,c = các hoán vị số ; ; Không tính tổng quát giả sử 2 c = min{a,b,c}, p đó ≤ c ≤ p p Đặt f (a; b; c) = 2(a2 + b2 ) + 21c + 2(b2 + c2 ) + 21a + 2(c2 + a2 ) + 21b Ta chứng minh   a+b a+b ; ;c f (a; b; c) ≥ f 2 Bước Ta chứng minh p p p 2(b2 + c2 ) + 21a + 2(c2 + a2 ) + 21b ≥ 2(a + b)2 + 8c2 + 42(a + b) − 2(a − b)2 Bình phương hai vế và khai triển ta có hp i2 p 2 2 4(a − b) ≥ 2(b + c ) + 21a − 2(c + a ) + 21b Nhân liên hợp cho bất đẳng thức trên ta đưa hp i2 p 2(b2 + c2 ) + 21a + 2(c2 + a2 ) + 21b ≥ [21 − 2(a + b)]2 (8) Áp dụng tính chất 2(x2 + y ) ≥ (x + y)2 và bất đẳng thức Minkowski, ta có p p 2(b2 + c2 ) + 21a + 2(c2 + a2 ) + 21b p p ≥ (b + c)2 + 21a + (c + a)2 + 21b q q √ √ 2 = (b + c) + ( 21a) + (c + a)2 + ( 21b)2 q √ √ ≥ (a + b + 2c)2 + ( 21a + 21b)2 Từ a + b + c = và c = min{a; b; c}, suy q √ √ (a + b + 2c)2 + ( 21a + 21b)2 q √ = (3 − c + 2c)2 + 21(a + b + ab) p √ ≥ (3 + c)2 + 21(3 − c + 2c) = c2 + 27c + 72 Ngoài [21 − 2(a + b)]2 = [21 − 2(3 − c)]2 = 225 + 60c + 4c2 ,kết hợp với (8) ta cần có 4(c2 + 27c + 72) ≥ 225 + 60c + 4c2 ⇔ 16c + 21 ≥ Bất đẳng thức này đúng vì ≤ c ≤ Bước giải hoàn toàn Bước Để hoàn thành bước dồn biến, ta cần chứng minh p p 2(a2 + b2 ) + 21c + 2(a + b)2 + 8c2 + 42(a + b) − 2(a − b)2 p p ≥ (a + b)2 + 21c + 2(a + b)2 + 8c2 + 42(a + b) Bất đẳng thức tương đương với p p 2(a + b)2 + 8c2 + 42(a + b) − 2(a + b)2 + 8c2 + 42(a + b) − 2(a − b)2 p p ≤ 2(a2 + b2 ) + 21c − (a + b)2 + 21c 143 (144) PHƯƠNG PHÁP DỒN BIẾN Nhân lượng liên hợp ta p p 2(a + b)2 + 8c2 + 42(a + b) + 2(a + b)2 + 8c2 + 42(a + b) − 2(a − b)2 p p ≥2 2(a2 + b2 ) + 21c + (a + b)2 + 21c.(9) Đầu tiên ta chứng minh p p 2(a + b)2 + 8c2 + 42(a + b) ≥ 2(a2 + b2 ) + 21c Thật ta có c = min{a,b,c} suy 2(a2 + b2 ) = 2(a + b)2 − 4ab ≤ 2(a + b)2 − 4c2 Khi đó ta thu p p 2(a2 + b2 ) + 21c ≥ 2(a + b)2 − 4c2 + 21c Do đó ta cần chứng minh p p 2(a + b)2 + 8c2 + 42(a + b) ≥ 2(a + b)2 − 4c2 + 21c Từ a + b + c = 3, bình phương hai vế và biến đổi ta cần chứng minh 2(3 − c)2 + 8c2 + 42(3 − c) ≥ 8(3 − c)2 − 16c2 + 84c ⇔ (1 − c)(4 − c) ≥ Bất đẳng thức này đúng ≤ c ≤ Tiếp theo ta chứng minh p p 2(a + b)2 + 8c2 + 42(a + b) − 2(a − b)2 ≥ (a + b)2 + 21c p p Để ý 2(a + b)2 + 8c2 + 42(a + b) − 2(a − b)2 = 8c2 + 42(a + b) + 8ab nên ta cần chứng minh p p 8c2 + 42(a + b) + 8ab ≥ (a + b)2 + 21c Từ a + b + c = nên điều này tương đương với 8c2 + 42(3 − c) + 8ab ≥ 4[(3 − c)2 + 21c] Từ c = min{a,b,c} ta có 8ab ≥ 8c2 , đó ta cần chứng minh 8c2 + 42(3 − c) + 8c2 ≥ 4[(3 − c)2 + 21c] ⇔ (1 − c)(15 − 2c) ≥ Bất đẳng thức cuối cùng đúng ≤ c ≤ Đến đây thì bất đẳng thức (9) đã chứng minh hay bước giải hoàn toàn Bước Ta cần chứng minh   p p a+b a+b f ; ; c = (a + b)2 + 21c + 2(a + b)2 + 8c2 + 42(a + b) ≥ 15 2 Từ a + b + c = thì ta cần chứng minh p p (3 − c)2 + 21c + 2(3 − c)2 + 8c2 + 42(3 − c) ≥ 15 Bình phương khử ta đưa c(c − 1)2 (40 − 3c) ≥ Bất đẳng thức cuối đúng vì ≤ c ≤ Bước giải hoàn toàn Vậy bài giải Dấu xảy và a = b = c = a = b = ,c = b = c = a = c = ,b = 144 toán đã ,a = (145) PHƯƠNG PHÁP DỒN BIẾN Câu 6.6 Bài toán này ngoài trường hợp dấu xảy a = b = c = Còn xảy a = b = 32 ,c = các hoán vị 23 , 32 ,0 Không tính tổng quát giả sử c = {a,b,c} Từ đây suy 0√≤ c ≤ p √ √ Đặt f (a,b,c) = 3a2 − a + + 3b2 − b + + 3c2 − c + − 6(a2 + b2 + c2 ) + Bước Ta dễ chứng minh p √ √ √ 3a2 − a + + 3b2 − b + + 3c2 − c + ≤ 6(a2 + b2 − 2(a + b) + − (a + b)2 Bước Để hoàn thành bước dồn biến ta cần chứng minh p p 6(a2 + b2 − 2(a + b) + − (a + b)2 − (a2 + b2 + c2 ) + ≤ p p ≤ 3(a + b)2 − 2(a + b) + + 3(a + b)2 + 6c2 + Nhân liên hợp và phân tích tổng bình phương, ta đưa q  q 2 2 (a + b ) − (a + b) + − (a − b) + 3(a + b) − (a + b) + ≥  ≥2 p (a2 + b2 + c2 )  q 2 + + 3(a + b) + 6c + • Ta dễ chứng minh q q 3(a + b) − 2(a + b) + ≥ 3(a + b)2 + 6c2 + • Ta dễ chứng minh q p (a2 + b2 ) − 2(a + b) + − (a − b)2 ≥ (a2 + b2 + c2 ) + Bước Ta cần chứng minh q q √ 2 3(a + b) − (a + b) + + 3c − c + ≤ 3(a + b)2 + 6c2 + Từ a + b + c = ta suy p √ √ 3c2 − 16c + 25 + 3c2 − c + ≤ (c2 − 2c + 4) Bình phương hai vế và rút gọn, ta 9c(c − 1)2 (16 − 3c) ≥ Bất đẳng thức cuối đúng vì ≤ c ≤ Bước giải hoàn toàn Vậy bài toán giải Đẳng thức xảy a = b = c = a = b = 32 ,c = 0, a = 0,b = c = 23 , b = 0,c = a = 32 Câu 6.7 Bài toán này ngoài trường hợp dấu xảy a = b = c = 23 Còn xảy a = b = các hoán vị (1,1,0) Không tính tổng quát giả sử c = {a,b,c} Từ đây suy ≤ c ≤ 32 √ √ √ Đặt f (a,b,c) = a + b − 2ab + b + c − 2bc + c + a − 2ca Bước Ta chứng minh r √ √ b + c − 2bc + c + a − 2ca ≥ 2(a + b) + 4c − 4(a + b)c − (a − b)2 145 (146) PHƯƠNG PHÁP DỒN BIẾN Bước Để hoàn thành bước dồn biến, ta cần chứng minh r √ a + b − 2ab + 2(a + b) + 4c − 4(a + b)c − (a − b)2 ≥ r (a + b)2 p ≥ a+b+ + 2(a + b) + 4c − 4c(a + b) Nhân lượng liên hợp, ta r p 2(a + b) + 4c − 4(a + b)c − (a − b)2 ≥ r √ (a + b)2 a + b − 2ab + a + b + 2(a + b) + 4c − 4c(a + b) + (i) Ta chứng minh p √ 2(a + b) + 4c − 4c(a + b) ≥ a + b − 2ab (ii) Ta chứng minh r 2(a + b) + 4c − 4c(a + b) − (a − b)2 ≥ r a+b− (a + b)2 Bước Ta cần chứng minh r 2c − c2 √ + − 6c + 4c2 ≥ 2 Bình phương hai vế và rút gọn, ta c(3c − 2)2 (16 − 9c) ≥ Bất đẳng thức cuối đúng vì ≤ c ≤ 23 Bước giải hoàn toàn Vậy bài toán đã cho giải Đẳng thức xảy và a = b = c = , a = b = 1,c = 0, a = 0,b = c = 1, b = 0,c = a = Câu 6.8 Bài toán ngày ngoài trường hợp dấu  xảy a = b = c = Còn xảy 2 , , Không tính tổng quát giả sử a = b = ,c = , các hoán vị số 3 3 c = {a,b,c} Từ đây suy ≤ c ≤ Đặt f (a,b,c) = √ 2a2 − a + + √ √ 1p 2b2 − b + + 2c2 − c + − 21(a2 + b2 + c2 ) + 99 Bước Dễ dàng chứng minh √ 2a2 − a + + √ r 2b2 − b + 1+ ≥ 2(a + b)2 − 2(a + b) + + (a − b)2 16 Bước Để hoàn thành bước dồn biến, ta cần chứng minh r 1p 2(a + b)2 − 2(a + b) + + (a − b)2 − 21(a2 + b2 + c2 + 99 ≥ 16 146 (147) PHƯƠNG PHÁP DỒN BIẾN ≥ p 2(a + b)2 − 2(a + b) + − r 21c2 + 21 (a + b)2 + 99 Nhân lượng liên hợp và biến đổi ta đưa r p 21(a2 + b2 + c2 ) + 99 + r ≥8 2(a + b)2 − 2(a + b) + + 21c2 + 21 (a + b)2 + 99 ≥ p (a − b)2 + 2(a + b)2 − 2(a + b) + 16 Ta dễ dàng chứng minh các kết sau: r p 21 21c2 + (a + b)2 + 99 ≥ 2(a + b)2 − 2(a + b) + và r p 2 21(a + b + c + 99 ≥ 2(a + b)2 − 2(a + b) + + (a − b)2 16 Bước Ta cần chứng minh p √ 2(a + b)2 − 2(a + b) + + 2c2 − c + − r 21c2 + 21 (a + b)2 + 99 ≥ Từ a + b + c = 3, ta suy √ 2c2 − 10c + 16 + √ r 2c2 − c + ≥ 7c2 − 14c + 43 Bình phương hai vế và rút gọn, ta 7(3c − 5)2 (c − 1)2 ≥ Bất đẳng thức cuối cùng là đúng Bước giải hoàn toàn và kết thúc bài toán Đẳng thức xảy và a = b = c = a = b = 32 ,c = 53 a = 53 ,bc = 23 b = 35 ,c = a = 32 Câu 6.15 Chuẩn hóa ab + bc + ca = Bất đẳng thức cần chứng minh trở thành 1 + + ≥ 2 (a + b) (b + c) (c + a) 1 + + và t là số thực dương thỏa mãn 2at + t2 = Xét 2 (a + b) (b + c) (c + a)     1 1 P = f (a, b, c) − f (a,t,t) = + − + − (a + b)2 (a + c)2 (a + t)2 (b + c)2 4t2 Đặt f (a, b, c) = Ta có 2at + t2 = ab + bc + ca ⇔ a(b + c − 2t) = t2 − bc (1)  2 a+b Nếu b + c < 2t thì V T (1) < và t2 > ≥ ab nên V P (1) ≥ 0, dẫn đến mâu thuẫn với (1) Từ đó, suy b + c ≥ 2t ⇒ (b + c)2 ≥ 4t2 ⇒ (a + b)2 147 (148) PHƯƠNG PHÁP P, Q, R §4 Phương pháp p, q, r Câu 1.1 Bất đẳng thức cần chứng minh tương đương với 6p ≥ ⇔ pq + 3q − 6p ≥ ⇔ q ≥ p q p+3 √ Ta có p2 ≥ 3pr = 3p, nên q ≥ 3p Do đó, để chứng minh (1) ta chứng minh 1+ p 3p ≥ (1) 6p ⇔ p(p + 3)2 ≥ 12p2 ⇔ (p − 3)2 ≥ (đúng) p+3 Câu 1.2 Ta có q + 6r = và bất đẳng thức cần chứng minh trở thành p + 3r ≥ ⇔ 2p ≥ q + (2) Theo BĐT Schur ta có 9r ≥ 4pq − p3 ⇔ · 9−q 27 + 2p3 ≥ 4pq − p3 ⇔ q ≤ 8p + Do đó để chứng minh (2) ta chứng minh 2p ≥ 27 + 2p3 + ⇔ p3 − 2p2 + 9p + 18 ≤ ⇔ (p + 1)(p − 3)(p − 6) ≤ 8p + (3) p2 6p3 + ⇔ p ≥ nên (3) đúng p ≤ Nếu p > thì a + b + c + 3abc > 27  3 a+b+c 64 3 Câu 1.3 Nếu a + b + c > thì ta có a + b + c ≥ Áp dụng BĐT Schur, > ta có: a3 + b3 + c3 + 3abc ≥ ab(a + b) + bc(b + c) + ca(c + a) ⇔ a3 + b3 + c3 + 6abc ≥ (ab + bc + ca)(a + b + c) = pq = 3p Ta có = q + 6r ≤ và r≥ p(4q − p2 ) p(12 − p2 ) = 9 Ta cần chứng minh: p(12 − p2 ) ≥ 10 (p − 3)[(16 − p2 ) + 3(4 − p) + 2] ⇔ ≥ Bất đẳng thức cuối hiển nhiên đúng nên ta có đpcm Đẳng thức xảy a = b = c = 3p + Câu 1.4 Bât đẳng thức cần chứng minh viết lại sau: 3r + 12 ≥ 5q Mặt khác,theo BDT Schur,ta có: 3r ≥ 3p(4q − p2 ) = 4q − 9 Nên ta chứng minh 4q − + 12 ≥ 5q ⇔ q ≤ (đúng) 148 (1) (149) PHƯƠNG PHÁP P, Q, R Câu 1.5 Bất đẳng thức cần chứng minh tương đương với 8p + 3r ≥ 12 + 5q (1) Áp dụng BDT Schur,ta có: 3r ≥ p(2q − 3) p(4q − p2 ) = 3 p2 − Từ giả thiết p − 2q = ⇒ q = Nên ta cần chứng minh: 8p + p(p2 − 6) 5(p2 − 3) ≥ 12 + ⇔ (2p − 3)(p − 3)2 ≥ Bất đẳng thức cuối đúng nên ta có đpcm Câu 1.6 Biến đổi bất đẳng thức cần chứng minh và chuyển dạng p,q,r,ta có: 8(243 − 18p + 3r) ≤ 3(729 − 81q + 27r − r2 ) ⇔ 243 − 99q + 57r − 3r2 ≥ Theo BDT AM-GM thì = 3( a+b+c ) ≥ 3(abc)2 = r2 Theo BDT Schur,ta có: r≥ p(4q − p2 ) 4q − = ⇒ 57r ≥ 19(4q − 9) 3 Nên ta cần chứng minh: 72 − 23q − 3r2 ≥ ⇔ 3(1 − r2 ) + 23(3 − q) ≥ Vậy BDT chứng minh Câu 1.7 Đưa bất đẳng thức dạng p,q,r,từ giả thiết,ta có q + r = và lúc đó,bất đẳng thức trở thành p2 − 2q + 5r ≥ ⇔ p2 − 7q + 12 ≥ Nếu ≥ p,sử dụng BDT Schur,ta có: r≥ p(4q − p2 ) , suy p(4q − p2 ) p3 + 36 ⇔q≤ 4p + 7(p + 36) ⇒p2 − + 12 ≥ ⇔ (p − 3)(p2 − 16) ≤ 4p + √ điều này đúng vì ≥ p ≥ 3q ≥ Nếu p ≥ 4, ta có p2 ≥ 16 ≥ 4q và 4≥q+ p2 − 2q + 5r ≥ p2 − 2q ≥ p2 ≥ Vậy BDT chứng minh Đẳng thức xảy x = y = z = x = y = 2,z = và các hoán vị 149 (150) PHƯƠNG PHÁP TIẾP TUYẾN VÀ CÁT TUYẾN §5 Phương pháp tiếp tuyến và cát tuyến Câu 2.1 Vì BĐT đã cho nên ta cần chứng minh Bđt đúng với số thực dương a,b,c thỏa mãn a√ + b2 + c2 = 1, đó bđt cần chứng minh trở thành: f (a) + f (b) + f (c) ≥ 1trong 1+ đó: f (x) = √ − x với < x < Dễ thấy hàm số f (x) có f 00 (x) > ∀x ∈ (0; 1) 3 x Theo BĐT tiếp tuyến ta có :     √ 1 f (a) + f (b) + f (c) ≥ f √ (a + b + c − 3) + 3f √ 3       f √1 <0 = Do ⇒ f (a) + f (b) + f (c) ≥ 3f √  a + b + c ≤ p3(a2 + b2 + c2 ) = √3 Câu 2.2 Đặt = tan xi (i = 1,2, ,n) ⇒ > i = 1,2, ,n và n P ≤ n i=1 Ta cần chứng minh : n Y √ ≤ 2n + a2i p i=1 x Xét hàm số f (x) = √ , x > có + x2 f (x) = q (1 + Do đó ⇒ f 00 (x) < ∀x > x2 )3 1 f (x) ≤ f (1)(x − 1) + f (1) = √ (x − 1) + √ = √ (x + 1) 2 23 Suy  n Y i=1 n P n (ai + 1)  n n  Y i=1   ≤ √2 = √1 p (a = f (a i) ≤ √ i + 1) ≤ √  n 8n i=1 8n  8n 2n + a2i i=1 n Y Đẳng thức xảy a1 = a2 = · · · = an = hay tan x1 = tan x2 = · · · = tan xn = ⇔ x1 = π x2 = · · · = xn = Câu 2.3 Ta có : bc ≤ ( b+c 1−a a+c 1−b b+a 1−c ) =( ) ; ca ≤ ( ) =( ) ; ab ≤ ( ) =( ) 2 2 2 nên a b c 4a 4b 4c + + ≥ + + = f (a) + f (b) + f (c) + bc + ac + ab a − 2a + b − 2b + c − 2c + 4x Ta thấy đẳng thức xảy a = b = c = và tiếp tuyến đồ thị hàm số f (x) = x − 2x + 99x − điểm có hoành độ x = là :y = , nên ta xét 100 4x 99x − (3x − 1)2 (15 − 11x) − = ≥ ∀x ∈ (0; 1) x2 − 2x + 100 100(x2 − 2x + 5) 150 (151) PHƯƠNG PHÁP TIẾP TUYẾN VÀ CÁT TUYẾN Suy 4a 4b 4c 99(a + b + c) − 9 + + ≥ = − 2a + b − 2b + c − 2c + 100 10 Bài toán chứng minh a2 Câu 2.4 Vì Bđt cần chứng minh là nên ta cần chứng minh Bđt đúng với số thực dương a,b,c thỏa mãn a + b + c = Khi đó Bđt đã cho trở thành: (1 − 2a)2 (1 − 2b)2 (1 − 2c)2 + + ≥ 2 (1 − a) + a2 (1 − b) + b2 (1 − c) + c2 4a2 − 4a + 4b2 − 4b + 4c2 − 4c + + + ≥ ⇔ 2a − 2a + 2b − 2b + 2c − 2c + 1 27 ⇔ + + ≤ 2a − 2a + 2b − 2b + 2c − 2c + 27 ⇔ f (a) + f (b) + f (c) ≤ Trong đó f (x) = với x ∈ (0; 1) 2x − 2x + 1 54x + 27 Tiếp tuyến đồ thị hàm số y = f (x) điểm có hoành độ x = là y = 25 Ta có 2(54x3 − 27x2 + 1) 2(3x − 1)2 (6x + 1) 54x + 27 − f (x) = = ≥ ∀x ∈ (0; 1) 25 25(2x2 − 2x + 1) 25(2x2 − 2x + 1) Suy f (a) + f (b) + f (c) ≤ 54(a + b + c) + 81 27 = (đpcm) 25 Câu 2.5 Phân tích Với điều kiện a + b + c = ta chọn điểm rơi a = b = c = = x0 và     1 11 16 đặt f (a) = a3 + 5a2 Khi đó f (a) = 3a2 + 10a, f = ,f = 3 27 Ta chứng minh   11 16 a− + a + 5a ≥ 3 27 11 17 ⇔ a3 + 5a2 − a + ≥0 27  2   17 ⇔ a− a+ ≥ (BĐT cuối luôn đúng với điều kiện a > −1) 3 Chứng minh tương tự với các biểu thức còn lại:  11 b− b + 5b ≥  11 c + 5c ≥ c− 3  +  + 16 , 27 16 27 Cộng vế với vế, ta S≥ 16 16 11 (a + b + c − 1) + = 9 16 Khi a = b = c = thì S = 16 Vậy S = a = b = c = 151 (152) PHƯƠNG PHÁP TIẾP TUYẾN VÀ CÁT TUYẾN a Câu 2.6 Phân tích Chọn điểm rơi a = b = c = −2 và đặt f (a) = Tính a +a+1 f (−2) = − , f (−2) = − Ghép vào công thức (∗∗) 3 Ta chứng minh a ≤ − (a + 2) − +a+1 3 ⇔ −a − 5a − 8a − ≥ ⇔ (a + 2)2 (a + 1) ≤ (luôn đúng với a < −1) a2 Chứng minh tương tự với các biến b, c, ta có b ≤ − (b + 2) − +b+1 c ≤ − (c + 2) − c +c+1 b2 , Cộng vế với vế S ≤ − (a + b + c) − = − = −2 Với a = b = c = −2 thì S = −2 Vậy max S = −2 a = b = c = −2 Câu 2.7 Phân tích Lần này biến không còn đối xứng (chỉ là hoán vị vòng quanh) và không nhóm dạng phân ly biến Ta nhóm riêng (a3 +2a2 b) để đánh giá và coi b là tham số (hoặc có thể nhóm a3 + 2c2 a) Từ điều kiện ta có thể nhẩm dấu xảy a = b = c = và đặt f (a) = a3 + 2a2 b ⇒ f (a) = 3a2 + 4ab, f (1) = + 4b, f (1) = + 2b Ta có chứng minh a3 + 2a2 b ≥ (3 + 4b)(a − 1) + (1 + 2b) ⇔ (a3 − 1) + (2a2 b − 2b) − (3 + 4b)(a − 1) ≥ ⇔ (a − 1)2 (a + 2b + 2) ≥ (BĐT cuối luôn đúng với điều kiện ban đầu) Chứng minh tương tự, ta có b3 + 2b2 c ≥ (3 + 4c)(b − 1) + (1 + 2c), c3 + 2c2 a ≥ (3 + 4a)(c − 1) + (1 + 2a) Cộng vế với vế S ≥ 4(ab + bc + ca) + a + b + c − ≥ 15 − = Với a = b = c = thì S = Vậy S = a = b = c = Câu 2.8 Phân tích Từ điều kiện đề bài nhẩm a = b = c = Đặt f (a) = 2a3 − 3a2 b suy f (1) = − 6b, f (1) = − 3b Ta có chứng minh 2a3 − 3a2 b ≥ (6 − 6b)(a − 1) + − 3b ⇔ (a − 1)2 (2a − 3b + 4) ≥ (BĐT cuối luôn đúng điều kiện xác định) Chứng minh tương tự, ta có 2b3 − 3b2 c ≥ (6 − 6c)(b − 1) + − 3c, 2c3 − 3c2 a ≥ (6 − 6a)(c − 1) + − 3a Cộng vế với vế ta S ≥ 9(a + b + c) − 6(ab + bc + ac) − 12 ≥ · − 12 = −3 Với a = b = c = thì S = −3 Vậy S = −3 a = b = c = 152 (153) PHƯƠNG PHÁP TIẾP TUYẾN VÀ CÁT TUYẾN Câu 2.9 Phân tích Đặt f (a) = 5a3 + 2a2 b, tính f (1) = 15 + 4b, f (1) = + 2b Ta có 5a3 + 2a2 b ≥ (15 + 4b)(a − 1) + + 2b ⇔ 5(a − 1)(a2 + a + 1) + 2b(a − 1)(a + 1) − (15 + 4b)(a − 1) ≥ ⇔ (a − 1)[5a2 + 5a − 10 + 2b(a − 1)] ≥ ⇔ (a − 1)2 (5a + 2b + 10) ≥ (BĐT cuối luôn đúng vì điều kiện xác định) Chứng minh tươngp tự cộng vế với vế S ≥ 13(a + b + c) + 4(ab + bc + ca) − 30 Lại có a + b + c ≥ 3(ab + bc + ca) = Suy S ≥ 13 · + · − 30 = 21 Với a = b = c = thì S = 21 Vậy S = 21 a = b = c = Câu 2.10 Phân tích Đặt f (a) = b , tính f (1) = −2 + b, f (1) = − b − a2 a Ta có b − ≥ (−2 + b)(a − 1) + − b a a    1+a a−1 ⇔ (1 − a) +b + (−2 + b)(1 − a) ≥ a2 a   1+a b ⇔ (1 − a) −2+b− ≥0 a2 a   2a + b − ≥ (BĐT cuối luôn đúng vì điều kiện xác định) ⇔ (1 − a) a2 a Chứng minh tương tự cộng vế với vế T ≥ −4(a + b + c) + ab + bc + ca + = Với a = b = c = thì T = Vậy T = a = b = c = Câu 2.11 Phân tích Đặt f (a) = a5 + 3a4 b, tính f (1) = + 12b, f (1) = + 3b Ta có a5 + 3a4 b ≥ (5 + 12b)(a − 1) + + 3b ⇔ (a − 1)(a4 + a3 + a2 + a + 1) + 3b(a − 1)(a3 + a2 + a + 1) − (5 + 12b)(a − 1) ≥ ⇔ (a − 1)[3b(a3 + a2 + a − 3) + a4 + a3 + a2 + a − 4] ≥ ⇔ (a − 1)2 [3b(a2 + 2a + 3) + a3 + 2a2 + 3a + 4] ≥ (BĐT cuối luôn đúng vì điều kiện xác định) Chứng minh tương tự cộng vế với vế A ≥ 12(ab+bc+ca)−4(a+b+c)−12 ≥ 24−12 = 12 Với a = b = c = thì A = 12 Vậy A = 12 a = b = c = √ Câu 2.12 Phân tích Đặt f (a) = a2 + b 3a + 1, tính f (1) = + b, f (1) = + 2b Ghép vào công thức Ta có   √ a + b 3a + ≥ + b (a − 1) + + 2b   3b ⇔ (a − 1) a + + √ −2− b ≥0 3a + +   9b ⇔ (a − 1)2 − √ ≥ (BĐT cuối luôn đúng vì điều kiện xác định) 4( 3a + + 2)2 153 (154) PHƯƠNG PHÁP TIẾP TUYẾN VÀ CÁT TUYẾN 13 Chứng minh tương tự cộng vế với vế B ≥ (ab + bc + ca) + (a + b + c) − ≥ 4 Với a = b = c = thì B = Vậy B = a = b = c =  π Câu 2.13 Xét hàm số f (x) = tan x, x ∈ 0; , có f (x) = + tan2 x, suy f 00 (x) =  π tan x(1 + tan2 x) > 0, ∀x ∈ 0; Áp dụng BĐT tiếp tuyến với ∆M N P nhọn, ta có : f (A) ≥ f (M )(A − M ) + f (M ) = (A − M ) + tan M, cos2 M suy cos2 M.f (A) ≥ sin 2M + A − M Tương tự : cos2 N.f (B) ≥ 1 sin 2N + B − N ; cos2 P.f (C) ≥ sin 2P + C − P 2 Suy cos2 M.f (A) + cos2 N.f (B) + cos2 P.f (C) ≥ sin 2M + sin 2N + sin 2P Ta chọn các góc M, N, P cho cos M = k > 0; cos N = √ 2k; cos P = √ 3k Vì M, N, P là ba góc tam giác nên ta có đẳng thức : cos2 M + cos2 N + cos2 P + cos M cos N cos P = 1, √ √ √ nên ta có (1 + + 3)k + 6k = 1, đó k là nghiệm dương phương trình : √ √ √ 6x3 + (1 + + 3)x − = (1) Suy √ √ sin 2M =p − cos2 M · cos M = 2k p 1−k ; sin 2N = 2k 2(1 − 2k ); sin 2P = 2k 3(1 − 3k ) Dẫn tới p p √ − k + 2(1 − 2k ) + 3(1 − 3k ) sin 2M + sin 2N + sin 2P F ≥ = 2k k p p √ − k + 2(1 − 2k ) + 3(1 − 3k ) Vậy F = đạt A = M ; B = N ; C = P với k M, N, P là ba góc tam giác nhọn xác định √ √ cos M = k > 0; cos N = 2k; cos P = 3k, đó k là nghiệm dương (1) 154 (155) PHƯƠNG PHÁP TIẾP TUYẾN VÀ CÁT TUYẾN √ √ Câu 2.14 Ta có các hàm số f (t) = t3 ; g(t) = + t2 ; h(t) = + t4 , t ∈ (0; 1) là hàm số có đạo hàm cấp hai dương trên khoảng (0; 1) Nên với a,b,c > thỏa a + b + c = áp dụng BĐT tiếp tuyến, ta có: f (x) ≥ f (a)(x − a) + f (a), h(y) ≥ h0 (b)(y − b) + h(b), g(z) ≥ g (c)(z − c) + g(c) Ta chọn a,b,c cho r   k  3a = k     a=       b   √  =k k + b2 f (a) = g (b) = h0 (c) = k ⇔ ⇔ b= √ − k2     c3 √     q =k   k    (1 + c4 )3  p c =  √ 1−k 3k (1) Do a + b + c = nên ta có r √ k k k +p +√ √ = − k2 1−k 3k Dễ thấy phương trình (2) luôn có nghiệm khoảng (0; 1) Suy √ 1 k 3k +√ +p P = f (x) + g(y) + h(z) ≥ f (a) + h(b) + g(c) = √ − k2 1−k 3k Đẳng thức xảy √ ⇔ x = a; y = b; z = c k 3k 1 Vậy P = +√ +p √ với k là nghiệm nằm (0; 1) (2) − k2 1−k 3k 155 (2) (156) Chương Một số chuyên đề §1 Ứng dụng kiện có nghiệm phương trình bậc ba Câu 1.1 Đặt n = ab + bc + ca, p = −abc Suy a, b, c là ba nghiệm phương trình : x3 − mx + n = (4) Ta có: 27 p2 ≤ − n3 ⇒ n3 ≤ − p2 27 Do đó: 27 13p + 2p − + 2n ≤ 13p + 2p − − p2 = − 2  2 p − ≤ Suy ra: 13p2 + 2p − ≤ −2n3 ⇔ 13a2 b2 c2 − 2ab − ≤ −2 (ab + bc + ca)3 Mà: (a + b + c)2 = ⇒ ab + bc + ca = −  a + b2 + c2 , dẫn tới: 3 13a2 b2 c2 − 2abc − ≤ a + b2 + c2 ⇒ P ≤ 4 ( n=2 Đẳng thức xảy ⇔ ⇔ a,b,c là ba nghiệm phương trình m=3 x3 − 3x + = ⇔ (x − 1)2 (x + 2) = ⇔ x = 1,x = −2 Vậy max P = đạt (a,b,c) = (1,1, − 2) và các hoán vị Câu 1.2 Đặt n = ab + bc + ca, p = −abc Ta có: p2 ≤ − 27 27 n ⇒ −n3 ≥ p2 ⇒ n3 ≥ p2 27 4 Vì a + b + c = ⇒ a2 + b2 + c2 = −2(ab + bc + ca) = −2n ⇒ n ≤ Do đó:   P = −32n5 − 32np2 − |p| = 32 (−n)5 + (−n)p2 − |p|  ≥ 64 n3 |p| − |p| ≥ 54|p|3 − |p| 156 (157) ỨNG DỤNG ĐỀU KIỆN CÓ NGHIỆM CỦA PHƯƠNG TRÌNH BẬC BA Xét hàm số f (t) = 54t3 − t,t ≥ ta có: √ f (t) = 162t − 1,f (t) = ⇔ t = 18 √ ! √ 2 Lập bảng biến thiên ta có minf (t) = f =− t≥0 18 27  √  √   p = 18 Suy P ≥ − Đẳng thức xảy hay a,b,c là nghiệm phương trình  27  n = − √ 24 ! √ √ ! √ 2 2 =0⇔ t− p √ =0⇔t= p √ t −√ t+ t+ √ ,t = − √ 3 3 18 24 9 6 √ √ 2 Đạt a = b = p √ ,c = − √ và các hoán vị Vậy P = − 3 27 Câu 1.3 Đặt m = −(a + b + c), n = ab + bc + ca, p = −abc Từ giả thiết ta suy ra: (a + b + c)2 = (ab + bc + ca) ⇔ n = m2 Suy m3 |m3 | ≤ 4 ⇔ 108p − m ≤ m3 ⇔ p(54p − m3 ) ≤ ⇔ pm3 ≥ 54p2 27p − Do đó: r 1 1 P = pm3 + ≥ 54p2 + = 27p2 + 27p2 + ≥ 3 27p2 27p2 = 27 (đpcm) p p p p     m2 √ p =     n=       q √ Đẳng thức xảy ⇔ 27p2 = , chẳng hạn ta chọn m = 18 hay a,b,c là nghiệm     √ p4       972  n = 54p = m phương trình: ! p √ √ √ !2 q 3 √ 972 18 t+ √ =0⇔ t+ t+ √ = t3 + 18 3t2 + 3 p √ √ 3 18 Vậy P = 27 đạt a = b = − ,c = √ và các hoán vị Câu 1.4 Đặt m = −(a + b + c), n = ab + bc + ca, p = −abc Từ giả thiết ta suy ra: (a + b + c)2 = (ab + bc + ca) + ⇒ m2 = 3n + Mặt khác : q 27p + 2m − 9mn ≤ (m2 − 3n)3 157 (158) ỨNG DỤNG ĐỀU KIỆN CÓ NGHIỆM CỦA PHƯƠNG TRÌNH BẬC BA Suy |27p + 2m (3n + 4) − 9mn| ≤ 18 ⇔ |27p − 3mn + 8m| ≤ 16 8m − 16 ⇒ mn − 9p ≥ Mặt khác: P = 18 (ab + bc + ca)2 + 48 (ab + bc + ca) − (ab + bc + ca) (a + b + c) + 9abc = [3 (ab + bc + ca) + 4]2 − (ab + bc + ca) (a + b + c) + 9abc − 16 = (a + b + c)2 − (ab + bc + ca) (a + b + c) + 9abc − 16  8m − 16 − 16 = 6m4 + 8m − 64 = 2m4 + mn − 9p − 16 ≥ 2m4 + 3 Xét hàm số f (m) = 6m4 + 8m − 64, ta có: f (m) = 24m3 + ⇒ f (m) = ⇔ m = − √ 3 Suy  f (m) ≥ f Nên P ≥ −  −√ 3  = −√ − 64 3  √ + 64 3    m = −√         1 √ − , suy a,b,c là nghiệm phương trình Đẳng thức xảy và n = 39        47   √ + p = 33    1 47 √ −4 t+ √ t2 + + = : t3 − √ 3 3 9 3  √ + 64 Vậy P = − 33 Câu 1.5 Chuẩn hoá abc = ⇒ a + b + c = Đặt n = ab + bc + ca, suy q |18n − 91| ≤ (16 − 3n)3 ⇔ 3n3 − 12n2 − 108n + 465 ≤ √ −1 + 5 ⇔ (n − 5)(3n + 3n − 93) ≤ ⇔ ≤ n ≤ 2 Mặt khác: a4 + b4 + c4 = a2 + b2 + c2 2  − a2 b + b2 c2 + c2 a2  = (16 − 2n)2 − n2 − 16 = 2n2 − 64n + 288 Nên   1 a4 + b4 + c4 = n2 − 32n + 144 256 " 128√ # 5 − Vì hàm f (n) = n2 − 32n + 144 nghịch biến trên 5; nên ta suy ! √ √ 5−1 383 − 165 max P = f (5) = và P = f = 128 128 128 2 P = 158 (159) BÀI TOÁN TÌM HẰNG SỐ TỐT NHẤT §2 Bài toán tìm số tốt Câu 2.1 Vì bất đẳng thức đúng với giá trị a,b,cnên phải đúng với a = b = c = ⇒ k ≤ Ta chứng minh k = là giá trị lớn Xét k = bất đẳng thức trở thành a4 + b4 + c4 + abc (a + b + c) ≥ (ab + bc + ca)2 , hay   a4 + b4 + c4 ≥ a2 b2 + b2 c2 + c2 a2 + abc (a + b + c) (1) Áp dụng bđt AM – GM ta có    a4 + b4 + b4 + c4 + b4 + c4 ≥ 2a2 b2 + 2b2 c2 + 2c2 a2 Suy   a4 + b4 + c4 ≥ a2 b2 + b2 c2 + c2 a2 (2) Mặt khác a2 b2 + b2 c2 + c2 a2 − abc (a + b + c) = 1 (ab − bc)2 + (bc − ca)2 + (ca − ab)2 ≥ 2 (3) Từ (2) và (3) suy (1) chứng minh Vậy số k lớn k = Câu 2.2 Không tính tổng quát, ta giả sử a = {a; b; c} Chọn b = c Từ điều kiện, ta 1 có a + 2b = ⇒ ≤ b ≤ Bất đẳng thức trở thành: a 2b + ≥ + 9b + 9ab + k(a − b)     − 2b 2b − − ⇔ + ≥0 + 9b2 + 9b (1 − 2b) + k(1 − 3b)2 (3b + 5) (1 − 3b) (3b − 1) (6b + 1) − k(3b − 1)2  ≥0 +  (1 + 9b2 ) 9b − 18b2 + + k(1 − 3b)2 6b + − k (3b − 1) 3b + ⇔ ≥ (1 + 9b2 ) 9b − 18b + + k(1 − 3b) 54b3 + 27b2 + 12b − ⇔ ≥k 9b2 + 4b − 18b2 + 15b + ⇔ ≥k 9b2 + 4b − 7b + ≥ k − ⇔ 9b + 4b −   7b + 1 Ta xét hàm số f (b) = trên ; 9b + 4b −     1 Ta dễ dàng thấy f nghịch biến trên đoạn ; nên suy f (b) ≥ f = 2 ⇔ 159 (160) BÀI TOÁN TÌM HẰNG SỐ TỐT NHẤT Do đó k − ≤ ⇔ k ≤ Ta chứng minh k = là giá trị lớn cần tìm Tức là ta cần chứng minh: b c a + + ≥ + 9bc + 4(b − c) + 9ca + 4(c − a) + 9ab + 4(a − b) (1) Áp dụng Bất đẳng thức Cauchy – Schwarz, ta có: (a + b + c)2 (a + b + c) + 27abc + 4a(b − c)2 + 4b(c − a)2 + 4c(a − b)2 = + 3abc + 4ab (a + b) + 4bc (b + c) + 4ca (c + a) VT ≥ Do đó để chứng minh (1) ta cần chứng minh: ≥ 3abc + 4ab (a + b) + 4bc (b + c) + 4ca (c + a) ⇔ (a + b + c)3 ≥ 3abc + 4ab (a + b) + 4bc (b + c) + 4ca (c + a) ⇔ a3 + b3 + c3 + 3abc ≥ ab (a + b) + bc (b + c) + ca (c + a) (điều này đúng, đây là bất đẳng thức Schur) Do đó (1) đúng với a, b, c không âm và thỏa a + b + c = và với kmax = Vậy k = là số cần tìm a Câu 2.3 Thử chọn b = c xét hàm theo biến , ta không dẫn đến kết cần tìm Như b ta thử chọn biến 0, đây là biến tiến dần đến Trong bất đẳng thức, ta cho c → 0, đó a3 + b3 ≥ M ab2 Nhưng lại theo bất đẳng thức AM – GM, ta có b3 b3 a3 + b3 = a3 + + ≥ √ · ab2 2 3 Như ta thấy M ≤ √ Do đó ta chứng minh M = √ là giá trị lớn cần tìm sau: 3 4 Không giảm tính tổng quát, giả sử c = {a,b,c} Đặt a = u + c, b = v + c với u, v ≥ Ta chứng minh: (u + c)3 + (v + c)3 + c3 − 3c (u + c) (v + c)   ≥ M (u + c) (v + c)2 + (v + c) c2 + c(u + c)2 − 3c (u + c) (v + c)  ⇔ (3 − M ) u2 − uv + v + u3 + v − M uv ≥ Bất đẳng thức cuối cùng luôn đúng vì, − M > 0, u2 − uv + v ≥ uv ≥ và u3 + v = u3 + v3 v3 + ≥ √ uv = M uv 2 Từ đó ta hoàn tất phần chứng minh Vậy giá trị lớn cần tìm là: M = √ Câu 2.4 Vì tính bất đẳng thức nên ta chuẩn hóa xyz = Từ đó ta biến đổi: a b c x = , y = , z = Khi đó bất đẳng thức trở thành: b c a   a2 b2 c2 a b c + + + 3k ≥ (k + 1) + + bc ca ab b c a  ⇔ a3 + b3 + c3 + 3k · abc ≥ (k + 1) ab2 + bc2 + ca2  ⇔ a3 + b3 + c3 − 3abc ≥ (k + 1) ab2 + bc2 + ca2 − 3abc 160 (161) BÀI TOÁN TÌM HẰNG SỐ TỐT NHẤT Đây là bất đẳng thức bài trên 3 ⇔ k = −1 + √ Vậy k + = √ 3 4 Câu 2.5 Ta chọn b = c thì giả thiết trở thành: a + 2b = b2 + 2ab ⇒ a = Vì a > nên 2b − b2 2b − 2b − b2 > ⇔ < b ≤ 2b − Bất đẳng thức trở thành:  + ≥ k (a + 2b + 1) (a + 2b) 2b a + b       1  2b − b2 2b − b ≥k ⇒ + 2b  + 2b +  2b + 2b − b2  2b − 2b − +b 2b − 3b2 − b 2k ⇔ ≥ 3b + 5b + b −   3b2 − b trên ;2 Xét hàm số f (b) = 3b + 5b2 + b − Ta có − (b2 − 1) (3b − 1)2 f (b) = ⇔ = ⇔ b = (3b3 + 5b2 + b − 1)2 Bằng cách lập bảng biến thiên ta tìm minf (b) = b = Từ đó tìm k ≤   ;2 Ta chứng minh k = là giá trị lớn cần tìm Thực ta cần chứng minh:   1 (a + b + c) + + ≥a+b+c+1 a+b b+c c+a   1 ⇔ (ab + bc + ca) + + ≥a+b+c+1 a+b b+c c+a ab bc ca ⇔ + + ≥ a+b b+c c+a Nhận thấy dấu “=” đạt biến và hai biến nên đánh giá bất đẳng thức thông thường có dấu “=” tâm không dẫn đến kết Để đảm bảo dấu “=” ta loại bỏ dấu “=” biên này cách sử dụng abc ≥ sau: Theo Cauchy - Schwarz: ab bc ca (ab + bc + ca)2 + + ≥ a+b b+c c+a ab (a + b) + bc (b + c) + ca (c + a) (ab + bc + ca)2 = (a + b + c) (ab + bc + ca) − 3abc (ab + bc + ca)2 ≥ = (a + b + c) (ab + bc + ca) Chứng minh hoàn thành Ta kết luận số k lớn cần tìm là k = 161 (162) BÀI TOÁN TÌM HẰNG SỐ TỐT NHẤT Câu 2.6 Cho b = c, ta có bất đẳng thức trở thành:     2ab + b2 (a + 2b) + −9≥k 1− a b a + 2b2 (a − b)2 2(a − b)2 ≥k· ab a + 2b2 k a2 + 2b2 ≥ ⇔ ab ⇔ Theo bất đẳng thức AM – GM, ta có: √ a2 + 2b2 ≥ 2 ab √ Suy k ≤ Ta chứng minh đây là giá trị k lớn cần tìm Sử dụng bất đẳng thức Schur – SOS sau:     √ 1 ab + bc + ca (a + b + c) + + −9≥4 1− a b c a + b2 + c2 √ a2 + b2 + c2 − ab − bc − ca (a + b + c) (ab + bc + ca) − 9abc ≥4 2· ⇔ abc a2 + b2 + c2 √ (a − b)2 + (a − c) (b − c) 2c(a − b)2 + (a + b) (a − c) (b − c) ⇔ ≥4 2· abc a2 + b2 + c2 ⇔ M · (a − b)2 + N · (a − c) (b − c) ≥ √ √ 2c a+b Trong đó: M = − ,N= − abc a2 + b2 + c2 abc a + b2 + c2 Giả sử c = max {a,b,c} thì việc hoàn tất N ≥ 0, hay √  (a + b) a2 + b2 + c2 ≥ 2abc Do tính nên ta chuẩn hóa a + b + c = Ta chứng minh kết mạnh sau: √ √ a2 + b2 + c2 ≥ c (a + b) ⇔ − 2c (a + b) ≥ c (a + b) + 2ab Và mạnh nữa: √ (a + b)2 2 √ t ⇔ − · t (1 − t) ≥ · t (1 − t) + , với t = a + b !2 √  √   √ −2 + t 3−2 2 ⇔ t − 2+ t+1≥0⇔ − ≥ 2 − · c (a + b) ≥ · c (a + b) + Điều này đúng Như chứng minh hoàn √ tất Vậy số k lớn thỏa đề bài là k = Câu 2.7 Ta chọn a = b = t, a = b = t, c = − 2t Khi đó, ta có:   k 2t4 + (3 − 2t)4 − ≥ 2t3 + (3 − 2t)3 + 3t2 (3 − 2t) − −12t3 + 45t2 − 54t + 21 ⇔k≥ 18t4 − 96t3 + 216t2 − 216t + 78 3(t − 1)2 (7 − 4t) = (t − 1)2 (18t2 − 60t + 78) − 4t = = f (t) 2 (3t − 10t + 13) 162 (163) BÀI TOÁN TÌM HẰNG SỐ TỐT NHẤT Ta có:   12 (2t2 − 7t + 3) = ⇔ t ∈ 3; f (t) = 4(3t2 − 10t + 13)2 Khoảng giá trị t là (0; 3), từ đó ta thấy minf (t) = t = (0;3) Vậy trở lại bài toán, ta cho a = b = , c = thì ta k ≥ 27 , ta chứng minh giá trị nhỏ 2 k là cách chứng minh:  a + b4 + c4 − ≥ a3 + b3 + c3 + 3abc − Đặt f (a,b,c) = V T − V P Ta chứng minh:   a+b a+b , ,c f (a,b,c) ≥ f 2 " # " # " # 2 (a + b) (a + b) (a + b) ⇔ a + b4 − − a3 + b3 − − 3c ab − ≥0 4 ⇔ ⇔ ⇔ ⇔ ⇔ (a − b)4 + 6(a − b)2 (a + b)2 (a + b) (a − b)2 (a − b)2 − + 3c · ≥0 28 4 7a2 + 7b2 + 10ab 3c − (a + b) + ≥0 28 4 a2 + b2 + 10ab − 21 (a + b) + 21c ≥  a2 + b2 + 10ab − 21 (a + b) + 21 (3 − a − b) ≥  (a + b)2 + a2 + b2 − 42 (a + b) + 63 ≥ Do (a2 + b2 ) ≥ (a + b)2 nên ta chứng minh: 6(a + b)2 − 42(a + b) + 63 ≥ Và điều này đúng ta giả sử c = max {a,b,c} vì đó a + b ≤ Vậy ta cần ra:   a+b a+b a+b f , ,c ≥ ⇔ f (t; t; − t) ≥ 0,t = 2  2 ⇔ 2t + (3 − 2t)4 − ≥ 2t3 + (3 − 2t)3 + 3t2 (3 − 2t) − Bất đẳng thức này đúng theo lập luận chặn t phía trên Vậy k = là giá trị nhỏ cần tìm Câu 2.8 Cho b = c giả thiết đã cho viết thành: a + 2b = b2 + 2ab ⇒ a = b2 − 2b ≥ ⇒ ≤ b ≤ − 2b Lưu ý vì tìm số thực k lớn nên xét k > Thay vào bất đẳng thức, ta được: b2 − 2b b2 (b2 − 2b) +k ≥k+3 − 2b −2b  ⇔ 2b (1 − 2b) + b2 − 2b + kb2 b2 − 2b ≥ k (1 − 2b) + (1 − 2b)  ⇔ (b − 1)2 ≥ k b2 − (b − 1)2 ⇔ ≥ b2 − k 2b + 163 (164) BÀI TOÁN TÌM HẰNG SỐ TỐT NHẤT Mà b2 − ≤ 22 − = suy k ≤ Ta chứng minh k = là giá trị lớn cần tìm Với giả thiết a + b + c = ab + bc + ca, ta cần chứng tỏ ab + bc + ca + abc ≥ Ta phản chứng ab + bc + ca + abc = và chứng minh a + b + c ≥ ab + bc + ca Đây là kết quen thuộc kì thi VMO 1996 Câu 2.9 Trong bất đẳng thức ban đầu, ta cho b = c, ta được: a3 + 2b3 b2 + 2ab k +k· + ≥ 2b (a + b) (a + 2b)     a3 + 2b3 1 b2 + 2ab ⇔ − ≥k· − b(a + b)2 (a + 2b)2 (a − b)2 (a − b)2 (4a + 5b) ≥ k · ⇔ 8b(a + b)2 3(a + 2b)2 8k (a + 2b)2 (4a + 5b) ⇔ ≤ b(a + b)2 Do tính nên ta có thể chọn b = Khi đó, ta có: 8k (a + 2)2 (4a + 5) ≤ (a + 1)2 (a + 2)2 (4a + 5) Ta xét hàm f (a) = , a > Ta có: (a + 1)2 √ −1 + (a + 2) (2a + 2a − 1) =0⇔a= f (a) = (do a > 0) (a + 1) √  √ ! √ 9+6 −1 + Lập bảng biến thiên ta f (a) ≥ f = + Suy ra: k ≤ √  9+6 Ta chứng minh giá trị k lớn cần tìm là Ta sử dụng kĩ thuật Schur – SOS phần chứng minh Bất đẳng thức cần chứng minh viết dạng: (a3 + b3 + c3 − 3abc) − ((a + b) (b + c) (c + a) − 8abc) a2 + b2 + c2 − ab − bc − ca ≥k· (a + b) (b + c) (c + a) 3(a + b + c)2 Ta sử dụng các khai triển:   a3 + b3 + c3 − 3abc = (a + b + c) (a − b)2 + (a − c) (b − c) (a + b) (b + c) (c + a) − 8abc = 2c (a − b)2 + (a + b) (a − c) (b − c) a2 + b2 + c2 − ab − bc − ca = (a − b)2 + (a − c) (b − c) Từ đó ta nhóm điều cần chứng minh thành: M (a − b)2 + N (a − c) (b − c) ≥ 0, đó: 8a + 8b + 2c k − (a + b) (b + c) (c + a) 3(a + b + c)2 5a + 5b + 8c k N= − (a + b) (b + c) (c + a) 3(a + b + c)2 M= 164 (165) BÀI TOÁN TÌM HẰNG SỐ TỐT NHẤT Bây giờ, không giảm tổng quát ta giả sử c = {a,b,c} nên a + b ≥ 2c Từ đó 8a + 8b + 2c ≥ 5a + 5b + 8c Từ đây ta thấy M ≥ N Hơn có (a − c) (b − c) ≥ Hơn có (a − c) (b − c) ≥ Như bất đẳng thức chứng minh hoàn tất ta N ≥ Tức là: (5a + 5b + 8c) (a + b + c)2 ≥ 8k (a + b) (b + c) (c + a) Đổi biến (a + b,b + c,c + a) → (X,Y,Z) Khi đó cần chứng minh: (X + 4Y + 4Z) (X + Y + Z)2 ≥ 32k · XY Z Ta chứng minh kết mạnh là:  √  (X + 4Y + 4Z) (X + Y + Z)2 ≥ 8k · X (Y + Z)2 = 27 + 18 X (Y + Z)2 Vì tính nên ta có thể chuẩn hóa X + Y + Z = Từ đó ta cần chứng minh:  √  √ − 3X (4 − 3X) ≥ 27 + 18 X (1 − X)2 ⇔ g (X) = 3, ≥ + X(1 − X)2 đó < X < √ Khảo sát hàm g trên khoảng (0; 1) ta g (X) ≥ + Như ta hoàn tất chứng minh √  9+6 Vậy k lớn Câu 2.10 Cho b = c ta được: 2  3  b +k +k ≥ k+ 2b b+a a Cho a → ta được: √ −1 − k ≤ 4√ 8k (k + 1)2 ≥ (2k + 1)3 ⇔ 4k + 2k ≥ ⇔   −1 + k≥  Ta chứng minh đây là toàn giá trị k cần tìm Đặt x = 2b 2c 2a ,y= ,z= thì b+c c+a a+b xy + yz + zx + xyz = Ta cần chứng minh: (x + 2k) (y + 2k) (z + 2k) ≥ (2k + 1)3 ⇔ xyz + 2k (xy + yz + zx) + 4k (x + y + z) + 8k ≥ (2k + 1)3 Theo kết VMO 1996, ta được: x + y + z ≥ xy + yz + zx Từ đó: xyz + 2k (xy + yz + zx) + 4k (x + y + z) + 8k ≥ xyz + 2k (xy + yz + zx) + 4k (xy + yz + zx) + 8k  = (xy + yz + zx + xyz) + 4k + 2k − (xy + yz + zx) + 8k  = 8k + + 4k + 2k − (xy + yz + zx) 165 (166) BÀI TOÁN TÌM HẰNG SỐ TỐT NHẤT Hơn ta có: q q xy + yz + zx ≥ (xyz)2 ⇒ xyz ≤ (xy + yz + zx)3 √ 3 Từ đó: q = xy + yz + zx + xyz ≤ xy + yz + zx + với t = (xy + yz + zx)3 t3 √ = t2 + √ , 3 3 √ √ xy + yz + zx Suy ra: t ≥ 3⇒ xy + yz + zx ≥ Suy ra: xyz + 2k (xy + yz + zx) + 4k (x + y + z) + 8k  ≥ 8k + + 4k + 2k − (xy + yz + zx)  ≥ 8k + + 4k + 2k − = (2k + 1)3 Bài toán hoàn tất Câu 2.11 Trước hết ta tìm điều kiện cần cho k Thay a = b = x, c = với x > 0, x 6= 1, bất đẳng thức đã cho có thể viết lại thành x k k ≥3+ 2x + + x  x2 2 − ⇔ x + −3≥k x 2x3 + x2 + x2 + + x (x − 1)2 (x + 2) k(x − 1)2 (2x + 1) ≥ x (2x3 + x2 + 1) k (x + 2) (2x3 + x2 + 1) ⇔ ≤ x (2x + 1) k ⇔ ≤ x2 + 2x + − , ∀x > 0, x 6= x 2x + ⇔ √ 3+1 Bây giờ, ta cho x = và tính giá trị biểu thức f (x) = x2 + 2x + − x 2x + bên vế phải Để ý x là nghiệm phương trình, dó ta có x2 = = − 4x Suy 2x + 4x + 1 , = 8x − 4, x 4x + + 2x + (8x − 4) − (4 − 4x) √ √ 95 45 45 − 50 45 + 95 = x− = · − = 8 f (x) = Với kết này, ta thu √ √ 45 − 50 k≤ = 45 × − 25 ≈ 45 × 0.866 − 25 ≈ 13.97 2 166 (167) BÀI TOÁN TÌM HẰNG SỐ TỐT NHẤT Mặt khác, vì k là số nguyên nên từ đây có k ≤ 13 Tiếp theo, ta chứng minh k = 13 thỏa mãn yêu cầu bài toán, tức là 1 13 25 + + + ≥ a b c a+b+c+1 Đặt f (a,b,c) = 1 13 + + + a b c a+b+c+1 Không tính tổng quát, giả sử a = max{a,b,c}, ta có     √ √  1 1 √ f (a,b,c) − f a, bc, bc = + −√ + 13 − b c a + b + c + a + bc + bc   √  √ 13   = b− c  − bc (a + b + c + 1) a + 2√bc + Do a = max{a; b; c} và giả thiết abc = nên ta có bc ≤ 1, suy ≥ Mặt khác, sử dụng bất bc đẳng thức AM - GM, ta lại có 13 13 13  ≤ √  √ = < √ 16 (a + b + c + 1) a + bc + 3 abc + 3 abc + Từ đây ta đưa bài toán chứng minh  f  25 ,x,x ≥ x √ với x = bc, < x ≤ Nếu x = thì bất đẳng thức trên trở thành đẳng thức Trong trường hợp < x < cách sử dụng lại biến đổi đã thực quá trình tìm điều kiện cho k, ta thấy bất đẳng thức tương đương với 13 (x + 2) (2x3 + x2 + 1) ≥ x (2x + 1)  ⇔ (x + 2) 2x + x + ≥ 13x (2x + 1)  ⇔ 2x4 + 5x3 + 2x2 + x + ≥ 26x2 + 13x ⇔ 8x4 + 20x3 − 18x2 − 9x + ≥ Ta có    8x4 + 20x3 − 18x2 − 9x + = 8x4 − 8x2 + + 20x3 − 20x2 + 5x + 10x2 − 14x + 2  = 2x2 − + 5x (2x − 1)2 + 5x2 − 7x + > Do (2x2 − 1) ≥ 0,5 · x (2x − 1)2 ≥ và 5x2 − 7x + > (tam thức bất hai có hệ số cao dương và biệt thức ∆ = −11 < 0) Như vậy, bất đẳng thức cuối cùng hiển nhiên đúng Vậy k = 13 là giá trị cần tìm 167 (168) BÀI TOÁN TÌM HẰNG SỐ TỐT NHẤT ta t2   2t6 + t3 + 2 +t ⇔ + 3k ≥ (k + 1) · 2t + + 3k ≥ (k + 1) t t t4 t     6 3 t + 2t + −t + 2t3 − t +2 3t − t − ≥ − ≥ ⇔ k 3− ⇔ k t t t4 t t4 Câu 2.12 Cho a = b = t > 0, c = (t3 − 1) ⇔ k (t − 1) (t + 2) ≤ t3 2 (t + t + 1) ⇔ k (t + 2) ≤ t3 2  1 1+ + 2 (t2 + t + 1) t t ⇔k≤ ⇔ k ≤ t4 + 2t3 1+ t Đặt z = Khi đó t k≤ (z + z + 1) 2z + Khi t → +∞ thì z → Như kmax = f (z) , [0;+∞) với f (z) = (z + z + 1) , với z ≥ Ta có 2z + (z + z + 1) (2z + 1)2 − (z + z + 1) (2z + 1)2   (z + z + 1) (2z + 1)2 − = ≥ 0, ∀z ≥ (2z + 1)2 f (z) = Suy hàm số f đồng biến trên [0; +∞), đó với z ≥ thì f (z) ≥ f (0) = Do đó k ≤ Từ đó kmax =   1 2 + + a +b +c +3≥2 a b c ⇔ a2 + b2 + c2 + ≥ (ab + bc + ca) (do abc = 1) ⇔ a2 + b2 + c2 + 2abc + ≥ (ab + bc + ca) Đây là kết quen thuộc chứng minh nguyên lí Diricle (ta có thể dùng dồn biến để chứng minh) Dấu đẳng thức xảy (a,b,c) = (1; 1; 1); a = b → +∞, c → và các hoán vị Câu 2.13 Thông thường dạng toán này, đẳng thức đạt ngoài giá trị tâm (1; 1; 1) thì còn đạt hai biến Như vạy để chặn k, ta chọn a = b = t, c = 3−2t Ta có:  + − ≥ k 2t2 + (3 − 2t)2 − t − 2t  − 4t + t − (3t − 2t2 ) ⇔ ≥ k 6t − 12t + 3t − 2t2 6t2 − 12t + ⇔k≤ ⇔k≤ 2 (3t − 2t ) (6t − 12t + 6) t (3 − 2t) 168 (169) BÀI TOÁN TÌM HẰNG SỐ TỐT NHẤT Như giá trị c = − 2t lớn cần tìm chính là giá trị lớn hàm số   f (t) = , t ∈ 0; t (3 − 2t) Theo bất đẳng thức AM - GM, ta có t (3 − 2t) = Từ đó kết luận kmax = 1 (2t + − 2t)2 · 2t (3 − 2t) ≤ · = 2 8 chứng tỏ  1 + + −3≥ a + b2 + c2 − a b c Đặt f (a,b,c) = 1 + + − − (a2 + b2 + c2 − 3) Xét hiệu a b c   a+b a+b f (a,b,c) − f , ,c 2  1 = + + −3− a + b2 + c2 − − a b c ! (a + b)2 − − +3+ + c2 − a+b c !   1 (a + b) = + − − a2 + b2 − a b a+b (a + b)2 − 4ab a2 + b2 − 2ab = − · ab (a + b)   = (a − b)2 − ab (a + b) Ta giả sử c = max {a,b,c} Khi đó a + b ≤ 2, suy ab (a + b) ≤ (a + b)3 ≤ < 4 Như  f (a,b,c) ≥ f  a+b a+b , ,c 2 Ta cần chứng minh   a+b a+b a+b f , ,c ≥ ⇔ f (t,t,3 − 2t) ≥ 0, với t = 2 Tức là chứng minh  + −3≥ 2t2 + (3 − 2t)2 − t − 2t 24t − 16t2 − ⇔ ≤ ⇔ ≤0 t (3 − 2t) t (3 − 2t) 16t2 − 24t + (4t − 3)2 ⇔ ≥0⇔ ≥ (đúng) t (3 − 2t) t (3 − 2t) Vậy (∗) chứng minh Từ đó kết luận kmax = 169 (1) (170) BÀI TOÁN TÌM HẰNG SỐ TỐT NHẤT Câu 2.14 Kí hiệu (1) là bất đẳng thức đã nêu đề bài Giả sử k là số thực cho bất đẳng thức (1) đúng với ba số thực a, b, c, mà abc ≥ Trong (1) thay a = 0, b = c = 2, ta + 8k ≥ Suy k ≥ Ta chứng minh k = là giá trị nhỏ cần tìm, tức chứng minh với ba số thực không âm a, b, c, ta luôn có abc +  (a − b)2 + (b − c)2 + (c − a)2 + ≥ a + b + c hay a2 + b2 + c2 + 2abc + ≥ ab + bc + ca + (a + b + c) (2) Xét số (b − 1) (c − 1), (c − 1) (a − 1), (a − 1) (b − 1), ta có a (b − 1) (c − 1) · b (c − 1) (a − 1) · c (a − 1) (b − 1) = abc (a − 1)2 (b − 1)2 (c − 1) ≥ Suy có ít số số nêu trên không âm Không tính tổng quát, giả sử a (b − 1) (c − 1) ≥ Khi đó, ta có abc ≥ a (b + c − 1) Do vậy, bất đẳng thức (2) chứng minh, ta chứng minh a2 + b2 + c2 + 2a (b + c − 1) + ≥ ab + bc + ca + (a + b + c) , hay a2 − (4 − b − c) a + b2 + c2 − bc − (b + c) + ≥ 0, (3) với a, b, c, mà abc ≥ Ta xem vế trái (3) là tam thức bậc hai theo ẩn a, ta có: ∆ = (b + c − 4)2 − 4b2 − 4c2 + 4bc + (b + c) − 16 = −3b2 − 3c2 + 6bc = −3 (b − c)2 ≤ 0, với b, c nên (3) đúng với a, b, c, mà abc ≥ Vậy k = là giá trị cần tìm, theo yêu cầu đề bài Câu 2.15 Rõ ràng, các giá trị x = y = z = thỏa mãn ràng buộc nêu đề bài Vì thế, bất đẳng thức đề bài, cho x = y = z = ta k + ≥ Suy k ≥ Tiếp theo, ta chứng minh với k = 1, bất đẳng thức đề bài là bất đẳng thức đúng; tức ta chứng minh  2 p x+y+z 2 (x + 1) (y + 1) (z + 1) ≤ + 1, (1) với số thực dương x, y, z mà {xy,yz,zx} ≥ Thật vậy, trước hết, ta chứng minh nhận xét sau: Nhận xét: Với a, b là hai số thực dương thỏa mãn ab ≥ 1, ta luôn có: ! 2    a + b a2 + b + ≤ +1  2 a+b Chứng minh: Vì ab ≥ nên − ≥ ab − ≥ Do đó 2 a +1   2 b + = (ab − 1) + (a + b) ≤  a+b 170 2 !2 −1 + (a + b) =  a+b !2 2 +1 (171) BÀI TOÁN TÌM HẰNG SỐ TỐT NHẤT Nhận xét chứng minh Không tổng quát, giả sử x ≤ y ≤ z Khi đó, từ ràng buộc đề bài, suy x ≥ x+y+z Đặt t = Ta có xt = x2 + xy + xz 1+1+1 x (x + y + z) = ≥ = 3 Do đó, áp dụng nhận xét cho cặp (x,t) và cặp (y,z), ta 2   t +1 ≤  z2 + ≤ x +1  x+t 2 y+z 2 !2 +1 và y2 +   , (2) (3) !2 +1 Nhân (2) và (3), vế theo vế, ta x2 + Nhận thấy  y2 +  z2 +  t2 + ≤   x+t !2  2 +1 y+z !2 2 +1 x+t y+z √ √ · ≥ xt · yz ≥ Do đó, theo nhận xét, ta có 2 !2  !2 !4 2 2 2   y+z x+y+z+t x+t +1 +1 ≤ +1 2 (4) (5) Từ (4) và (5), suy x2 +  y2 +  z2 +   4 t2 + ≤ t2 + Do đó p  (x2 + 1) (y + 1) (z + 1) (t2 + 1) ≤ x+y+z 2 + Suy (1) chứng minh và vì thế, giá trị k nhỏ cần tìm theo yêu cầu bài là k = Câu 2.16 Kí hiệu (1) là bất đẳng thức đã nêu đề bài Giả sử k là số thực cho bất đẳng thức (1) đúng với ba số thực a, b, c là độ dài ba cạnh tam giác Trong (1) thay b = c > ta a 2b 2ab + b2 + +k· ≤ + k 2b a + b a + 2b2 k ≤ + k Suy k ≥ 2 Ta chứng minh k = là giá trị nhỏ cần tìm, tức chứng minh với ba số thực a, b, c là độ dài ba cạnh tam giác, ta luôn có Cố định b và cho a tiến tới 0+ ta + a b c ab + bc + ca + + + ≤ b + c c + a a + b a + b2 + c2 171 (2) (172) BÀI TOÁN TÌM HẰNG SỐ TỐT NHẤT Thật vậy, ta có:       b c ab + bc + ca a + 1− + 1− ≥ + (2) ⇔ − b+c c+a a+b a + b2 + c2 b+c−a c+a−b a+b−c (a + b + c)2 ⇔ + + ≥ b+c c+a a+b (a2 + b2 + c2 ) (b + c − a)2 (c + a − b)2 (a + b − c)2 (a + b + c)2 ⇔ + + ≥ (b + c) (b + c − a) (c + a) (c + a − b) (a + b) (a + b − c) (a2 + b2 + c2 ) (3) Do a, b, c là độ dài ba cạnh tam giác nên b + c − a > 0, c + a − b > 0, a + b − c > Do đó tất các phân thức nằm vế trái (3) có mẫu thức dương Vì thế, kí hiệu V T là biểu thức nằm vế trái (3), theo bất đẳng thức Cauchy – Schwarz dạng Engel, ta có: VT ≥ (a + b + c)2 (a2 + b2 + c2 ) Vì (b + c − a) + (c + a − b) + (a + b − c) = a + b + c và  (b + c − a) (b + c) + (c + a − b) (c + a) + (a + b − c) (a + b) = a2 + b2 + c2 Nên (3) chứng minh và vì (2) chứng minh Vậy k = là giá trị cần tìm, theo yêu cầu đề bài thỏa mãn ràng buộc nêu đề bài Vì thế, 9k + ≤ , mà k > nên suy bất đẳng thức đề bài, cho x = y = z = ta 3k + 2 k ≥ Tiếp theo, ta chứng minh với k = , bất đẳng thức đề bài là bất đẳng thức đúng; tức ta chứng minh 1 + + ≤ 3, (1) 1 x+y+ y+z+ z+x+ 3 √ √ √ với số thực dương x, y, z mà xy + yz + zx = Thật vậy, đặt √ √ 2 √ T = x + y + z = x + y + z + Câu 2.17 Rõ ràng, các giá trị x = y = z = Áp dụng bất đẳng thức Cauchy-Schwarz cho hai số dương, ta có     2 √ 1 1 1 √ T √ x+y+ + +z ≥ x· √ + y· √ + √ · z = 3 3 3 hay x+y+ ≤ 3z + T Tương tự ta có: y+z+ ≤ 3x + ; T z+x+ 172 ≤ 3y + T (173) BÀI TOÁN TÌM HẰNG SỐ TỐT NHẤT Cộng bất đẳng thức vừa nêu trên, vế với vế, với lưu ý T = x + y + z + 2, ta x+y+ + y+z+ + z+x+ ≤ 3z + + 3x + + 3y + = T (1) chứng minh Vậy k = là giá trị cần tìm, theo yêu cầu đề bài Câu 2.18 Kí hiệu (1) là bất đẳng thức cần chứng minh Trong (1) cho a = b = c = 1, suy k ≤ 729 Ta chứng minh (1) đúng với k = 729 Từ giả thiết ta có q 3 |abc| ≥ |ab| + |bc| + |ca| ≥ (abc)2 ⇒ abc ≥ Do đó       c + a2 + b2 b + a2 + c2 a + b2 + c2    ≥ a2 + |bc| b2 + |ac| c2 + |ab| ≥ 93 (abc)2 ≥ 729 Sử dụng bất đẳng thức Cauchy cho chín số a2 và số |bc| Vậy max k = 729 Câu 2.19 Gọi (1) là bất đẳng thức cần tìm Trong (1) cho a1 = 1; a2 = · · · = an = − ta n−1 n c ≤ n Ta chứng minh BĐT (1) đúng với c = Tức là cần chứng minh n X nX |ai − aj | ≥ |ai | (2) i=1 1≤i<j≤n Ta có thể giả sử a1 ≥ a2 ≥ · · · ≥ an đó tồn k với (1 ≤ k ≤ n − 1) cho a1 ≥ a2 ≥ · · · ≥ ak ≥ ≥ ak+1 ≥ · · · ≥ an Khi đó ta có a1 + a2 + · · · + ak = − (ak+1 + · · · + an ) và X |ai − aj | = 1≤i<j≤n X 1≤i<j≤n (ai − aj ) = n X (n + − 2i) ; i=1 n X |ai | = i=1 n X i=1 BĐT (2) trở thành n X i=1 (n + − 2i) ≥ n k X i=1 ⇔ k X (n + − 2i) + i=1 n X (n + − 2i) ≥ n k X i=k+1 Ta có a1 ≥ a2 ≥ · · · ≥ ak và n + − 2.1 ≥ · · · ≥ n + − 2.k Áp dụng bất đẳng thức Chebychev ta " k ! k !# ! k k X X X X (n + − 2i) ≥ (n + − 2i) = (n − k) k i=1 i=1 i=1 i=1 (3) i=1 (4) Tương tự thì ak+1 ≥ · · · ≥ an và n + − 2(k + 1) ≥ · · · ≥ n + − 2.n nên " ! !# ! ! n n n n k X X X X X (n + − 2i) ≥ (n + − 2i) = −k = k n−k i=1 i=k+1 i=k+1 i=k+1 i=k+1 Từ (4) và (5) suy BĐT (3) đúng n Vậy max cn = 173 (174) BÀI TOÁN TÌM HẰNG SỐ TỐT NHẤT Câu 2.20 Ta chứng minh max ak < và max bk < k=1,n k=1,n n−1 Thật vậy, đặt L = max ak , từ (ii) suy L2 ≤ + L ⇒ L < k=1,n Đặt bm = max bk , từ (i) dễ thấy k=1,n bk ≥ (k − 1) bm + (m − k) b1 , ≤ k ≤ m và m−1 (k − m) bn + (n − k) bm , m ≤ k ≤ n n−m (n − k) bm ≤ k ≤ m và bk ≥ m ≤ k ≤ n n−m bk ≥ Suy bk > (k − 1) bm m−1 Do đó ! m X 1= (k − 1) bm bk = bk + bk > m − k=1 k=1 k=1 k=m+1 ! n X n−1 + bm ⇒ bm < (n − k) bm = n − m k=m+1 n−1 n X m X n X P √ Đặt x0 = 1; xk = 1+ ki=1 bi , k = 1,n thì xk −xk−1 = ak bk Từ (ii) suy a2k ≤ xk ⇒ ak ≤ xk Từ max ak < suy xk − xk−1 < 2bk Do đó với k = 1,n thì k=1,n √ ! √ xk xk − xk−1 = bk + √  xk + xk−1 2 √xk + √xk−1     1 bk + < bk + < bk 2 (n − 1) √ xk − xk−1 < bk √ Lấy tổng k từ đến n ta an ≤ √ n X √ xn < x0 + bk k=1  1 + 2 (n − 1)  = + 2 (n − 1) Cho n → +∞ ta M = an ≤ k Cuối cùng chọn ak = + ; bk = thỏa các điều kiện đề bài Vậy max M = 2n n 174 (175)

Ngày đăng: 06/06/2021, 03:34

w